MedSurg Midterm Practice ?s
5. A patient is brought to the ED by her family after falling off the roof. A family member tells the nurse that when the patient fell she was knocked out, but came to and seemed okay. Now she is complaining of a severe headache and not feeling well. The care team suspects an epidural hematoma, prompting the nurse to prepare for which priority intervention? A) Insertion of an intracranial monitoring device B) Treatment with antihypertensives C) Emergency craniotomy D) Administration of anticoagulant therapy
C
A clinic nurse is caring for a patient with suspected gout. While explaining the pathophysiology of gout to the patient, the nurse should describe which of the following? A)Autoimmune processes in the joints B)Chronic metabolic acidosis C)Increased uric acid levels D)Unstable serum calcium levels
C. Increased uric acid levels. Gout is caused by hyperuricemia (increased serum uric acid). Gout is not categorized as an autoimmune disease and it does not result from metabolic acidosis or unstable serum calcium levels.
27. The critical care nurse is admitting a patient in myasthenic crisis to the ICU. The nurse should prioritize what nursing action in the immediate care of this patient? A) Suctioning secretions B) Facilitating ABG analysis C) Providing ventilatory assistance D) Administering tube feedings
Ans: C Feedback: Providing ventilatory assistance takes precedence in the immediate management of the patient with myasthenic crisis. It may be necessary to suction secretions and/or provide tube feedings, but they are not the priority for this patient. ABG analysis will be done, but this is not the priority. Test Bank - Brunner & Suddarth's Textbook of Medical-Surgical Nursing 14e (Hinkle 2017) 1307
24. A patient who has been experiencing numerous episodes of unexplained headaches and vomiting has subsequently been referred for testing to rule out a brain tumor. What characteristic of the patients vomiting is most consistent with a brain tumor? A) The patients vomiting is accompanied by epistaxis. B) The patients vomiting does not relieve his nausea. C) The patients vomiting is unrelated to food intake. D) The patients emesis is blood-tinged.
Ans: C Feedback: Vomiting is often unrelated to food intake if caused by a brain tumor. The presence or absence of blood is not related to the possible etiology and vomiting may or may not relieve the patients nausea.
8. A nurse caring for a patient with diabetes insipidus is reviewing laboratory results. What is an expected urinalysis finding? A) Glucose in the urine B) Albumin in the urine C) Highly dilute urine D) Leukocytes in the urine
: C Feedback: Patients with diabetes insipidus produce an enormous daily output of very dilute, water-like urine with a specific gravity of 1.001 to 1.005. The urine contains no abnormal substances such as glucose or albumin. Leukocytes in the urine are not related to the condition of diabetes insipidus, but would indicate a urinary tract infection, if present in the urine.
6. The nurse is teaching a patient that the body needs iodine for the thyroid to function. What food would be the best source of iodine for the body? A) Eggs B) Shellfish C) Table salt D) Red meat
: C Feedback: The major use of iodine in the body is by the thyroid. Iodized table salt is the best source of iodine.
14. A patient with a C5 spinal cord injury is tetraplegic. After being moved out of the ICU, the patient complains of a severe throbbing headache. What should the nurse do first? A) Check the patients indwelling urinary catheter for kinks to ensure patency. B) Lower the HOB to improve perfusion. Test Bank - Brunner & Suddarth's Textbook of Medical-Surgical Nursing 14e (Hinkle 2017) 1283 C) Administer analgesia. D) Reassure the patient that headaches are expected after spinal cord injuries.
A
38. A patient with spinal cord injury is ready to be discharged home. A family member asks the nurse to review potential complications one more time. What are the potential complications that should be monitored for in this patient? Select all that apply. A) Orthostatic hypotension B) Autonomic dysreflexia C) DVT D) Salt-wasting syndrome E) Increased ICP
A B C
When circulatory shock occurs, there is massive vasodilation causing pooling of the blood in the periphery of the body. An ICU nurse caring for a patient in circulatory shock should know that the pooling of blood in the periphery leads to what pathophysiological effect? A) Increased stroke volume B) Increased cardiac output C) Decreased heart rate D) Decreased venous return
Ans: D Feedback: Pooling of blood in the periphery results in decreased venous return. Decreased venous return results in decreased stroke volume and decreased cardiac output. Decreased cardiac output, in turn, causes decreased blood pressure and, ultimately, decreased tissue perfusion. Heart rate increases in an attempt to meet the demands of the body.
An adult patient has survived an episode of shock and will be discharged home to finish the recovery phase of his disease process. The home health nurse plays an integral part in monitoring this patient. What aspect of his care should be prioritized by the home health nurse? A) Providing supervision to home health aides in providing necessary patient care B) Assisting the patient and family to identify and mobilize community resources C) Providing ongoing medical care during the family's rehabilitation phase D) Reinforcing the importance of continuous assessment with the family
Ans: B Feedback: The home care nurse reinforces the importance of continuing medical care and helps the patient and family identify and mobilize community resources. The home health nurse is part of a team that provides patient care in the home. The nurse does not directly supervise home health aides. The nurse provides nursing care to both the patient and family, not just the family. The nurse performs continuous and ongoing assessment of the patient; he or she does not just reinforce the importance of that assessment.
8. A patient with metastatic cancer has developed trigeminal neuralgia and is taking carbamazepine (Tegretol) for pain relief. What principle applies to the administration of this medication? A) Tegretol is not known to have serious adverse effects. B) The patient should be monitored for bone marrow depression. C) Side effects of the medication include renal dysfunction. D) The medication should be first taken in the maximum dosage form to be effective.
Ans: B Feedback: The anticonvulsant agents carbamazepine (Tegretol) and phenytoin (Dilantin) relieve pain in most patients diagnosed with trigeminal neuralgia by reducing the transmission of impulses at certain nerve terminals. Side effects include nausea, dizziness, drowsiness, and aplastic anemia. Carbamazepine should be gradually increased until pain relief is obtained.
8. A patient with Parkinsons disease is undergoing a swallowing assessment because she has recently developed adventitious lung sounds. The patients nutritional needs should be met by what method? A) Total parenteral nutrition (TPN) B) Provision of a low-residue diet C) Semisolid food with thick liquids D) Minced foods and a fluid restriction
Ans: C Feedback: A semisolid diet with thick liquids is easier for a patient with swallowing difficulties to consume than is a solid diet. Low-residue foods and fluid restriction are unnecessary and counterproductive to the patients nutritional status. The patients status does not warrant TPN.
9. The OR nurse is explaining to a patient that cardiac surgery requires the absence of blood from the surgical field. At the same time, it is imperative to maintain perfusion of body organs and tissues. What technique for achieving these simultaneous goals should the nurse describe? A) Coronary artery bypass graft (CABG) B) Percutaneous transluminal coronary angioplasty (PTCA) C) Atherectomy D) Cardiopulmonary bypass
Ans: D Feedback: Cardiopulmonary bypass is often used to circulate and oxygenate blood mechanically while bypassing the heart and lungs. PTCA, atherectomy, and CABG are all surgical procedures, none of which achieves the two goals listed.
32. A patient diagnosed with Bells palsy is having decreased sensitivity to touch of the involved nerve. What should the nurse recommend to prevent atrophy of the muscles? A) Blowing up balloons B) Deliberately frowning C) Smiling repeatedly Test Bank - Brunner & Suddarth's Textbook of Medical-Surgical Nursing 14e (Hinkle 2017) 1309 D) Whistling
Ans: D Feedback: Facial exercises, such as wrinkling the forehead, blowing out the cheeks, and whistling, may be performed with the aid of a mirror to prevent muscle atrophy. Blowing up balloons, frowning, and smiling are not considered facial exercises.
The nurse is auscultating the breath sounds of a patient with pericarditis. What finding is most consistent with this diagnosis? A) Wheezes B) Friction rub C) Fine crackles D) Coarse crackles
Ans: Friction rub Feedback: A pericardial friction rub is diagnostic of pericarditis. Crackles are associated with pulmonary edema and fluid accumulation, whereas wheezes signal airway constriction; neither of these occurs with pericarditis.
4. The nurse is caring for a patient who has been in a motor vehicle accident and the care team suspects that the patient has developed pleurisy. Which of the nurse's assessment findings would best corroborate this diagnosis? A) The patient is experiencing painless hemoptysis. B) The patient's arterial blood gases (ABGs) are normal, but he demonstrates increased work of breathing. C) The patient's oxygen saturation level is below 88%, but he denies shortness of breath. D) The patient's pain intensifies when he coughs or takes a deep breath.
D Feedback: The key characteristic of pleuritic pain is its relationship to respiratory movement. Taking a deep breath, coughing, or sneezing worsens the pain. The patient's ABGs would most likely be abnormal and shortness of breath would be expected.
3. The public health nurse is administering Mantoux tests to children who are being registered for kindergarten in the community. How should the nurse administer this test? A) Administer intradermal injections into the children's inner forearms. B) Administer intramuscular injections into each child's vastus lateralis. C) Administer a subcutaneous injection into each child's umbilical area. D) Administer a subcutaneous injection at a 45-degree angle into each child's deltoid.
A Feedback: The purified protein derivative (PPD) is always injected into the intradermal layer of the inner aspect of the forearm. The subcutaneous and intramuscular routes are not utilized.
28. The school nurse is giving a presentation on preventing spinal cord injuries (SCI). What should the nurse identify as prominent risk factors for SCI? Select all that apply. A) Young age B) Frequent travel C) African American race D) Male gender E) Alcohol or drug use
A D E
A nurse is planning the care of a patient who has a long history of chronic pain, which has only recently been diagnosed as fibromyalgia. What nursing diagnosis is most likely to apply to this womans care needs? A)Ineffective Role Performance Related to Pain B)Risk for Impaired Skin Integrity Related to Myalgia C)Risk for Infection Related to Tissue Alterations D)Unilateral Neglect Related to Neuropathic Pain
A) Ineffective Role Performance related to pain Typically, patients with fibromyalgia have endured their symptoms for a long period of time. The neuropathic pain accompanying FM can often impair a patients ability to perform normal roles and functions. Skin integrity is unaffected and the disease has no associated infection risk. Activity limitations may result in neglect, but not of a unilateral nature.
31. The occupational health nurse is assessing new employees at a company. What would be important to assess in employees with a potential occupational respiratory exposure to a toxin? Select all that apply. A) Time frame of exposure B) Type of respiratory protection used C) Immunization status D) Breath sounds E) Intensity of exposure
A, B, D, E Feedback: Key aspects of any assessment of patients with a potential occupational respiratory history include job and job activities, exposure levels, general hygiene, time frame of exposure, effectiveness of respiratory protection used, and direct versus indirect exposures. The patient's current respiratory status would also be a priority. Occupational lung hazards are not normally influenced by immunizations.
A patient who has been newly diagnosed with systemic lupus erythematosus (SLE) has been admitted to the medical unit. Which of the following nursing diagnoses is the most plausible inclusion in the plan of care? A)Fatigue Related to Anemia B)Risk for Ineffective Tissue Perfusion Related to Venous Thromboembolism C)Acute Confusion Related to Increased Serum Ammonia Levels D)Risk for Ineffective Tissue Perfusion Related to Increased Hematocrit
A. Fatigue related to anemia Patients with SLE nearly always experience fatigue, which is partly attributable to anemia. Ammonia levels are not affected and hematocrit is typically low, not high. VTE is not one of the central complications of SLE.
A nurse is working with a patient with rheumatic disease who is being treated with salicylate therapy. What statement would indicate that the patient is experiencing adverse effects of this drug? A)I have this ringing in my ears that just wont go away. B)I feel so foggy in the mornings and it takes me so long to wake up. C)When I eat a meal thats high in fat, I get really nauseous. D)I seem to have lost my appetite, which is unusual for me.
A. I have this ringing in my ears that just won't go away Tinnitus is associated with salicylate therapy. Salicylates do not normally cause drowsiness, intolerance of high-fat meals, or anorexia.
A patient has a diagnosis of rheumatoid arthritis and the primary care provider has now prescribed cyclophosphamide (Cytoxan). The nurses subsequent assessments should address what potential adverse effect? A) Infection B) Acute confusion C) Sedation D) Malignant hyperthermia
A. Infection When administering immunosuppressives such as Cytoxan, the nurse should be alert to manifestations of bone marrow suppression and infection. Confusion and sedation are atypical adverse effects. Malignant hyperthermia is a surgical complication and not a possible adverse effect.
A patient is undergoing diagnostic testing to determine the etiology of recent joint pain. The patient asks the nurse about the difference between osteoarthritis (OA) and rheumatoid arthritis (RA). What is the best response by the nurse? A)OA is a considered a noninflammatory joint disease. RA is characterized by inflamed, swollen joints. B)OA and RA are very similar. OA affects the smaller joints such as the fingers, and RA affects the larger, weight-bearing joints like the knees. C)OA originates with an infection. RA is a result of your bodys cells attacking one another. D)OA is associated with impaired immune function; RA is a consequence of physical damage.
A. OA is a considered a noninflammatory joint disease. RA is characterized by inflamed, swollen joints. OA is a degenerative arthritis with a noninflammatory etiology, characterized by the loss of cartilage on the articular surfaces of weight-bearing joints, with spur development. RA is characterized by inflammation of synovial membranes and surrounding structures. The diseases are not distinguished by the joints affected and neither has an infectious etiology.
The nurse is preparing to care for a patient who has scleroderma. The nurse refers to resources that describe CREST syndrome. Which of the following is a component of CREST syndrome? A) Raynauds phenomenon B) Thyroid dysfunction C) Esophageal varices D) Osteopenia
A. Raynauds Phenomenon The R in CREST stands for Raynauds phenomenon. Thyroid dysfunction, esophageal varices, and osteopenia are not associated with scleroderma.
A patients decreased mobility is ultimately the result of an autoimmune reaction originating in the synovial tissue, which caused the formation of pannus. This patient has been diagnosed with what health problem? A)Rheumatoid arthritis (RA) B)Systemic lupus erythematosus C)Osteoporosis D)Polymyositis
A. Rheumatoid arthritis (RA) In RA, the autoimmune reaction results in phagocytosis, producing enzymes within the joint that break down collagen, cause edema and proliferation of the synovial membrane, and ultimately form pannus. Pannus destroys cartilage and bone. SLE, osteoporosis, and polymyositis do not involve pannus formation.
A nurse is caring for a 78-year-old patient with a history of osteoarthritis (OA). When planning the patients care, what goal should the nurse include? A)The patient will express satisfaction with her ability to perform ADLs. B)The patient will recover from OA within 6 months. C)The patient will adhere to the prescribed plan of care. D)The patient will deny signs or symptoms of OA.
A. The patient will express satisfaction with her ability to perform ADLs. Pain management and optimal functional ability are major goals of nursing interventions for OA. Cure is not a possibility and it is unrealistic to expect a complete absence of signs and symptoms. Adherence to the plan of care is highly beneficial, but this is not the priority goal of care.
A patient has been diagnosed with a valvular disorder. The patient tells the nurse that he has read about numerous treatment options, including valvuloplasty. What should the nurse teach the patient about valvuloplasty? A) "For some patients, valvuloplasty can be done in a cardiac catheterization laboratory." B) "Valvuloplasty is a dangerous procedure, but it has excellent potential if it goes well." C) "Valvuloplasty is open heart surgery, but this is very safe these days and normally requires only an overnight hospital stay." D) "It's prudent to get a second opinion before deciding to have valvuloplasty."
Ans: "For some patients, valvuloplasty can be done in a cardiac catheterization laboratory." Feedback: Some valvuloplasty procedures do not require general anesthesia or cardiopulmonary bypass and can be performed in a cardiac catheterization laboratory or hybrid room. Open heart surgery is not required and the procedure does not carry exceptional risks that would designate it as being dangerous. Normally there is no need for the nurse to advocate for a second opinion.
26. The nurse is planning the care of a patient who has been recently diagnosed with a cerebellar tumor. Due to the location of this patients tumor, the nurse should implement measures to prevent what complication? A) Falls B) Audio hallucinations C) Respiratory depression D) Labile BP
Ans: A Feedback: A cerebellar tumor causes dizziness, an ataxic or staggering gait with a tendency to fall toward the side of the lesion, and marked muscle incoordination. Because of this, the patient faces a high risk of falls. Hallucinations and unstable vital signs are not closely associated with cerebellar tumors.
33. The nurse caring for a patient diagnosed with Parkinsons disease has prepared a plan of care that would include what goal? A) Promoting effective communication B) Controlling diarrhea C) Preventing cognitive decline D) Managing choreiform movements
Ans: A Feedback: The goals for the patient may include improving functional mobility, maintaining independence in ADLs, achieving adequate bowel elimination, attaining and maintaining acceptable nutritional status, achieving effective communication, and developing positive coping mechanisms. Constipation is more likely than diarrhea and cognition largely remains intact. Choreiform movements are related to Huntington disease.
A critical care nurse is planning assessments in the knowledge that patients in shock are vulnerable to developing fluid replacement complications. For what signs and symptoms should the nurse monitor the patient? Select all that apply. A) Hypovolemia B) Difficulty breathing C) Cardiovascular overload D) Pulmonary edema E) Hypoglycemia
Ans: B, C, D Feedback: Fluid replacement complications can occur, often when large volumes are administered rapidly. Therefore, the nurse monitors the patient closely for cardiovascular overload, signs of difficulty breathing, and pulmonary edema. Hypovolemia is what necessitates fluid replacement, and hypoglycemia is not a central concern with fluid replacement.
21. A 69-year-old patient is brought to the ED by ambulance because a family member found him lying on the floor disoriented and lethargic. The physician suspects bacterial meningitis and admits the patient to the ICU. The nurse knows that risk factors for an unfavorable outcome include what? Select all that apply. A) Blood pressure greater than 140/90 mm Hg Test Bank - Brunner & Suddarth's Textbook of Medical-Surgical Nursing 14e (Hinkle 2017) 1304 B) Heart rate greater than 120 bpm C) Older age D) Low Glasgow Coma Scale E) Lack of previous immunizations
Ans: B, C, D Feedback: Risks for an unfavorable outcome of meningitis include older age, a heart rate greater than 120 beats/minute, low Glasgow Coma Scale score, cranial nerve palsies, and a positive Gram stain 1 hour after presentation to the hospital. A BP greater than 140/90 mm Hg is indicative of hypertension, but is not necessarily related to poor outcomes related to meningitis. Immunizations are not normally relevant to the course of the disease.
A patient has been admitted to the medical unit with signs and symptoms suggestive of endocarditis. The physician's choice of antibiotics would be primarily based on what diagnostic test? A) Echocardiography B) Blood cultures C) Cardiac aspiration D) Complete blood count
Ans: Blood cultures Feedback: To help determine the causative organisms and the most effective antibiotic treatment for the patient, blood cultures are taken. A CBC can help establish the degree and stage of infection, but not the causative microorganism. Echocardiography cannot indicate the microorganisms causing the infection. "Cardiac aspiration" is not a diagnostic test.
26. The nurse is caring for patient who tells the nurse that he has an angina attack beginning. What is the nurses most appropriate initial action? A) Have the patient sit down and put his head between his knees. B) Have the patient perform pursed-lip breathing. C) Have the patient stand still and bend over at the waist. D) Place the patient on bed rest in a semi-Fowlers position.
Ans: D Feedback: When a patient experiences angina, the patient is directed to stop all activities and sit or rest in bed in a semi-Fowlers position to reduce the oxygen requirements of the ischemic myocardium. Pursed-lip breathing and standing will not reduce workload to the same extent. No need to have the patient put his head between his legs because cerebral perfusion is not lacking.
A nurse is caring for a patient who is suspected of having giant cell arteritis (GCA). What laboratory tests are most useful in diagnosing this rheumatic disorder? Select all that apply. A)Erythrocyte count B)Erythrocyte sedimentation rate C)Creatinine clearance D)C-reactive protein E)D-dimer
B, D Simultaneous elevation in the ESR and CRP have a sensitivity of 88% and a specificity of 98% in making the diagnosis of GCA when coupled with clinical findings. Erythrocyte counts, creatinine clearance, and D-dimer are not diagnostically useful.
A patient with polymyositisis experiencing challenges with activities of daily living as a result of proximal muscle weakness. What is the most appropriate nursing action? A)Initiate a program of passive range of motion exercises B)Facilitate referrals to occupational and physical therapy C)Administer skeletal muscle relaxants as ordered D)Encourage a progressive program of weight-bearing exercise
B. Facilitate referrals to occupational and physical therapy. Patients with polymyositis may have symptoms similar to those of other inflammatory diseases. However, proximal muscle weakness is characteristic, making activities such as hair combing, reaching overhead, and using stairs difficult. Therefore, use of assistive devices may be recommended, and referral to occupational or physical therapy may be warranted. The muscle weakness is a product of the disease process, not lack of exercise. Skeletal muscle relaxants are not used in the treatment of polymyositis.
A patient has just been told by his physician that he has scleroderma. The physician tells the patient that he is going to order some tests to assess for systemic involvement. The nurse knows that priority systems to be assessed include what? A) Hepatic B) Gastrointestinal C) Genitourinary D) Neurologic
B. Gastrointestinal Assessment of systemic involvement with scleroderma requires a systems review with special attention to gastrointestinal, pulmonary, renal, and cardiac systems. Liver, GU, and neurologic functions are not central priorities.
A patient with rheumatic disease is complaining of stomatitis. The nurse caring for the patient should further assess the patient for the adverse effects of what medications? A) Corticosteroids B) Gold-containing compounds C) Antimalarials D) Salicylate therapy
B. Gold-containing compounds Stomatitis is an adverse effect that is associated with gold therapy. Steroids, antimalarials, and salicylates do not normally have this adverse effect.
A nurse is providing care for a patient who has a rheumatic disorder. The nurses comprehensive assessment includes the patients mood, behavior, LOC, and neurologic status. What is this patients most likely diagnosis? A)Osteoarthritis (OA) B)Systemic lupus erythematosus (SLE) C)Rheumatoid arthritis (RA) D)Gout
B. Systemic Lupus erythematosus (SLE) SLE has a high degree of neurologic involvement, and can result in central nervous system changes. The patient and family members are asked about any behavioral changes, including manifestations of neurosis or psychosis. Signs of depression are noted, as are reports of seizures, chorea, or other central nervous system manifestations. OA, RA, and gout lack this dimension.
A nurse is creating a teaching plan for a patient who has a recent diagnosis of scleroderma. What topics should the nurse address during health education? Select all that apply. A)Surgical treatment options B)The importance of weight loss C)Managing Raynauds-type symptoms D)Smoking cessation E)The importance of vigilant skin care
C, D, E Patient teaching for the patient with scleroderma focuses on management of Raynauds phenomenon, smoking cessation, and meticulous skin care. Surgical treatment options do not exist and weight loss is not a central concern.
A patient with rheumatoid arthritis comes into the clinic for a routine check-up. On assessment the nurse notes that the patient appears to have lost some of her ability to function since her last office visit. Which of the following is the most appropriate action? A)Arrange a family meeting in order to explore assisted living options. B)Refer the patient to a support group. C)Arrange for the patient to be assessed in her home environment. D)Refer the patient to social work.
C. Arrange for the patient to be assessed in her home environment Assessment in the patients home setting can often reveal more meaningful data than an assessment in the health care setting. There is no indication that assisted living is a pressing need or that the patient would benefit from social work or a support group.
33. A nurse on the neurologic unit is providing care for a patient who has spinal cord injury at the level of C4. When planning the patients care, what aspect of the patients neurologic and functional status should the nurse consider? A) The patient will be unable to use a wheelchair. B) The patient will be unable to swallow food. C) The patient will be continent of urine, but incontinent of bowel. D) The patient will require full assistance for all aspects of elimination.
D
7. An ED nurse has just received a call from EMS that they are transporting a 17-year-old man who has just sustained a spinal cord injury (SCI). The nurse recognizes that the most common cause of this type of injury is what? A) Sports-related injuries B) Acts of violence C) Injuries due to a fall D) Motor vehicle accidents
D
11. A patient who suffered a spinal cord injury is experiencing an exaggerated autonomic response. What aspect of the patients current health status is most likely to have precipitated this event? A) The patient received a blood transfusion. B) The patients analgesia regimen was recent changed. C) The patient was not repositioned during the night shift. D) The patients urinary catheter became occluded.
D
13. A patient with a head injury has been increasingly agitated and the nurse has consequently identified a risk for injury. What is the nurses best intervention for preventing injury? A) Restrain the patient as ordered. B) Administer opioids PRN as ordered. C) Arrange for friends and family members to sit with the patient. D) Pad the side rails of the patients bed.
D
21. The ED is notified that a 6-year-old is in transit with a suspected brain injury after being struck by a car. The child is unresponsive at this time, but vital signs are within acceptable limits. What will be the primary goal of initial therapy? A) Promoting adequate circulation B) Treating the childs increased ICP C) Assessing secondary brain injury D) Preserving brain homeostasis
D
23. A 13-year-old was brought to the ED, unconscious, after being hit in the head by a baseball. When the child regains consciousness, 5 hours after being admitted, he cannot remember the traumatic event. MRI shows no structural sign of injury. What injury would the nurse suspect the patient has? A) Diffuse axonal injury B) Grade 1 concussion with frontal lobe involvement C) Contusion D) Grade 3 concussion with temporal lobe involvement
D
27. The nurse has implemented interventions aimed at facilitating family coping in the care of a patient with a traumatic brain injury. How can the nurse best facilitate family coping? A) Help the family understand that the patient could have died. B) Emphasize the importance of accepting the patients new limitations. C) Have the members of the family plan the patients inpatient care. D) Assist the family in setting appropriate short-term goals
D
A clinic nurse is caring for a patient newly diagnosed with fibromyalgia. When developing a care plan for this patient, what would be a priority nursing diagnosis for this patient? A)Impaired Urinary Elimination Related to Neuropathy B)Altered Nutrition Related to Impaired Absorption C)Disturbed Sleep Pattern Related to CNS Stimulation D)Fatigue Related to Pain
D. Fatigue Related to Pain Fibromyalgia is characterized by fatigue, generalized muscle aching, and stiffness. Impaired urinary elimination is not a common manifestation of the disease. Altered nutrition and disturbed sleep pattern are potential nursing diagnoses, but are not the priority.
A nurses plan of care for a patient with rheumatoid arthritis includes several exercise-based interventions. Exercises for patients with rheumatoid disorders should have which of the following goals? A)Maximize range of motion while minimizing exertion B)Increase joint size and strength C)Limit energy output in order to preserve strength for healing D)Preserve and increase range of motion while limiting joint stress
D. Preserve and increase range of motion while limiting joint stress Exercise is vital to the management of rheumatic disorders. Goals should be preserving and promoting mobility and joint function while limiting stress on the joint and possible damage. Cardiovascular exertion should remain within age-based limits and individual ability, but it is not a goal to minimize exertion. Increasing joint size is not a valid goal.
A patients rheumatoid arthritis (RA) has failed to respond appreciably to first-line treatments and the primary care provider has added prednisone to the patients drug regimen. What principle will guide this aspect of the patients treatment? A)The patient will need daily blood testing for the duration of treatment. B)The patient must stop all other drugs 72 hours before starting prednisone. C)The drug should be used at the highest dose the patient can tolerate. D)The drug should be used for as short a time as possible.
D. The drug should be used for as short a time as possible. Corticosteroids are used for shortest duration and at lowest dose possible to minimize adverse effects. Daily blood work is not necessary and the patient does not need to stop other drugs prior to using corticosteroids.
A community health nurse is performing a visit to the home of a patient who has a history of rheumatoid arthritis (RA). On what aspect of the patients health should the nurse focus most closely during the visit? A)The patients understanding of rheumatoid arthritis B)The patients risk for cardiopulmonary complications C)The patients social support system D)The patients functional status
D. The patient's functional status The patients functional status is a central focus of home assessment of the patient with RA. The nurse may also address the patients understanding of the disease, complications, and social support, but the patients level of function and quality of life is a primary concern.
34. The nurse is providing health education to a patient who has a C6 spinal cord injury. The patient asks why autonomic dysreflexia is considered an emergency. What would be the nurses best answer? A) The sudden increase in BP can raise the ICP or rupture a cerebral blood vessel. B) The suddenness of the onset of the syndrome tells us the body is struggling to maintain its normal state. C) Autonomic dysreflexia causes permanent damage to delicate nerve fibers that are healing. D) The sudden, severe headache increases muscle tone and can cause further nerve damage
A
4. The nurse is caring for a patient with increased intracranial pressure (ICP) caused by a traumatic brain injury. Which of the following clinical manifestations would suggest that the patient may be experiencing increased brain compression causing brain stem damage? A) Hyperthermia B) Tachycardia C) Hypertension D) Bradypnea
A
6. A patient has been brought to the ED by the paramedics. The patient is suspected of having ARDS. What intervention should the nurse first anticipate? A) Preparing to assist with intubating the patient B) Setting up oxygen at 5 L/minute by nasal cannula C) Performing deep suctioning D) Setting up a nebulizer to administer corticosteroids
A Feedback: A patient who has ARDS usually requires intubation and mechanical ventilation. Oxygen by nasal cannula would likely be insufficient. Deep suctioning and nebulizers may be indicated, but the priority is to secure the airway.
17. While planning a patient's care, the nurse identifies nursing actions to minimize the patient's pleuritic pain. Which intervention should the nurse include in the plan of care? A) Avoid actions that will cause the patient to breathe deeply. B) Ambulate the patient at least three times daily. C) Arrange for a soft-textured diet and increased fluid intake. D) Encourage the patient to speak as little as possible
A Feedback: The key characteristic of pleuritic pain is its relationship to respiratory movement. Taking a deep breath, coughing, or sneezing worsens the pain. A soft diet is not necessarily indicated and there is no need for the patient to avoid speaking. Ambulation has multiple benefits, but pain management is not among them.
33. A patient with Cushing syndrome has been hospitalized after a fall. The dietician consulted works with the patient to improve the patients nutritional intake. What foods should a patient with Cushing syndrome eat to optimize health? Select all that apply. A) Foods high in vitamin D B) Foods high in calories C) Foods high in protein D) Foods high in calcium E) Foods high in sodium
: A, C, D Feedback: Foods high in vitamin D, protein, and calcium are recommended to minimize muscle wasting and osteoporosis. Referral to a dietitian may assist the patient in selecting appropriate foods that are also low in sodium and calories.
7. A patient is prescribed corticosteroid therapy. What would be priority information for the nurse to give the patient who is prescribed long-term corticosteroid therapy? A) The patients diet should be low protein with ample fat. B) The patient may experience short-term changes in cognition. C) The patient is at an increased risk for developing infection. D) The patient is at a decreased risk for development of thrombophlebitis and thromboembolism.
: C Feedback: The patient is at increased risk of infection and masking of signs of infection. The cardiovascular effects of corticosteroid therapy may result in development of thrombophlebitis or thromboembolism. Diet should be high in protein with limited fat. Changes in appearance usually disappear when therapy is no longer necessary. Cognitive changes are not common adverse effects.
17. A patient with a T2 injury is in spinal shock. The nurse will expect to observe what assessment finding? A) Absence of reflexes along with flaccid extremities B) Positive Babinskis reflex along with spastic extremities C) Hyperreflexia along with spastic extremities D) Spasticity of all four extremities
A
22. A patient is admitted to the neurologic ICU with a suspected diffuse axonal injury. What would be the primary neuroimaging diagnostic tool used on this patient to evaluate the brain structure? A) MRI B) PET scan C) X-ray D) Ultrasound
A
24. An 82-year-old man is admitted for observation after a fall. Due to his age, the nurse knows that the patient is at increased risk for what complication of his injury? A) Hematoma B) Skull fracture C) Embolus D) Stroke
A
29. The school nurse has been called to the football field where player is immobile on the field after landing awkwardly on his head during a play. While awaiting an ambulance, what action should the nurse perform? A) Ensure that the player is not moved. B) Obtain the players vital signs, if possible. C) Perform a rapid assessment of the players range of motion. D) Assess the players reflexes.
A
31. The nurse is planning the care of a patient with a T1 spinal cord injury. The nurse has identified the diagnosis of risk for impaired skin integrity. How can the nurse best address this risk? A) Change the patients position frequently. B) Provide a high-protein diet. C) Provide light massage at least daily. D) Teach the patient deep breathing and coughing exercises.
A
10. An x-ray of a trauma patient reveals rib fractures and the patient is diagnosed with a small flail chest injury. Which intervention should the nurse include in the patient's plan of care? A) Suction the patient's airway secretions. B) Immobilize the ribs with an abdominal binder. C) Prepare the patient for surgery. D) Immediately sedate and intubate the patient.
A Feedback: As with rib fracture, treatment of flail chest is usually supportive. Management includes clearing secretions from the lungs, and controlling pain. If only a small segment of the chest is involved, it is important to clear the airway through positioning, coughing, deep breathing, and suctioning. Intubation is required for severe flail chest injuries, and surgery is required only in rare circumstances to stabilize the flail segment.
14. The nurse at a long-term care facility is assessing each of the residents. Which resident most likely faces the greatest risk for aspiration? A) A resident who suffered a severe stroke several weeks ago B) A resident with mid-stage Alzheimer's disease C) A 92-year-old resident who needs extensive help with ADLs D) A resident with severe and deforming rheumatoid arthritis
A Feedback: Aspiration may occur if the patient cannot adequately coordinate protective glottic, laryngeal, and cough reflexes. These reflexes are often affected by stroke. A patient with mid-stage Alzheimer's disease does not likely have the voluntary muscle problems that occur later in the disease. Clients that need help with ADLs or have severe arthritis should not have difficulty swallowing unless it exists secondary to another problem.
18. The perioperative nurse is writing a care plan for a patient who has returned from surgery 2 hours prior. Which measure should the nurse implement to most decrease the patient's risk of developing pulmonary emboli (PE)? A) Early ambulation B) Increased dietary intake of protein C) Maintaining the patient in a supine position D) Administering aspirin with warfarin
A Feedback: For patients at risk for PE, the most effective approach for prevention is to prevent deep vein thrombosis. Active leg exercises to avoid venous stasis, early ambulation, and use of elastic compression stocking are general preventive measures. The patient does not require increased dietary intake of protein directly related to prevention of PE, although it will assist in wound healing during the postoperative period. The patient should not be maintained in one position, but frequently repositioned, unless contraindicated by the surgical procedure. Aspirin should never be administered with warfarin because it will increase the patient's risk for bleeding.
13. The nurse is assessing an adult patient following a motor vehicle accident. The nurse observes that the patient has an increased use of accessory muscles and is complaining of chest pain and shortness of breath. The nurse should recognize the possibility of what condition? A) Pneumothorax B) Anxiety C) Acute bronchitis D) Aspiration
A Feedback: If the pneumothorax is large and the lung collapses totally, acute respiratory distress occurs. The patient is anxious, has dyspnea and air hunger, has increased use of the accessory muscles, and may develop central cyanosis from severe hypoxemia. These symptoms are not definitive of pneumothorax, but because of the patient's recent trauma they are inconsistent with anxiety, bronchitis, or aspiration.
7. The nurse is caring for a patient who is scheduled for a lobectomy for a diagnosis of lung cancer. While assisting with a subclavian vein central line insertion, the nurse notes the client's oxygen saturation rapidly dropping. The patient complains of shortness of breath and becomes tachypneic. The nurse suspects a pneumothorax has developed. Further assessment findings supporting the presence of a pneumothorax include what? A) Diminished or absent breath sounds on the affected side B) Paradoxical chest wall movement with respirations C) Sudden loss of consciousness D) Muffled heart sounds
A Feedback: In the case of a simple pneumothorax, auscultating the breath sounds will reveal absent or diminished breath sounds on the affected side. Paradoxical chest wall movements occur in flail chest conditions. Sudden loss of consciousness does not typically occur. Muffled or distant heart sounds occur in pericardial tamponade.
19. The school nurse is presenting a class on smoking cessation at the local high school. A participant in the class asks the nurse about the risk of lung cancer in those who smoke. What response related to risk for lung cancer in smokers is most accurate? A) "The younger you are when you start smoking, the higher your risk of lung cancer." B) "The risk for lung cancer never decreases once you have smoked, which is why smokers need annual chest x-rays." C) "The risk for lung cancer is determined mostly by what type of cigarettes you smoke." D) "The risk for lung cancer depends primarily on the other risk factors for cancer that you have."
A Feedback: Risk is determined by the pack-year history (number of packs of cigarettes used each day, multiplied by the number of years smoked), the age of initiation of smoking, the depth of inhalation, and the tar and nicotine levels in the cigarettes smoked. The younger a person is when he or she starts smoking, the greater the risk of developing lung cancer. Risk declines after smoking cessation. The type of cigarettes is a significant variable, but this is not the most important factor.
16. The nurse is caring for a patient at risk for atelectasis. The nurse implements a first-line measure to prevent atelectasis development in the patient. What is an example of a first-line measure to minimize atelectasis? A) Incentive spirometry B) Intermittent positive-pressure breathing (IPPB) C) Positive end-expiratory pressure (PEEP) D) Bronchoscopy
A Feedback: Strategies to prevent atelectasis, which include frequent turning, early ambulation, lung-volume expansion maneuvers (deep breathing exercises, incentive spirometry), and coughing, serve as the first-line measures to minimize or treat atelectasis by improving ventilation. In patients who do not respond to first-line measures or who cannot perform deep-breathing exercises, other treatments such as positive end-expiratory pressure (PEEP), continuous or intermittent positive-pressure breathing (IPPB), or bronchoscopy may be used.
12. An adult patient has tested positive for tuberculosis (TB). While providing patient teaching, what information should the nurse prioritize? A) The importance of adhering closely to the prescribed medication regimen B) The fact that the disease is a lifelong, chronic condition that will affect ADLs C) The fact that TB is self-limiting, but can take up to 2 years to resolve D) The need to work closely with the occupational and physical therapists
A Feedback: Successful treatment of TB is highly dependent on careful adherence to the medication regimen. The disease is not self-limiting; occupational and physical therapy are not necessarily indicated. TB is curable.
20. The nurse is assessing a patient who has a 35 pack-year history of cigarette smoking. In light of this known risk factor for lung cancer, what statement should prompt the nurse to refer the patient for further assessment? A) "Lately, I have this cough that just never seems to go away." B) "I find that I don't have nearly the stamina that I used to." C) "I seem to get nearly every cold and flu that goes around my workplace." D) "I never used to have any allergies, but now I think I'm developing allergies to dust and pet hair."
A Feedback: The most frequent symptom of lung cancer is cough or change in a chronic cough. People frequently ignore this symptom and attribute it to smoking or a respiratory infection. A new onset of allergies, frequent respiratory infections and fatigue are not characteristic early signs of lung cancer.
2. A critical-care nurse is caring for a patient diagnosed with pneumonia as a surgical complication. The nurse's assessment reveals that the patient has an increased work of breathing due to copious tracheobronchial secretions. What should the nurse encourage the patient to do? A) Increase oral fluids unless contraindicated. B) Call the nurse for oral suctioning, as needed. C) Lie in a low Fowler's or supine position. D) Increase activity.
A Feedback: The nurse should encourage hydration because adequate hydration thins and loosens pulmonary secretions. Oral suctioning is not sufficiently deep to remove tracheobronchial secretions. The patient should have the head of the bed raised, and rest should be promoted to avoid exacerbation of symptoms.
A patient has been admitted to a medical unit with a diagnosis of polymyalgia rheumatica (PMR). The nurse should be aware of what aspects of PMR? Select all that apply. A)PMR has an association with the genetic marker HLA-DR4. B)Immunoglobulin deposits occur in PMR. C)PMR is considered to be a wear-and-tear disease. D)Foods high in purines exacerbate the biochemical processes that occur in PMR. E)PMR occurs predominately in Caucasians.
A, B, E The underlying mechanism involved with polymyalgia rheumatica is unknown. This disease occurs predominately in Caucasians and often in first-degree relatives. An association with the genetic marker HLA-DR4 suggests a familial predisposition. Immunoglobulin deposits in the walls of inflamed temporal arteries also suggest an autoimmune process. Purines are unrelated and it is not a result of physical degeneration.
A patient with an exacerbation of systemic lupus erythematosus (SLE) has been hospitalized on the medical unit. The nurse observes that the patient expresses anger and irritation when her call bell isn't answered immediately. What would be the most appropriate response? A)You seem like you're feeling angry. Is that something that we could talk about? B) Try to remember that stress can make your symptoms worse. C) Would you like to talk about the problem with the nursing supervisor? D) I can see you're angry. I'll come back when you've calmed down.
A. You seem like you're feeling angry. Is that something that we could talk about? The changes and the unpredictable course of SLE necessitate expert assessment skills and nursing care, as well as sensitivity to the psychological reactions of the patient. Offering to listen to the patient express anger can help the nurse and the patient understand its cause and begin to deal with it. Although stress can exacerbate the symptoms of SLE, telling the patient to calm down doesnt acknowledge her feelings. Ignoring the patients feelings suggests that the nurse has no interest in what the patient has said. Offering to get the nursing supervisor also does not acknowledge the patients feelings.
A patient is a candidate for percutaneous balloon valvuloplasty, but is concerned about how this procedure will affect her busy work schedule. What guidance should the nurse provide to the patient? A) "Patients generally stay in the hospital for 6 to 8 days." B) "Patients are kept in the hospital until they are independent with all aspects of their care." C) "Patients need to stay in the hospital until they regain normal heart function for their age." D) "Patients usually remain at the hospital for 24 to 48 hours."
Ans: "Patients usually remain at the hospital for 24 to 48 hours." Feedback: After undergoing percutaneous balloon valvuloplasty, the patient usually remains in the hospital for 24 to 48 hours. Prediagnosis levels of heart function are not always attainable and the patient does not need to be wholly independent prior to discharge.
A patient is undergoing diagnostic testing for mitral stenosis. What statement by the patient during the nurse's interview is most suggestive of this valvular disorder? A) "I get chest pain from time to time, but it usually resolves when I rest." B) "Sometimes when I'm resting, I can feel my heart skip a beat." C) "Whenever I do any form of exercise I get terribly short of breath." D) "My feet and ankles have gotten terribly puffy the last few weeks."
Ans: "Whenever I do any form of exercise I get terribly short of breath." Feedback: The first symptom of mitral stenosis is often breathing difficulty (dyspnea) on exertion as a result of pulmonary venous hypertension. Patients with mitral stenosis are likely to show progressive fatigue as a result of low cardiac output. Palpitations occur in some patients, but dyspnea is a characteristic early symptom. Peripheral edema and chest pain are atypical.
The nurse is caring for a patient who is exhibiting signs and symptoms of hypovolemic shock following injuries suffered in a motor vehicle accident. The nurse anticipates that the physician will promptly order the administration of a crystalloid IV solution to restore intravascular volume. In addition to normal saline, which crystalloid fluid is commonly used to treat hypovolemic shock? A) Lactated Ringer's B) Albumin C) Dextran D) 3% NaCl
Ans: A Feedback: Crystalloids are electrolyte solutions used for the treatment of hypovolemic shock. Lactated Ringer's and 0.9% sodium chloride are isotonic crystalloid fluids commonly used to manage hypovolemic shock. Dextran and albumin are colloids, but Dextran, even as a colloid, is not indicated for the treatment of hypovolemic shock. 3% NaCl is a hypertonic solution and is not isotonic.
In an acute care setting, the nurse is assessing an unstable patient. When prioritizing the patient's care, the nurse should recognize that the patient is at risk for hypovolemic shock in which of the following circumstances? A) Fluid volume circulating in the blood vessels decreases. B) There is an uncontrolled increase in cardiac output. C) Blood pressure regulation becomes irregular. D) The patient experiences tachycardia and a bounding pulse.
Ans: A Feedback: Hypovolemic shock is characterized by a decrease in intravascular volume. Cardiac output is decreased, blood pressure decreases, and pulse is fast, but weak.
The nurse, a member of the health care team in the ED, is caring for a patient who is determined to be in the irreversible stage of shock. What would be the most appropriate nursing intervention? A) Provide opportunities for the family to spend time with the patient, and help them to understand the irreversible stage of shock. B) Inform the patient's family immediately that the patient will likely not survive to allow the family time to make plans and move forward. C) Closely monitor fluid replacement therapy, and inform the family that the patient will probably survive and return to normal life. D) Protect the patient's airway, optimize intravascular volume, and initiate the early rehabilitation process.
Ans: A Feedback: The irreversible (or refractory) stage of shock represents the point along the shock continuum at which organ damage is so severe that the patient does not respond to treatment and cannot survive. Providing opportunities for the family to spend time with the patient and helping them to understand the irreversible stage of shock is the best intervention. Informing the patient's family early that the patient will likely not survive does allow the family to make plans and move forward, but informing the family too early will rob the family of hope and interrupt the grieving process. The chance of surviving the irreversible (or refractory) stage of shock is very small, and the nurse needs to help the family cope with the reality of the situation. With the chances of survival so small, the priorities shift from aggressive treatment and safety to addressing the end-of-life issues.
A patient is responding poorly to interventions aimed at treating shock and appears to be transitioning to the irreversible stage of shock. What action should the intensive care nurse include during this phase of the patient's care? A) Communicate clearly and frequently with the patient's family. B) Taper down interventions slowly when the prognosis worsens. C) Transfer the patient to a subacute unit when recovery appears unlikely. D) Ask the patient's family how they would prefer treatment to proceed.
Ans: A Feedback: As it becomes obvious that the patient is unlikely to survive, the family must be informed about the prognosis and likely outcome. Opportunities should be provided, throughout the patient's care, for the family to see, touch, and talk to the patient. The onus should not be placed on the family to guide care, however. Interventions are not normally reduced gradually when they are deemed ineffective; instead, they are discontinued when they appear futile. The patient would not be transferred to a subacute unit.
The nurse in the ED is caring for a patient recently admitted with a likely myocardial infarction. The nurse understands that the patient's heart is pumping an inadequate supply of oxygen to the tissues. For what health problem should the nurse assess? A) Dysrhythmias B) Increase in blood pressure C) Increase in heart rate D) Decrease in oxygen demands
Ans: A Feedback: Cardiogenic shock occurs when the heart's ability to pump blood is impaired and the supply of oxygen is inadequate for the heart and tissues. Symptoms of cardiogenic shock include angina pain and dysrhythmias. Cardiogenic shock does not cause increased blood pressure, increased heart rate, or a decrease in oxygen demands.
The nurse in the ICU is admitting a 57-year-old man with a diagnosis of possible septic shock. The nurse's assessment reveals that the patient has a normal blood pressure, increased heart rate, decreased bowel sounds, and cold, clammy skin. The nurse's analysis of these data should lead to what preliminary conclusion? A) The patient is in the compensatory stage of shock. B) The patient is in the progressive stage of shock. C) The patient will stabilize and be released by tomorrow. D) The patient is in the irreversible stage of shock.
Ans: A Feedback: In the compensatory stage of shock, the blood pressure remains within normal limits. Vasoconstriction, increased heart rate, and increased contractility of the heart contribute to maintaining adequate cardiac output. Patients display the often-described "fight or flight" response. The body shunts blood from organs such as the skin, kidneys, and GI tract to the brain and heart to ensure adequate blood supply to these vital organs. As a result, the skin is cool and clammy, and bowel sounds are hypoactive. In progressive shock, the blood pressure drops. In septic shock, the patient's chance of survival is low and he will certainly not be released within 24 hours. If the patient were in the irreversible stage of shock, his blood pressure would be very low and his organs would be failing.
A critical care nurse is aware of the high incidence of ventilator-associated pneumonia (VAP) in patients who are being treated for shock. What intervention should be specified in the patient's plan of care while the patient is ventilated? A) Performing frequent oral care B) Maintaining the patient in a supine position C) Suctioning the patient every 15 minutes unless contraindicated D) Administering prophylactic antibiotics, as ordered
Ans: A Feedback: Nursing interventions that reduce the incidence of VAP must also be implemented. These include frequent oral care, aseptic suction technique, turning, and elevating the head of the bed at least 30 degrees to prevent aspiration. Suctioning should not be excessively frequent and prophylactic antibiotics are not normally indicated.
The ICU nurse is caring for a patient with multiple organ dysfunction syndrome (MODS) due to shock. What nursing action should be prioritized at this point during care? A) Providing information and support to family members B) Preparing the family for a long recovery process C) Educating the patient regarding the use of supportive fluids D) Facilitating the rehabilitation phase of treatment
Ans: A Feedback: Providing information and support to family members is a critical role of the nurse. Most patients with MODS do not recover, so the rehabilitation phase of recovery is not a short-term priority. Educating the patient about the use of supportive fluids is not a high priority.
13. You are precepting a new graduate nurse in the ICU. You are collaborating in the care of a patient who is receiving large volumes of crystalloid fluid to treat hypovolemic shock. In light of this intervention, for what sign would you teach the new nurse to monitor the patient? A) Hypothermia B) Bradycardia C) Coffee ground emesis D) Pain
Ans: A Feedback: Temperature should be monitored closely to ensure that rapid fluid resuscitation does not precipitate hypothermia. IV fluids may need to be warmed during the administration of large volumes. The nurse should monitor the patient for cardiovascular overload and pulmonary edema when large volumes of IV solution are administered. Coffee ground emesis is an indication of a GI bleed, not shock. Pain is related to cardiogenic shock.
14. The nurse is caring for a patient in the ICU whose condition is deteriorating. The nurse receives orders to initiate an infusion of dopamine. What would be the priority assessment and interventions specific to the administration of vasoactive medications? A) Frequent monitoring of vital signs, monitoring the central line site, and providing accurate drug titration B) Reviewing medications, performing a focused cardiovascular assessment, and providing patient education C) Reviewing the laboratory findings, monitoring urine output, and assessing for peripheral edema D) Routine monitoring of vital signs, monitoring the peripheral IV site, and providing early discharge instructions
Ans: A Feedback: When vasoactive medications are administered, vital signs must be monitored frequently (at least every 15 minutes until stable, or more often if indicated). Vasoactive medications should be administered through a central venous line because infiltration and extravasation of some vasoactive medications can cause tissue necrosis and sloughing. An IV pump should be used to ensure that the medications are delivered safely and accurately. Individual medication dosages are usually titrated by the nurse, who adjusts drip rates based on the prescribed dose and the patient's response. Reviewing medications, performing a focused cardiovascular assessment, and providing patient education are important nursing tasks, but they are not specific to the administration of IV vasoactive drugs. Reviewing the laboratory findings, monitoring urine output, and assessing for peripheral edema are not the priorities for administration of IV vasoactive drugs. Vital signs are taken on a frequent basis when monitoring administration of IV vasoactive drugs, vasoactive medications should be administered through a central venous line, and early discharge instructions would be inappropriate in this time of crisis.
The ICU nurse caring for a patient in shock is administering vasoactive medications as per orders. The nurse should know that vasoactive medications should be administered in what way? A) Through a central venous line B) By a gravity infusion IV set C) By IV push for rapid onset of action D) Mixed with parenteral feedings to balance osmosis
Ans: A Feedback: Whenever possible, vasoactive medications should be administered through a central venous line because infiltration and extravasation of some vasoactive medications can cause tissue necrosis and sloughing. An IV pump or controller must be used to ensure that the medications are delivered safely and accurately. They are never mixed with parenteral nutrition.
37. A patient with MS has been admitted to the hospital following an acute exacerbation. When planning the patients care, the nurse addresses the need to enhance the patients bladder control. What aspect of nursing care is most likely to meet this goal? A) Establish a timed voiding schedule. B) Avoid foods that change the pH of urine. C) Perform intermittent catheterization q6h. D) Administer anticholinergic drugs as ordered.
Ans: A Feedback: A timed voiding schedule addresses many of the challenges with urinary continence that face the patient with MS. Interventions should be implemented to prevent the need for catheterization and anticholinergics are not normally used.
1. A nurse is assessing a patient with an acoustic neuroma who has been recently admitted to an oncology unit. What symptoms is the nurse likely to find during the initial assessment? A) Loss of hearing, tinnitus, and vertigo B) Loss of vision, change in mental status, and hyperthermia C) Loss of hearing, increased sodium retention, and hypertension D) Loss of vision, headache, and tachycardia
Ans: A Feedback: An acoustic neuroma is a tumor of the eighth cranial nerve, the cranial nerve most responsible for hearing and balance. The patient with an acoustic neuroma usually experiences loss of hearing, tinnitus, and episodes of vertigo and staggering gait. Acoustic neuromas do not cause loss of vision, increased sodium retention, or tachycardia.
10. The nurse has just admitted a 66-year-old patient for cardiac surgery. The patient tearfully admits to the nurse that she is afraid of dying while undergoing the surgery. What is the nurses best response? A) Explore the factors underlying the patients anxiety. B) Teach the patient guided imagery techniques. C) Obtain an order for a PRN benzodiazepine. D) Describe the procedure in greater detail.
Ans: A Feedback: An assessment of anxiety levels is required in the patient to assist the patient in identifying fears and developing coping mechanisms for those fears. The nurse must further assess and explore the patients anxiety before providing interventions such as education or medications.
15. The nurse is working with a patient who had an MI and is now active in rehabilitation. The nurse should teach this patient to cease activity if which of the following occurs? A) The patient experiences chest pain, palpitations, or dyspnea. B) The patient experiences a noticeable increase in heart rate during activity. C) The patients oxygen saturation level drops below 96%. D) The patients respiratory rate exceeds 30 breaths/min.
Ans: A Feedback: Any activity or exercise that causes dyspnea and chest pain should be stopped in the patient with CAD. Heart rate must not exceed the target rate, but an increase above resting rate is expected and is therapeutic. In most patients, a respiratory rate that exceeds 30 breaths/min is not problematic. Similarly, oxygen saturation slightly below 96% does not necessitate cessation of activity.
1. The nurse is caring for a patient who has been diagnosed with an elevated cholesterol level. The nurse is aware that plaque on the inner lumen of arteries is composed chiefly of what? A) Lipids and fibrous tissue B) White blood cells C) Lipoproteins D) High-density cholesterol
Ans: A Feedback: As T-lymphocytes and monocytes infiltrate to ingest lipids on the arterial wall and then die, a fibrous tissue develops. This causes plaques to form on the inner lumen of arterial walls. These plaques do not consist of white cells, lipoproteins, or high-density cholesterol.
18. The nurse is caring for a patient who is hospitalized with an exacerbation of MS. To ensure the patients safety, what nursing action should be performed? A) Ensure that suction apparatus is set up at the bedside. B) Pad the patients bed rails. C) Maintain bed rest whenever possible. D) Provide several small meals each day.
Ans: A Feedback: Because of the patients risk of aspiration, it is important to have a suction apparatus at hand. Bed rest should be generally be minimized, not maximized, and there is no need to pad the patients bed rails or to provide multiple small meals.
16. A patient with cardiovascular disease is being treated with amlodipine (Norvasc), a calcium channel blocking agent. The therapeutic effects of calcium channel blockers include which of the following? A) Reducing the hearts workload by decreasing heart rate and myocardial contraction B) Preventing platelet aggregation and subsequent thrombosis C) Reducing myocardial oxygen consumption by blocking adrenergic stimulation to the heart D) Increasing the efficiency of myocardial oxygen consumption, thus decreasing ischemia and relieving pain
Ans: A Feedback: Calcium channel blocking agents decrease sinoatrial node automaticity and atrioventricular node conduction, resulting in a slower heart rate and a decrease in the strength of the heart muscle contraction. These effects decrease the workload of the heart. Antiplatelet and anticoagulation medications are administered to prevent platelet aggregation and subsequent thrombosis, which impedes blood flow. Beta-blockers reduce myocardial consumption by blocking beta-adrenergic sympathetic stimulation to the heart. The result is reduced myocardial contractility (force of contraction) to balance the myocardium oxygen needs and supply. Nitrates reduce myocardial oxygen consumption, which decreases ischemia and relieves pain by dilating the veins and, in higher doses, the arteries.
5. A patient diagnosed with Bells palsy is being cared for on an outpatient basis. During health education, the nurse should promote which of the following actions? Test Bank - Brunner & Suddarth's Textbook of Medical-Surgical Nursing 14e (Hinkle 2017) 1297 A) Applying a protective eye shield at night B) Chewing on the affected side to prevent unilateral neglect C) Avoiding the use of analgesics whenever possible D) Avoiding brushing the teeth
Ans: A Feedback: Corneal irritation and ulceration may occur if the eye is unprotected. While paralysis lasts, the involved eye must be protected. The patient should be encouraged to eat on the unaffected side, due to swallowing difficulties. Analgesics are used to control the facial pain. The patient should continue to provide self-care including oral hygiene.
19. A patient with Huntington disease has just been admitted to a long-term care facility. The charge nurse is creating a care plan for this patient. Nutritional management for a patient with Huntington disease should be informed by what principle? A) The patient is likely to have an increased appetite. B) The patient is likely to required enzyme supplements. C) The patient will likely require a clear liquid diet. D) The patient will benefit from a low-protein diet.
Ans: A Feedback: Due to the continuous involuntary movements, patients will have a ravenous appetite. Despite this ravenous appetite, patients usually become emaciated and exhausted. As the disease progresses, patients experience difficulty in swallowing and thin liquids should be avoided. Protein will not be limited with this disease. Enzyme supplements are not normally required.
23. A gerontologic nurse is advocating for diagnostic testing of an 81-year-old patient who is experiencing Test Bank - Brunner & Suddarth's Textbook of Medical-Surgical Nursing 14e (Hinkle 2017) 1323 personality changes. The nurse is aware of what factor that is known to affect the diagnosis and treatment of brain tumors in older adults? A) The effects of brain tumors are often attributed to the cognitive effects of aging. B) Brain tumors in older adults do not normally produce focal effects. C) Older adults typically have numerous benign brain tumors by the eighth decade of life. D) Brain tumors cannot normally be treated in patient over age 75.
Ans: A Feedback: In older adult patients, early signs and symptoms of intracranial tumors can be easily overlooked or incorrectly attributed to cognitive and neurologic changes associated with normal aging. Brain tumors are not normally benign and they produce focal effects in all patients. Treatment options are not dependent primarily on age.
6. The nurse is working with a patient who is newly diagnosed with MS. What basic information should the nurse provide to the patient? A) MS is a progressive demyelinating disease of the nervous system. B) MS usually occurs more frequently in men. C) MS typically has an acute onset. D) MS is sometimes caused by a bacterial infection.
Ans: A Feedback: MS is a chronic, degenerative, progressive disease of the central nervous system, characterized by the occurrence of small patches of demyelination in the brain and spinal cord. The cause of MS is not known, and the disease affects twice as many women as men.
25. A male patient presents at the free clinic with complaints of impotency. Upon physical examination, the nurse practitioner notes the presence of hypogonadism. What diagnosis should the nurse suspect? A) Prolactinoma B) Angioma Test Bank - Brunner & Suddarth's Textbook of Medical-Surgical Nursing 14e (Hinkle 2017) 1324 C) Glioma D) Adrenocorticotropic hormone (ACTH)producing adenoma
Ans: A Feedback: Male patients with prolactinomas may present with impotence and hypogonadism. An ACTH-producing adenoma would cause acromegaly. The scenario contains insufficient information to know if the tumor is an angioma, glioma, or neuroma.
29. The nurse is teaching a patient with Guillain-Barr syndrome about the disease. The patient asks how he can ever recover if demyelination of his nerves is occurring. What would be the nurses best response? A) Guillain-Barr spares the Schwann cell, which allows for remyelination in the recovery phase of the disease. B) In Guillain-Barr, Schwann cells replicate themselves before the disease destroys them, so remyelination is possible. C) I know you understand that nerve cells do not remyelinate, so the physician is the best one to answer your question. D) For some reason, in Guillain-Barr, Schwann cells become activated and take over the remyelination process.
Ans: A Feedback: Myelin is a complex substance that covers nerves, providing insulation and speeding the conduction of impulses from the cell body to the dendrites. The cell that produces myelin in the peripheral nervous system is the Schwann cell. In Guillain-Barr syndrome, the Schwann cell is spared, allowing for remyelination in the recovery phase of the disease. The nurse should avoid downplaying the patients concerns by wholly deferring to the physician.
33. A patient with diabetes presents to the clinic and is diagnosed with a mononeuropathy. This patients nursing care should involve which of the following? A) Protection of the affected limb from injury B) Passive and active ROM exercises for the affected limb C) Education about improvements to glycemic control D) Interventions to prevent contractures
Ans: A Feedback: Nursing care involves protection of the affected limb or area from injury, as well as appropriate patient teaching about mononeuropathy and its treatment. Nursing care for this patient does not likely involve exercises or assistive devices, since these are unrelated to the etiology of the disease. Improvements to diabetes management may or may not be necessary.
33. The nurse is caring for a patient who has undergone percutaneous transluminal coronary angioplasty (PTCA). What is the major indicator of success for this procedure? A) Increase in the size of the arterys lumen B) Decrease in arterial blood flow in relation to venous flow C) Increase in the patients resting heart rate D) Increase in the patients level of consciousness (LOC)
Ans: A Feedback: PTCA is used to open blocked coronary vessels and resolve ischemia. The procedure may result in beneficial changes to the patients LOC or heart rate, but these are not the overarching goals of PTCA. Increased arterial flow is the focus of the procedures.
30. An older adult has encouraged her husband to visit their primary care provider, stating that she is concerned that he may have Parkinsons disease. Which of the wifes descriptions of her husbands health and function is most suggestive of Parkinsons disease? A) Lately he seems to move far more slowly than he ever has in the past. B) He often complains that his joints are terribly stiff when he wakes up in the morning. C) Hes forgotten the names of some people that weve known for years. D) Hes losing weight even though he has a ravenous appetite.
Ans: A Feedback: Parkinsons disease is characterized by bradykinesia. It does not manifest as memory loss, increased appetite, or joint stiffness.
37. A patient with a new diagnosis of amyotrophic lateral sclerosis (ALS) is overwhelmed by his diagnosis and the known complications of the disease. How can the patient best make known his wishes for care as his disease progresses? A) Prepare an advance directive. B) Designate a most responsible physician (MRP) early in the course of the disease. C) Collaborate with representatives from the Amyotrophic Lateral Sclerosis Association. D) Ensure that witnesses are present when he provides instruction.
Ans: A Feedback: Patients with ALS are encouraged to complete an advance directive or living will to preserve their autonomy in decision making. None of the other listed actions constitutes a legally binding statement of end-of-life care.
18. When discussing angina pectoris secondary to atherosclerotic disease with a patient, the patient asks why he tends to experience chest pain when he exerts himself. The nurse should describe which of the following phenomena? A) Exercise increases the hearts oxygen demands. B) Exercise causes vasoconstriction of the coronary arteries. C) Exercise shunts blood flow from the heart to the mesenteric area. D) Exercise increases the metabolism of cardiac medications.
Ans: A Feedback: Physical exertion increases the myocardial oxygen demand. If the patient has arteriosclerosis of the coronary arteries, then blood supply is diminished to the myocardium. Exercise does not cause vasoconstriction or interfere with drug metabolism. Exercise does not shunt blood flow away from the heart.
8. The nurse is caring for a patient who is scheduled for cardiac surgery. What should the nurse include in preoperative care? A) With the patient, clarify the surgical procedure that will be performed. B) Withhold the patients scheduled medications for at least 12 hours preoperatively. C) Inform the patient that health teaching will begin as soon as possible after surgery. D) Avoid discussing the patients fears as not to exacerbate them.
Ans: A Feedback: Preoperatively, it is necessary to evaluate the patients understanding of the surgical procedure, informed consent, and adherence to treatment protocols. Teaching would begin on admission or even prior to admission. The physician would write orders to alter the patients medication regimen if necessary; this will vary from patient to patient. Fears should be addressed directly and empathically.
27. A patient has been admitted to the neurologic ICU with a diagnosis of a brain tumor. The patient is scheduled to have a tumor resection/removal in the morning. Which of the following assessment parameters should the nurse include in the initial assessment? A) Gag reflex B) Deep tendon reflexes C) Abdominal girth D) Hearing acuity Test Bank - Brunner & Suddarth's Textbook of Medical-Surgical Nursing 14e (Hinkle 2017) 1325
Ans: A Feedback: Preoperatively, the gag reflex and ability to swallow are evaluated. In patients with diminished gag response, care includes teaching the patient to direct food and fluids toward the unaffected side, having the patient sit upright to eat, offering a semisoft diet, and having suction readily available. Deep tendon reflexes, abdominal girth, and hearing acuity are less commonly affected by brain tumors and do not affect the risk for aspiration.
40. A nurse is planning discharge education for a patient who underwent a cervical diskectomy. What strategies would the nurse assess that would aid in planning discharge teaching? A) Care of the cervical collar B) Technique for performing neck ROM exercises C) Home assessment of ABGs D) Techniques for restoring nerve function
Ans: A Feedback: Prior to discharge, the nurse should assess the patients use and care of the cervical collar. Neck ROM exercises would be contraindicated and ABGs cannot be assessed in the home. Nerve function is not compromised by a diskectomy.
20. The nurse is developing a plan of care for a patient with Guillain-Barr syndrome. Which of the following interventions should the nurse prioritize for this patient? A) Using the incentive spirometer as prescribed B) Maintaining the patient on bed rest C) Providing aids to compensate for loss of vision D) Assessing frequently for loss of cognitive function
Ans: A Feedback: Respiratory function can be maximized with incentive spirometry and chest physiotherapy. Nursing interventions toward enhancing physical mobility should be utilized. Nursing interventions are aimed at preventing a deep vein thrombosis. Guillain-Barr syndrome does not affect cognitive function or vision.
13. A 48-year-old man presents to the ED complaining of severe substernal chest pain radiating down his left arm. He is admitted to the coronary care unit (CCU) with a diagnosis of myocardial infarction (MI). What nursing assessment activity is a priority on admission to the CCU? A) Begin ECG monitoring. B) Obtain information about family history of heart disease. C) Auscultate lung fields. D) Determine if the patient smokes.
Ans: A Feedback: The 12-lead ECG provides information that assists in ruling out or diagnosing an acute MI. It should be obtained within 10 minutes from the time a patient reports pain or arrives in the ED. By monitoring serial ECG changes over time, the location, evolution, and resolution of an MI can be identified and monitored; life-threatening arrhythmias are the leading cause of death in the first hours after an MI. Obtaining information about family history of heart disease and whether the patient smokes are not immediate priorities in the acute phase of MI. Data may be obtained from family members later. Lung fields are auscultated after oxygenation and pain control needs are met.
27. A patient presents to the ED in distress and complaining of crushing chest pain. What is the nurses priority for assessment? A) Prompt initiation of an ECG B) Auscultation of the patients point of maximal impulse (PMI) C) Rapid assessment of the patients peripheral pulses D) Palpation of the patients cardiac apex
Ans: A Feedback: The 12-lead ECG provides information that assists in ruling out or diagnosing an acute MI. It should be obtained within 10 minutes from the time a patient reports pain or arrives in the ED. Each of the other listed assessments is valid, but ECG monitoring is the most time dependent priority.
26. You are the clinic nurse caring for a patient with a recent diagnosis of myasthenia gravis. The patient has begun treatment with pyridostigmine bromide (Mestinon). What change in status would most clearly suggest a therapeutic benefit of this medication? A) Increased muscle strength B) Decreased pain C) Improved GI function D) Improved cognition
Ans: A Feedback: The goal of treatment using pyridostigmine bromide is improvement of muscle strength and control of fatigue. The drug is not intended to treat pain, or cognitive or GI functions.
4. A patient with suspected Parkinsons disease is initially being assessed by the nurse. When is the best time to assess for the presence of a tremor? A) When the patient is resting B) When the patient is ambulating C) When the patient is preparing his or her meal tray to eat D) When the patient is participating in occupational therapy
Ans: A Feedback: The tremor is present while the patient is at rest; it increases when the patient is walking, concentrating, or feeling anxious. Resting tremor characteristically disappears with purposeful movement, but is evident when the extremities are motionless. Consequently, the nurse should assess for the presence of a tremor when the patient is not performing deliberate actions.
2. The nurse is planning discharge education for a patient with trigeminal neuralgia. The nurse knows to include information about factors that precipitate an attack. What would the nurse be correct in teaching the patient to avoid? A) Washing his face B) Exposing his skin to sunlight C) Using artificial tears D) Drinking large amounts of fluids
Ans: A Feedback: Washing the face should be avoided if possible because this activity can trigger an attack of pain in a patient with trigeminal neuralgia. Using artificial tears would be an appropriate behavior. Exposing the skin to sunlight would not be harmful to this patient. Temperature extremes in beverages should be Test Bank - Brunner & Suddarth's Textbook of Medical-Surgical Nursing 14e (Hinkle 2017) 1296 avoided.
4. The triage nurse in the ED assesses a 66-year-old male patient who presents to the ED with complaints of midsternal chest pain that has lasted for the last 5 hours. If the patients symptoms are due to an MI, what will have happened to the myocardium? A) It may have developed an increased area of infarction during the time without treatment. B) It will probably not have more damage than if he came in immediately. C) It may be responsive to restoration of the area of dead cells with proper treatment. D) It has been irreparably damaged, so immediate treatment is no longer necessary.
Ans: A Feedback: When the patient experiences lack of oxygen to myocardium cells during an MI, the sooner treatment is initiated, the more likely the treatment will prevent or minimize myocardial tissue necrosis. Delays in treatment equate with increased myocardial damage. Despite the length of time the symptoms have been present, treatment needs to be initiated immediately to minimize further damage. Dead cells cannot be restored by any means.
21. A patient with suspected adrenal insufficiency has been ordered an adrenocorticotropic hormone - 987 (ACTH) stimulation test. Administration of ACTH caused a marked increase in cortisol levels. How should the nurse interpret this finding? A) The patients pituitary function is compromised. B) The patients adrenal insufficiency is not treatable. C) The patient has insufficient hypothalamic function. D) The patient would benefit from surgery.
Ans: A Feedback: An adrenal response to the administration of a stimulating hormone suggests inadequate production of the stimulating hormone. In this case, ACTH is produced by the pituitary and, consequently, pituitary hypofunction is suggested. Hypothalamic function is not relevant to the physiology of this problem. Treatment exists, although surgery is not likely indicated.
36. A family member of a patient diagnosed with Huntington disease calls you at the clinic. She is requesting help from the Huntingtons Disease Society of America. What kind of help can this patient and family receive from this organization? Select all that apply. A) Information about this disease B) Referrals C) Public education D) Individual assessments Test Bank - Brunner & Suddarth's Textbook of Medical-Surgical Nursing 14e (Hinkle 2017) 1329 E) Appraisals of research studies
Ans: A, B, C Feedback: The Huntingtons Disease Society of America helps patients and families by providing information, referrals, family and public education, and support for research. It does not provide individual assessments or appraisals of individual research studies.
A patient is being treated in the ICU for neurogenic shock secondary to a spinal cord injury. Despite aggressive interventions, the patient's mean arterial pressure (MAP) has fallen to 55 mm Hg. The nurse should gauge the onset of acute kidney injury by referring to what laboratory findings? Select all that apply. A) Blood urea nitrogen (BUN) level B) Urine specific gravity C) Alkaline phosphatase level D) Creatinine level E) Serum albumin level
Ans: A, B, D Feedback: Acute kidney injury (AKI) is characterized by an increase in BUN and serum creatinine levels, fluid and electrolyte shifts, acid-base imbalances, and a loss of the renal-hormonal regulation of BP. Urine specific gravity is also affected. Alkaline phosphatase and albumin levels are related to hepatic function.
29. The nurse is assessing a patient with acute coronary syndrome (ACS). The nurse includes a careful history in the assessment, especially with regard to signs and symptoms. What signs and symptoms are suggestive of ACS? Select all that apply. A) Dyspnea B) Unusual fatigue C) Hypotension D) Syncope E) Peripheral cyanosis
Ans: A, B, D Feedback: Systematic assessment includes a careful history, particularly as it relates to symptoms: chest pain or discomfort, difficulty breathing (dyspnea), palpitations, unusual fatigue, faintness (syncope), or sweating (diaphoresis). Each symptom must be evaluated with regard to time, duration, and the factors that precipitate the symptom and relieve it, and in comparison with previous symptoms. Hypotension and peripheral cyanosis are not typically associated with ACS.
35. A 35-year-old woman is diagnosed with a peripheral neuropathy. When making her plan of care, the nurse knows to include what in patient teaching? Select all that apply. A) Inspect the lower extremities for skin breakdown. B) Footwear needs to be accurately sized. C) Immediate family members should be screened for the disease. D) Assistive devices may be needed to reduce the risk of falls. E) Dietary modifications are likely necessary.
Ans: A, B, D Feedback: The plan of care includes inspection of the lower extremities for skin breakdown. Footwear should be accurately sized. Assistive devices, such as a walker or cane, may decrease the risk of falls. Bath water temperature is checked to avoid thermal injury. Peripheral neuropathies do not have a genetic component and diet is unrelated.
29. A patient with an inoperable brain tumor has been told that he has a short life expectancy. On what aspects of assessment and care should the home health nurse focus? Select all that apply. A) Pain control B) Management of treatment complications C) Interpretation of diagnostic tests D) Assistance with self-care E) Administration of treatments
Ans: A, B, D, E Feedback: Test Bank - Brunner & Suddarth's Textbook of Medical-Surgical Nursing 14e (Hinkle 2017) 1326 Home care needs and interventions focus on four major areas: palliation of symptoms and pain control, assistance in self-care, control of treatment complications, and administration of specific forms of treatment, such as parenteral nutrition. Interpretation of diagnostic tests is normally beyond the purview of the nurse.
25. The nurse is caring for a 77-year-old woman with MS. She states that she is very concerned about the progress of her disease and what the future holds. The nurse should know that elderly patients with MS are known to be particularly concerned about what variables? Select all that apply. A) Possible nursing home placement B) Pain associated with physical therapy C) Increasing disability D) Becoming a burden on the family E) Loss of appetite
Ans: A, C, D Test Bank - Brunner & Suddarth's Textbook of Medical-Surgical Nursing 14e (Hinkle 2017) 1306 Feedback: Elderly patients with MS are particularly concerned about increasing disability, family burden, marital concern, and the possible future need for nursing home care. Older adults with MS are not noted to have particular concerns regarding the pain of therapy or loss of appetite.
38. The nurse providing care for a patient post PTCA knows to monitor the patient closely. For what complications should the nurse monitor the patient? Select all that apply. A) Abrupt closure of the coronary artery B) Venous insufficiency C) Bleeding at the insertion site D) Retroperitoneal bleeding E) Arterial occlusion
Ans: A, C, D, E Feedback: Complications after the procedure may include abrupt closure of the coronary artery and vascular complications, such as bleeding at the insertion site, retroperitoneal bleeding, hematoma, and arterial occlusion, as well as acute renal failure. Venous insufficiency is not a postprocedure complication of a PTCA.
The intensive care nurse is responsible for the care of a patient with shock. What cardiac signs or symptoms would suggest to the nurse that the patient may be experiencing acute organ dysfunction? Select all that apply. A) Drop in systolic blood pressure of ≥40 mm Hg from baselines B) Hypotension that responds to bolus fluid resuscitation C) Exaggerated response to vasoactive medications D) Serum lactate >4 mmol/L E) Mean arterial pressure (MAP) of ˂65 mm Hg
Ans: A, D, E Feedback: Signs of acute organ dysfunction in the cardiovascular system include systolic blood pressure <90 mm Hg or mean arterial pressure (MAP) <65 mm Hg, drop in systolic blood pressure >40 mm Hg from baselines or serum lactate >4 mmol/L. An exaggerated response to vasoactive medications and an adequate response to fluid resuscitation would not be noted.
A team of nurses are reviewing the similarities and differences between the different classifications of shock. Which subclassifications of circulatory shock should the nurses identify? Select all that apply. A) Anaphylactic B) Hypovolemic C) Cardiogenic D) Septic E) Neurogenic
Ans: A, D, E Feedback: The varied mechanisms leading to the initial vasodilation in circulatory shock provide the basis for the further subclassification of shock into three types: septic shock, neurogenic shock, and anaphylactic shock. Hypovolemic and cardiogenic shock are not subclassifications of circulatory shock.
A patient with pericarditis has just been admitted to the CCU. The nurse planning the patient's care should prioritize what nursing diagnosis? A) Anxiety related to pericarditis B) Acute pain related to pericarditis C) Ineffective tissue perfusion related to pericarditis D) Ineffective breathing pattern related to pericarditis
Ans: Acute pain related to pericarditis Feedback: The most characteristic symptom of pericarditis is chest pain, although pain also may be located beneath the clavicle, in the neck, or in the left trapezius (scapula) region. The pain or discomfort usually remains fairly constant, but it may worsen with deep inspiration and when lying down or turning. Anxiety is highly plausible and should be addressed, but chest pain is a nearly certain accompaniment to the disease. Breathing and tissue perfusion are likely to be at risk, but pain is certain, especially in the early stages of treatment.
A patient with a history rheumatic heart disease knows that she is at risk for bacterial endocarditis when undergoing invasive procedures. Prior to a scheduled cystoscopy, the nurse should ensure that the patient knows the importance of taking which of the following drugs? A) Enoxaparin (Lovenox) B) Metoprolol (Lopressor) C) Azathioprine (Imuran) D) Amoxicillin (Amoxil)
Ans: Amoxicillin (Amoxil) Feedback: Although rare, bacterial endocarditis may be life-threatening. A key strategy is primary prevention in high-risk patients (i.e., those with rheumatic heart disease, mitral valve prolapse, or prosthetic heart valves). Antibiotic prophylaxis is recommended for high-risk patients immediately before and sometimes after certain procedures. Amoxicillin is the drug of choice. None of the other listed drugs is an antibiotic.
Most individuals who have mitral valve prolapse never have any symptoms, although this is not the case for every patient. What symptoms might a patient have with mitral valve prolapse? Select all that apply. A) Anxiety B) Fatigue C) Shoulder pain D) Tachypnea E) Palpitations
Ans: Anxiety, Fatigue, Palpitations Feedback: Most people who have mitral valve prolapse never have symptoms. A few have symptoms of fatigue, shortness of breath, lightheadedness, dizziness, syncope, palpitations, chest pain, and anxiety. Hyperpnea and shoulder pain are not characteristic symptoms of mitral valve prolapse.
A patient with mitral stenosis exhibits new symptoms of a dysrhythmia. Based on the pathophysiology of this disease process, the nurse would expect the patient to exhibit what heart rhythm? A) Ventricular fibrillation (VF) B) Ventricular tachycardia (VT) C) Atrial fibrillation D) Sinus bradycardia
Ans: Atrial fibrillation Feedback: In patients with mitral valve stenosis, the pulse is weak and often irregular because of atrial fibrillation. Bradycardia, VF, and VT are not characteristic of this valvular disorder.
A patient with mitral valve stenosis is receiving health education at an outpatient clinic. To minimize the patient's symptoms, the nurse should teach the patient to do which of the following? A) Eat a high-protein, low-carbohydrate diet. B) Avoid activities that cause an increased heart rate. C) Avoid large crowds and public events. D) Perform deep breathing and coughing exercises.
Ans: Avoid activities that cause an increased heart rate. Feedback: Patients with mitral stenosis are advised to avoid strenuous activities, competitive sports, and pregnancy, all of which increase heart rate. Infection prevention is important, but avoiding crowds is not usually necessary. Deep breathing and coughing are not likely to prevent exacerbations of symptoms and increased protein intake is not necessary.
The critical care nurse is caring for a patient who is receiving cyclosporine postoperative heart transplant. The patient asks the nurse to remind him what this medication is for. How should the nurse best respond? A) Azathioprine decreases the risk of thrombus formation. B) Azathioprine ensures adequate cardiac output. C) Azathioprine increases the number of white blood cells. D) Azathioprine minimizes rejection of the transplant.
Ans: Azathioprine minimizes rejection of the transplant. Feedback: After heart transplant, patients are constantly balancing the risk of rejection with the risk of infection. Most commonly, patients receive cyclosporine or tacrolimus (FK506, Prograf), azathioprine (Imuran), or mycophenolate mofetil (CellCept), and corticosteroids (prednisone) to minimize rejection. Cyclosporine does not prevent thrombus formation, enhance cardiac output, or increase white cell counts.
A triage nurse in the ED is on shift when a grandfather carries his 4-year-old grandson into the ED. The child is not breathing, and the grandfather states the boy was stung by a bee in a nearby park while they were waiting for the boy's mother to get off work. Which of the following would lead the nurse to suspect that the boy is experiencing anaphylactic shock? A) Rapid onset of acute hypertension B) Rapid onset of respiratory distress C) Rapid onset of neurologic compensation D) Rapid onset of cardiac arrest
Ans: B Feedback: Characteristics of severe anaphylaxis usually include rapid onset of hypotension, neurologic compromise, and respiratory distress. Cardiac arrest can occur if prompt treatment is not provided.
A nurse in the ICU is planning the care of a patient who is being treated for shock. Which of the following statements best describes the pathophysiology of this patient's health problem? A) Blood is shunted from vital organs to peripheral areas of the body. B) Cells lack an adequate blood supply and are deprived of oxygen and nutrients. C) Circulating blood volume is decreased with a resulting change in the osmotic pressure gradient. D) Hemorrhage occurs as a result of trauma, depriving vital organs of adequate perfusion.
Ans: B Feedback: Shock is a life-threatening condition with a variety of underlying causes. Shock is caused when the cells have a lack of adequate blood supply and are deprived of oxygen and nutrients. In cases of shock, blood is shunted from peripheral areas of the body to the vital organs. Hemorrhage and decreased blood volume are associated with some, but not all, types of shock.
An 11-year-old boy has been brought to the ED by his teacher, who reports that the boy may be having a "really bad allergic reaction to peanuts" after trading lunches with a peer. The triage nurse's rapid assessment reveals the presence of respiratory and cardiac arrest. What interventions should the nurse prioritize? A) Establishing central venous access and beginning fluid resuscitation B) Establishing a patent airway and beginning cardiopulmonary resuscitation C) Establishing peripheral IV access and administering IV epinephrine D) Performing a comprehensive assessment and initiating rapid fluid replacement
Ans: B Feedback: If cardiac arrest and respiratory arrest are imminent or have occurred, CPR is performed. As well, a patent airway is an immediate priority. Epinephrine is not withheld pending IV access and fluid resuscitation is not a priority.
6. A nurse in the ICU receives report from the nurse in the ED about a new patient being admitted with a neck injury he received while diving into a lake. The ED nurse reports that his blood pressure is 85/54, heart rate is 53 beats per minute, and his skin is warm and dry. What does the ICU nurse recognize that that patient is probably experiencing? A) Anaphylactic shock B) Neurogenic shock C) Septic shock D) Hypovolemic shock
Ans: B Feedback: Neurogenic shock can be caused by spinal cord injury. The patient will present with a low blood pressure; bradycardia; and warm, dry skin due to the loss of sympathetic muscle tone and increased parasympathetic stimulation. Anaphylactic shock is caused by an identifiable offending agent, such as a bee sting. Septic shock is caused by bacteremia in the blood and presents with a tachycardia. Hypovolemic shock presents with tachycardia and a probable source of blood loss.
The nurse is transferring a patient who is in the progressive stage of shock into ICU from the medical unit. The medical nurse is aware that shock affects many organ systems and that nursing management of the patient will focus on what intervention? A) Reviewing the cause of shock and prioritizing the patient's psychosocial needs B) Assessing and understanding shock and the significant changes in assessment data to guide the plan of care C) Giving the prescribed treatment, but shifting focus to providing family time as the patient is unlikely to survive D) Promoting the patient's coping skills in an effort to better deal with the physiologic changes accompanying shock
Ans: B Feedback: Nursing care of patients in the progressive stage of shock requires expertise in assessing and understanding shock and the significance of changes in assessment data. Early interventions are essential to the survival of patients in shock; thus, suspecting that a patient may be in shock and reporting subtle changes in assessment are imperative. Psychosocial needs, such as coping, are important considerations, but they are not prioritized over physiologic health.
11. A patient with angina has been prescribed nitroglycerin. Before administering the drug, the nurse should inform the patient about what potential adverse effects? A) Nervousness or paresthesia B) Throbbing headache or dizziness C) Drowsiness or blurred vision D) Tinnitus or diplopia
Ans: B Feedback: Headache and dizziness commonly occur when nitroglycerin is taken at the beginning of therapy. Nervousness, paresthesia, drowsiness, blurred vision, tinnitus, and diplopia do not typically occur as a result of nitroglycerin therapy.
The nurse in the ICU is caring for a 47-year-old, obese male patient who is in shock following a motor vehicle accident. The nurse is aware that patients in shock possess excess energy requirements. What would be the main challenge in meeting this patient's elevated energy requirements during prolonged rehabilitation? A) Loss of adipose tissue B) Loss of skeletal muscle C) Inability to convert adipose tissue to energy D) Inability to maintain normal body mass
Ans: B Feedback: Nutritional energy requirements are met by breaking down lean body mass. In this catabolic process, skeletal muscle mass is broken down even when the patient has large stores of fat or adipose tissue. Loss of skeletal muscle greatly prolongs the patient's recovery time. Loss of adipose tissue, the inability to convert adipose tissue to energy, and the inability to maintain normal body mass are not main concerns in meeting nutritional energy requirements for this patient.
A patient who is in shock is receiving dopamine in addition to IV fluids. What principle should inform the nurse's care planning during the administration of a vasoactive drug? A) The drug should be discontinued immediately after blood pressure increases. B) The drug dose should be tapered down once vital signs improve. C) The patient should have arterial blood gases drawn every 10 minutes during treatment. D) The infusion rate should be titrated according the patient's subjective sensation of adequate perfusion.
Ans: B Feedback: When vasoactive medications are discontinued, they should never be stopped abruptly because this could cause severe hemodynamic instability, perpetuating the shock state. Subjective assessment data are secondary to objective data. Arterial blood gases should be carefully monitored, but every10-minute draws are not the norm.
34. The nurse is caring for a patient diagnosed with Parkinsons disease. The patient is having increasing problems with rising from the sitting to the standing position. What should the nurse suggest to the patient to use that will aid in getting from the sitting to the standing position as well as aid in improving bowel elimination? A) Use of a bedpan Test Bank - Brunner & Suddarth's Textbook of Medical-Surgical Nursing 14e (Hinkle 2017) 1328 B) Use of a raised toilet seat C) Sitting quietly on the toilet every 2 hours D) Following the outlined bowel program
Ans: B Feedback: A raised toilet seat is useful, because the patient has difficulty in moving from a standing to a sitting position. A handicapped toilet is not high enough and will not aid in improving bowel elimination. Sitting quietly on the toilet every 2 hours will not aid in getting from the sitting to standing position; neither will following the outlined bowel program.
15. A patient with herpes simplex virus encephalitis (HSV) has been admitted to the ICU. What medication would the nurse expect the physician to order for the treatment of this disease process? A) Cyclosporine (Neoral) B) Acyclovir (Zovirax) C) Cyclobenzaprine (Flexeril) D) Ampicillin (Prinicpen)
Ans: B Feedback: Acyclovir (Zovirax) or ganciclovir (Cytovene), antiviral agents, are the medications of choice in the treatment of HSV. The mode of action is the inhibition of viral DNA replication. To prevent relapse, treatment would continue for up to 3 weeks. Cyclosporine is an immunosuppressant and antirheumatic. Cyclobenzaprine is a centrally acting skeletal muscle relaxant. Ampicillin, an antibiotic, is ineffective against viruses.
34. A nurse has taken on the care of a patient who had a coronary artery stent placed yesterday. When reviewing the patients daily medication administration record, the nurse should anticipate administering what drug? A) Ibuprofen B) Clopidogrel C) Dipyridamole D) Acetaminophen
Ans: B Feedback: Because of the risk of thrombus formation within the stent, the patient receives antiplatelet medications, usually aspirin and clopidogrel. Ibuprofen and acetaminophen are not antiplatelet drugs. Dipyridamole is not the drug of choice following stent placement.
10. The nurse is developing a plan of care for a patient newly diagnosed with Bells palsy. The nurses plan of care should address what characteristic manifestation of this disease? A) Tinnitus B) Facial paralysis C) Pain at the base of the tongue D) Diplopia
Ans: B Feedback: Bells palsy is characterized by facial dysfunction, weakness, and paralysis. It does not result in diplopia, pain at the base of the tongue, or tinnitus.
1. A patient with possible bacterial meningitis is admitted to the ICU. What assessment finding would the nurse expect for a patient with this diagnosis? A) Pain upon ankle dorsiflexion of the foot B) Neck flexion produces flexion of knees and hips C) Inability to stand with eyes closed and arms extended without swaying D) Numbness and tingling in the lower extremities
Ans: B Feedback: Clinical manifestations of bacterial meningitis include a positive Brudzinskis sign. Neck flexion producing flexion of knees and hips correlates with a positive Brudzinskis sign. Positive Homans sign (pain upon dorsiflexion of the foot) and negative Rombergs sign (inability to stand with eyes closed and arms extended) are not expected assessment findings for the patient with bacterial meningitis. Peripheral neuropathy manifests as numbness and tingling in the lower extremities. Again, this would not be an initial assessment to rule out bacterial meningitis.
38. A patient with MS has developed dysphagia as a result of cranial nerve dysfunction. What nursing action should the nurse consequently perform? A) Arrange for the patient to receive a low residue diet. B) Position the patient upright during feeding. C) Suction the patient following each meal. D) Withhold liquids until the patient has finished eating.
Ans: B Feedback: Correct, upright positioning is necessary to prevent aspiration in the patient with dysphagia. There is no need for a low-residue diet and suctioning should not be performed unless there is an apparent need. Liquids do not need to be withheld during meals in order to prevent aspiration.
3. The nurse is caring for a patient with multiple sclerosis (MS). The patient tells the nurse the hardest thing to deal with is the fatigue. When teaching the patient how to reduce fatigue, what action should the nurse suggest? A) Taking a hot bath at least once daily B) Resting in an air-conditioned room whenever possible C) Increasing the dose of muscle relaxants D) Avoiding naps during the day
Ans: B Feedback: Fatigue is a common symptom of patients with MS. Lowering the body temperature by resting in an airconditioned room may relieve fatigue; however, extreme cold should be avoided. A hot bath or shower can increase body temperature, producing fatigue. Muscle relaxants, prescribed to reduce spasticity, can cause drowsiness and fatigue. Planning for frequent rest periods and naps can relieve fatigue. Other measures to reduce fatigue in the patient with MS include treating depression, using occupational therapy to learn energy conservation techniques, and reducing spasticity.
14. A patient diagnosed with a pituitary adenoma has arrived on the neurologic unit. When planning the patients care, the nurse should be aware that the effects of the tumor will primarily depend on what variable? Test Bank - Brunner & Suddarth's Textbook of Medical-Surgical Nursing 14e (Hinkle 2017) 1319 A) Whether the tumor utilizes aerobic or anaerobic respiration B) The specific hormones secreted by the tumor C) The patients pre-existing health status D) Whether the tumor is primary or the result of metastasis
Ans: B Feedback: Functioning pituitary tumors can produce one or more hormones normally produced by the anterior pituitary and the effects of the tumor depend largely on the identity of these hormones. This variable is more significant than the patients health status or whether the tumor is primary versus secondary. Anaerobic and aerobic respiration is not relevant.
18. A nurse is reviewing the trend of a patients scores on the Glasgow Coma Scale (GCS). This allows the nurse to gauge what aspect of the patients status? A) Reflex activity B) Level of consciousness C) Cognitive ability D) Sensory involvement
b
16. The nurse in an extended care facility is planning the daily activities of a patient with postpolio syndrome. The nurse recognizes the patient will best benefit from physical therapy when it is scheduled at what time? A) Immediately after meals B) In the morning Test Bank - Brunner & Suddarth's Textbook of Medical-Surgical Nursing 14e (Hinkle 2017) 1320 C) Before bedtime D) In the early evening
Ans: B Feedback: Important activities for patients with postpolio syndrome should be planned for the morning, as fatigue often increases in the afternoon and evening.
30. The nurse is creating a plan of care for a patient with acute coronary syndrome. What nursing action should be included in the patients care plan? A) Facilitate daily arterial blood gas (ABG) sampling. B) Administer supplementary oxygen, as needed. C) Have patient maintain supine positioning when in bed. D) Perform chest physiotherapy, as indicated.
Ans: B Feedback: Oxygen should be administered along with medication therapy to assist with symptom relief. Administration of oxygen raises the circulating level of oxygen to reduce pain associated with low levels of myocardial oxygen. Physical rest in bed with the head of the bed elevated or in a supportive chair helps decrease chest discomfort and dyspnea. ABGs are diagnostic, not therapeutic, and they are rarely needed on a daily basis. Chest physiotherapy is not used in the treatment of ACS.
2. A 25-year-old female patient with brain metastases is considering her life expectancy after her most recent meeting with her oncologist. Based on the fact that the patient is not receiving treatment for her brain metastases, what is the nurses most appropriate action? A) Promoting the patients functional status and ADLs B) Ensuring that the patient receives adequate palliative care C) Ensuring that the family does not tell the patient that her condition is terminal D) Promoting adherence to the prescribed medication regimen
Ans: B Feedback: Patients with intracerebral metastases who are not treated have a steady downhill course with a limited survival time, whereas those who are treated may survive for slightly longer periods, but for most cure is not possible. Palliative care is thus necessary. This is a priority over promotion of function and the family should not normally withhold information from the patient. Adherence to medications such as analgesics is important, but palliative care is a high priority. Test Bank - Brunner & Suddarth's Textbook of Medical-Surgical Nursing 14e (Hinkle 2017) 1314
13. A patient has been admitted to the neurologic unit for the treatment of a newly diagnosed brain tumor. The patient has just exhibited seizure activity for the first time. What is the nurses priority response to this event? A) Identify the triggers that precipitated the seizure. B) Implement precautions to ensure the patients safety. C) Teach the patients family about the relationship between brain tumors and seizure activity. D) Ensure that the patient is housed in a private room.
Ans: B Feedback: Patients with seizures are carefully monitored and protected from injury. Patient safety is a priority over health education, even though this is appropriate and necessary. Specific triggers may or may not be evident; identifying these is not the highest priority. A private room is preferable, but not absolutely necessary.
6. The nurse is caring for a boy who has muscular dystrophy. When planning assistance with the patients ADLs, what goal should the nurse prioritize? A) Promoting the patients recovery from the disease B) Maximizing the patients level of function C) Ensuring the patients adherence to treatment D) Fostering the familys participation in care
Ans: B Feedback: Priority for the care of the child with muscular dystrophy is the need to maximize the patients level of function. Family participation is also important, but should be guided by this goal. Adherence is not a central goal, even though it is highly beneficial, and the disease is not curable.
17. A patient newly diagnosed with a cervical disk herniation is receiving health education from the clinic nurse. What conservative management measures should the nurse teach the patient to implement? A) Perform active ROM exercises three times daily. B) Sleep on a firm mattress. C) Apply cool compresses to the back of the neck daily. D) Wear the cervical collar for at least 2 hours at a time.
Ans: B Feedback: Proper positioning on a firm mattress and bed rest for 1 to 2 days may bring dramatic relief from pain. The patient may need to wear a cervical collar 24 hours a day during the acute phase of pain from a cervical disk herniation. Hot, moist compresses applied to the back of the neck will increase blood flow to the muscles and help relax the spastic muscles.
14. To alleviate pain associated with trigeminal neuralgia, a patient is taking Tegretol (carbamazepine). What health education should the nurse provide to the patient before initiating this treatment? A) Concurrent use of calcium supplements is contraindicated. B) Blood levels of the drug must be monitored. C) The drug is likely to cause hyperactivity and agitation. Test Bank - Brunner & Suddarth's Textbook of Medical-Surgical Nursing 14e (Hinkle 2017) 1301 D) Tegretol can cause tinnitus during the first few days of treatment.
Ans: B Feedback: Side effects of Tegretol include nausea, dizziness, drowsiness, and aplastic anemia. The patient must also be monitored for bone marrow depression during long-term therapy. Skin discoloration, insomnia, and tinnitus are not side effects of Tegretol.
19. The nurse is caring for a patient who is believed to have just experienced an MI. The nurse notes changes in the ECG of the patient. What change on an ECG most strongly suggests to the nurse that ischemia is occurring? A) P wave inversion B) T wave inversion C) Q wave changes with no change in ST or T wave D) P wave enlargement
Ans: B Feedback: T-wave inversion is an indicator of ischemic damage to myocardium. Typically, few changes to P waves occur during or after an MI, whereas Q-wave changes with no change in the ST or T wave indicate an old MI.
34. A patient diagnosed with MS has been admitted to the medical unit for treatment of an MS exacerbation. Included in the admission orders is baclofen (Lioresal). What should the nurse identify as an expected outcome of this treatment? A) Reduction in the appearance of new lesions on the MRI B) Decreased muscle spasms in the lower extremities C) Increased muscle strength in the upper extremities D) Decreased severity and duration of exacerbations
Ans: B Feedback: Test Bank - Brunner & Suddarth's Textbook of Medical-Surgical Nursing 14e (Hinkle 2017) 1310 Baclofen, a g-aminobutyric acid (GABA) agonist, is the medication of choice in treating spasms. It can be administered orally or by intrathecal injection. Avonex and Betaseron reduce the appearance of new lesions on the MRI. Corticosteroids limit the severity and duration of exacerbations. Anticholinesterase agents increase muscle strength in the upper extremities.
38. The nurse is caring for a patient who is scheduled for a cervical discectomy the following day. During health education, the patient should be made aware of what potential complications? A) Vertebral fracture B) Hematoma at the surgical site C) Scoliosis D) Renal trauma
Ans: B Feedback: Test Bank - Brunner & Suddarth's Textbook of Medical-Surgical Nursing 14e (Hinkle 2017) 1330 Based on all the assessment data, the potential complications of diskectomy may include hematoma at the surgical site, resulting in cord compression and neurologic deficit and recurrent or persistent pain after surgery. Renal trauma and fractures are unlikely; scoliosis is a congenital malformation of the spine.
24. When assessing a patient diagnosed with angina pectoris it is most important for the nurse to gather what information? A) The patients activities limitations and level of consciousness after the attacks B) The patients symptoms and the activities that precipitate attacks C) The patients understanding of the pathology of angina D) The patients coping strategies surrounding the attacks
Ans: B Feedback: The nurse must gather information about the patients symptoms and activities, especially those that precede and precipitate attacks of angina pectoris. The patients coping, understanding of the disease, and status following attacks are all important to know, but causative factors are a primary focus of the assessment interview.
20. An adult patient is admitted to the ED with chest pain. The patient states that he had developed unrelieved chest pain that was present for approximately 20 minutes before coming to the hospital. To minimize cardiac damage, the nurse should expect to administer which of the following interventions? A) Thrombolytics, oxygen administration, and nonsteroidal anti-inflammatories B) Morphine sulphate, oxygen, and bed rest C) Oxygen and beta-adrenergic blockers D) Bed rest, albuterol nebulizer treatments, and oxygen
Ans: B Feedback: The patient with suspected MI should immediately receive supplemental oxygen, aspirin, nitroglycerin, and morphine. Morphine sulphate reduces preload and decreases workload of the heart, along with increased oxygen from oxygen therapy and bed rest. With decreased cardiac demand, this provides the best chance of decreasing cardiac damage. NSAIDs and beta-blockers are not normally indicated. Albuterol, which is a medication used to manage asthma and respiratory conditions, will increase the heart rate.
30. A patient diagnosed with myasthenia gravis has been hospitalized to receive plasmapheresis for a myasthenic exacerbation. The nurse knows that the course of treatment for plasmapheresis in a patient with myasthenia gravis is what? Test Bank - Brunner & Suddarth's Textbook of Medical-Surgical Nursing 14e (Hinkle 2017) 1308 A) Every day for 1 week B) Determined by the patients response C) Alternate days for 10 days D) Determined by the patients weight
Ans: B Feedback: The typical course of plasmapheresis consists of daily or alternate-day treatment, and the number of treatments is determined by the patients response.
22. In preparation for cardiac surgery, a patient was taught about measures to prevent venous thromboembolism. What statement indicates that the patient clearly understood this education? A) Ill try to stay in bed for the first few days to allow myself to heal. B) Ill make sure that I dont cross my legs when Im resting in bed. C) Ill keep pillows under my knees to help my blood circulate better. D) Ill put on those compression stockings if I get pain in my calves.
Ans: B Feedback: To prevent venous thromboembolism, patients should avoid crossing the legs. Activity is generally begun as soon as possible and pillows should not be placed under the popliteal space. Compression stockings are often used to prevent venous thromboembolism, but they would not be applied when symptoms emerge.
5. The clinic nurse caring for a patient with Parkinsons disease notes that the patient has been taking levodopa and carbidopa (Sinemet) for 7 years. For what common side effect of Sinemet would the nurse assesses this patient? Test Bank - Brunner & Suddarth's Textbook of Medical-Surgical Nursing 14e (Hinkle 2017) 1315 A) Pruritus B) Dyskinesia C) Lactose intolerance D) Diarrhea
Ans: B Feedback: Within 5 to 10 years of taking levodopa, most patients develop a response to the medication characterized by dyskinesia (abnormal involuntary movements). Another potential complication of longterm dopaminergic medication use is neuroleptic malignant syndrome characterized by severe rigidity, stupor, and hyperthermia. Side effects of long-term Sinemet therapy are not pruritus, lactose intolerance, or diarrhea.
9. A male patient presents to the clinic complaining of a headache. The nurse notes that the patient is guarding his neck and tells the nurse that he has stiffness in the neck area. The nurse suspects the patient may have meningitis. What is another well-recognized sign of this infection? A) Negative Brudzinskis sign B) Positive Kernigs sign C) Hyperpatellar reflex D) Sluggish pupil reaction
Ans: B Test Bank - Brunner & Suddarth's Textbook of Medical-Surgical Nursing 14e (Hinkle 2017) 1299 Feedback: Meningeal irritation results in a number of well-recognized signs commonly seen in meningitis, such as a positive Kernigs sign, a positive Brudzinskis sign, and photophobia. Hyperpatellar reflex and a sluggish pupil reaction are not commonly recognized signs of meningitis.
18. A patient has just returned to the unit from the PACU after surgery for a tumor within the spine. The patient complains of pain. When positioning the patient for comfort and to reduce injury to the surgical site, the nurse will position to patient in what position? A) In the high Fowlers position B) In a flat side-lying position C) In the Trendelenberg position D) In the reverse Trendelenberg position
Ans: B Test Bank - Brunner & Suddarth's Textbook of Medical-Surgical Nursing 14e (Hinkle 2017) 1321 Feedback: After spinal surgery, the bed is usually kept flat initially. The side-lying position is usually the most comfortable because this position imposes the least pressure on the surgical site. The Fowlers position, Trendelenberg position, and reverse Trendelenberg position are inappropriate for this patient because they would result in increased pain and complications.
7. A patient with an occluded coronary artery is admitted and has an emergency percutaneous transluminal coronary angioplasty (PTCA). The patient is admitted to the cardiac critical care unit after the PTCA. For what complication should the nurse most closely monitor the patient? A) Hyperlipidemia B) Bleeding at insertion site C) Left ventricular hypertrophy D) Congestive heart failure
Ans: B Test Bank - Brunner & Suddarth's Textbook of Medical-Surgical Nursing 14e (Hinkle 2017) 529 Feedback: Complications of PTCA may include bleeding at the insertion site, abrupt closure of the artery, arterial thrombosis, and perforation of the artery. Complications do not include hyperlipidemia, left ventricular hypertrophy, or congestive heart failure; each of these problems takes an extended time to develop and none is emergent.
An older adult patient has been diagnosed with aortic regurgitation. What change in blood flow should the nurse expect to see on this patient's echocardiogram? A) Blood to flow back from the aorta to the left ventricle B) Obstruction of blood flow from the left ventricle C) Blood to flow back from the left atrium to the left ventricle D) Obstruction of blood from the left atrium to left ventricle
Ans: Blood to flow back from the aorta to the left ventricle Feedback: Aortic regurgitation occurs when the aortic valve does not completely close, and blood flows back to the left ventricle from the aorta during diastole. Aortic regurgitation does not cause obstruction of blood flow from the left ventricle, blood to flow back from the left atrium to the left ventricle, or obstruction of blood from the left atrium to left ventricle.
The nurse is caring for a patient whose progressing infection places her at high risk for shock. What assessment finding would the nurse consider a potential sign of shock? A) Elevated systolic blood pressure B) Elevated mean arterial pressure (MAP) C) Shallow, rapid respirations D) Bradycardia
Ans: C Feedback: A symptom of shock is shallow, rapid respirations. Systolic blood pressure drops in shock, and MAP is less than 65 mm Hg. Bradycardia occurs in neurogenic shock; other states of shock have tachycardia as a symptom. Infection can lead to septic shock.
The acute care nurse is providing care for an adult patient who is in hypovolemic shock. The nurse recognizes that antidiuretic hormone (ADH) plays a significant role in this health problem. What assessment finding will the nurse likely observe related to the role of the ADH during hypovolemic shock? A) Increased hunger B) Decreased thirst C) Decreased urinary output D) Increased capillary perfusion
Ans: C Feedback: During hypovolemic shock, a state of hypernatremia occurs. Hypernatremia stimulates the release of ADH by the pituitary gland. ADH causes the kidneys to retain water further in an effort to raise blood volume and blood pressure. In a hypovolemic state the body shifts blood away from anything that is not a vital organ, so hunger is not an issue; thirst is increased as the body tries to increase fluid volume; and capillary profusion decreases as the body shunts blood away from the periphery and to the vital organs.
The ICU nurse is caring for a patient in neurogenic shock following an overdose of antianxiety medication. When assessing this patient, the nurse should recognize what characteristic of neurogenic shock? A) Hypertension B) Cool, moist skin C) Bradycardia D) Signs of sympathetic stimulation
Ans: C Feedback: In neurogenic shock, the sympathetic system is not able to respond to body stressors. Therefore, the clinical characteristics of neurogenic shock are signs of parasympathetic stimulation. It is characterized by dry, warm skin rather than the cool, moist skin seen in hypovolemic shock. Another characteristic is hypotension with bradycardia, rather than the tachycardia that characterizes other forms of shock.
The nurse is caring for a patient in the ICU who has been diagnosed with multiple organ dysfunction syndrome (MODS). The nurse's plan of care should include which of the following interventions? A) Encouraging the family to stay hopeful and educating them to the fact that, in nearly all cases, the prognosis is good B) Encouraging the family to leave the hospital and to take time for themselves as acute care of MODS patients may last for several months C) Promoting communication with the patient and family along with addressing end-of-life issues D) Discussing organ donation on a number of different occasions to allow the family time to adjust to the idea
Ans: C Feedback: Promoting communication with the patient and family is a critical role of the nurse with a patient in progressive shock. It is also important that the health care team address end-of-life decisions to ensure that supportive therapies are congruent with the patient's wishes. Many cases of MODS result in death and the life expectancy of patients with MODS is usually measured in hours and possibly days, but not in months. Organ donation should be offered as an option on one occasion, and then allow the family time to discuss and return to the health care providers with an answer following the death of the patient.
The ICU nurse is caring for a patient in hypovolemic shock following a postpartum hemorrhage. For what serious complication of treatment should the nurse monitor the patient? A) Anaphylaxis B) Decreased oxygen consumption C) Abdominal compartment syndrome D) Decreased serum osmolality
Ans: C Feedback: Abdominal compartment syndrome (ACS) is a serious complication that may occur when large volumes of fluid are administered. The scenario does not describe an antigen-antibody reaction of any type. Decreased oxygen consumption by the body is not a concern in hypovolemic shock. With a decrease in fluids in the intravascular space, increased serum osmolality would occur.
The intensive care nurse caring for a patient in shock is planning assessments and interventions related to the patient's nutritional needs. What physiologic process contributes to these increased nutritional needs? A) The use of albumin as an energy source by the body because of the need for increased adenosine triphosphate (ATP) B) The loss of fluids due to decreased skin integrity and decreased stomach acids due to increased parasympathetic activity C) The release of catecholamines that creates an increase in metabolic rate and caloric requirements D) The increase in GI peristalsis during shock and the resulting diarrhea
Ans: C Feedback: Nutritional support is an important aspect of care for patients in shock. Patients in shock may require 3,000 calories daily. This caloric need is directly related to the release of catecholamines and the resulting increase in metabolic rate and caloric requirements. Albumin is not primarily metabolized as an energy source. The special nutritional needs of shock are not related to increased parasympathetic activity, but are instead related to increased sympathetic activity. GI function does not increase during shock.
A critical care nurse is aware of similarities and differences between the treatments for different types of shock. Which of the following interventions is used in all types of shock? A) Aggressive hypoglycemic control B) Administration of hypertonic IV fluids C) Early provision of nutritional support D) Aggressive antibiotic therapy
Ans: C Feedback: Nutritional support is necessary for all patients who are experiencing shock. Hyperglycemic (not hypoglycemic) control is needed for many patients. Hypertonic IV fluids are not normally utilized and antibiotics are necessary only in patients with septic shock.
The nurse is providing care for a patient who is in shock after massive blood loss from a workplace injury. The nurse recognizes that many of the findings from the most recent assessment are due to compensatory mechanisms. What is a compensatory mechanism to increase cardiac output during hypovolemic states? A) Third spacing of fluid B) Dysrhythmias C) Tachycardia D) Gastric hypermotility
Ans: C Feedback: Tachycardia is a primary compensatory mechanism to increase cardiac output during hypovolemic states. The third spacing of fluid takes fluid out of the vascular space. Gastric hypermotility and dysrhythmias would not increase cardiac output and are not considered to be compensatory mechanisms.
The nurse in a rural nursing outpost has just been notified that she will be receiving a patient in hypovolemic shock due to a massive postpartum hemorrhage after her home birth. You know that the best choice for fluid replacement for this patient is what? A) 5% albumin because it is inexpensive and is always readily available B) Dextran because it increases intravascular volume and counteracts coagulopathy C) Whatever fluid is most readily available in the clinic, due to the nature of the emergency D) Lactated Ringer's solution because it increases volume, buffers acidosis, and is the best choice for patients with liver failure
Ans: C Feedback: The best fluid to treat shock remains controversial. In emergencies, the "best" fluid is often the fluid that is readily available. Fluid resuscitation should be initiated early in shock to maximize intravascular volume. Both crystalloids and colloids can be administered to restore intravascular volume. There is no consensus regarding whether crystalloids or colloids, such as dextran and albumin, should be used; however, with crystalloids, more fluid is necessary to restore intravascular volume. Albumin is very expensive and is a blood product so it is not always readily available for use. Dextran does increase intravascular volume, but it increases the risk for coagulopathy. Lactated Ringer's is a good solution choice because it increases volume and buffers acidosis, but it should not be used in patients with liver failure because the liver is unable to covert lactate to bicarbonate.
35. A nurse is working with a patient who has been scheduled for a percutaneous coronary intervention (PCI) later in the week. What anticipatory guidance should the nurse provide to the patient? A) He will remain on bed rest for 48 to 72 hours after the procedure. B) He will be given vitamin K infusions to prevent bleeding following PCI. C) A sheath will be placed over the insertion site after the procedure is finished. D) The procedure will likely be repeated in 6 to 8 weeks to ensure success.
Ans: C Feedback: A sheath is placed over the PCI access site and kept in place until adequate coagulation is achieved. Patients resume activity a few hours after PCI and repeated treatments may or may not be necessary. Anticoagulants, not vitamin K, are administered during PCI.
24. A patient is admitted through the ED with suspected St. Louis encephalitis. The unique clinical feature of St. Louis encephalitis will make what nursing action a priority? A) Serial assessments of hemoglobin levels B) Blood glucose monitoring C) Close monitoring of fluid balance D) Assessment of pain along dermatomes
Ans: C Feedback: A unique clinical feature of St. Louis encephalitis is SIADH with hyponatremia. As such, it is important to monitor the patients intake and output closely.
25. You are writing a care plan for a patient who has been diagnosed with angina pectoris. The patient describes herself as being distressed and shocked by her new diagnosis. What nursing diagnosis is most clearly suggested by the womans statement? A) Spiritual distress related to change in health status B) Acute confusion related to prognosis for recovery C) Anxiety related to cardiac symptoms D) Deficient knowledge related to treatment of angina pectoris
Ans: C Feedback: Although further assessment is warranted, it is not unlikely that the patient is experiencing anxiety. In patients with CAD, this often relates to the threat of sudden death. There is no evidence of confusion (i.e., delirium or dementia) and there may or may not be a spiritual element to her concerns. Similarly, it is not clear that a lack of knowledge or information is the root of her anxiety.
5. Family members bring a patient to the ED with pale cool skin, sudden midsternal chest pain unrelieved with rest, and a history of CAD. How should the nurse best interpret these initial data? A) The symptoms indicate angina and should be treated as such. B) The symptoms indicate a pulmonary etiology rather than a cardiac etiology. C) The symptoms indicate an acute coronary episode and should be treated as such. D) Treatment should be determined pending the results of an exercise stress test.
Ans: C Feedback: Angina and MI have similar symptoms and are considered the same process, but are on different points along a continuum. That the patients symptoms are unrelieved by rest suggests an acute coronary episode rather than angina. Pale cool skin and sudden onset are inconsistent with a pulmonary etiology. Treatment should be initiated immediately regardless of diagnosis.
31. The nurse is participating in the care conference for a patient with ACS. What goal should guide the care teams selection of assessments, interventions, and treatments? A) Maximizing cardiac output while minimizing heart rate B) Decreasing energy expenditure of the myocardium C) Balancing myocardial oxygen supply with demand D) Increasing the size of the myocardial muscle
Ans: C Feedback: Balancing myocardial oxygen supply with demand (e.g., as evidenced by the relief of chest pain) is the top priority in the care of the patient with ACS. Treatment is not aimed directly at minimizing heart rate because some patients experience bradycardia. Increasing the size of the myocardium is never a goal. Reducing the myocardiums energy expenditure is often beneficial, but this must be balanced with productivity.
28. A patient with a brain tumor has begun to exhibit signs of cachexia. What subsequent assessment should the nurse prioritize? A) Assessment of peripheral nervous function B) Assessment of cranial nerve function C) Assessment of nutritional status D) Assessment of respiratory status
Ans: C Feedback: Cachexia is a wasting syndrome of weight loss, muscle atrophy, fatigue, weakness, and significant loss of appetite. Consequently, nutritional assessment is paramount.
40. A patient is recovering in the hospital from cardiac surgery. The nurse has identified the diagnosis of risk for ineffective airway clearance related to pulmonary secretions. What intervention best addresses this risk? A) Administration of bronchodilators by nebulizer B) Administration of inhaled corticosteroids by metered dose inhaler (MDI) C) Patients consistent performance of deep breathing and coughing exercises D) Patients active participation in the cardiac rehabilitation program
Ans: C Feedback: Clearance of pulmonary secretions is accomplished by frequent repositioning of the patient, suctioning, and chest physical therapy, as well as educating and encouraging the patient to breathe deeply and cough. Medications are not normally used to achieve this goal. Rehabilitation is important, but will not necessarily aid the mobilization of respiratory secretions.
23. A patient is being admitted to the neurologic ICU with suspected herpes simplex virus encephalitis. What nursing action best addresses the patients complaints of headache? A) Initiating a patient-controlled analgesia (PCA) of morphine sulfate B) Administering hydromorphone (Dilaudid) IV as needed C) Dimming the lights and reducing stimulation Test Bank - Brunner & Suddarth's Textbook of Medical-Surgical Nursing 14e (Hinkle 2017) 1305 D) Distracting the patient with activity
Ans: C Feedback: Comfort measures to reduce headache include dimming the lights, limiting noise and visitors, grouping nursing interventions, and administering analgesic agents. Opioid analgesic medications may mask neurologic symptoms; therefore, they are used cautiously. Non-opioid analgesics may be preferred. Distraction is unlikely to be effective, and may exacerbate the patients pain.
4. A patient with Guillain-Barr syndrome has experienced a sharp decline in vital capacity. What is the nurses most appropriate action? A) Administer bronchodilators as ordered. B) Remind the patient of the importance of deep breathing and coughing exercises. C) Prepare to assist with intubation. D) Administer supplementary oxygen by nasal cannula.
Ans: C Feedback: For the patient with Guillain-Barr syndrome, mechanical ventilation is required if the vital capacity falls, making spontaneous breathing impossible and tissue oxygenation inadequate. Each of the other listed actions is likely insufficient to meet the patients oxygenation needs.
31. A patient, brought to the clinic by his wife and son, is diagnosed with Huntington disease. When providing anticipatory guidance, the nurse should address the future possibility of what effect of Huntington disease? A) Metastasis B) Risk for stroke C) Emotional and personality changes D) Pathologic bone fractures
Ans: C Feedback: Huntington disease causes profound changes to personality and behavior. It is a nonmalignant disease and stroke is not a central risk. The disease is not associated with pathologic bone fractures.
21. The nurse is assessing a patient who was admitted to the critical care unit 3 hours ago following cardiac surgery. The nurses most recent assessment reveals that the patients left pedal pulses are not palpable and that the right pedal pulses are rated at +2. What is the nurses best response? A) Document this expected assessment finding during the initial postoperative period. B) Reposition the patient with his left leg in a dependent position. C) Inform the patients physician of this assessment finding. D) Administer an ordered dose of subcutaneous heparin.
Ans: C Feedback: If a pulse is absent in any extremity, the cause may be prior catheterization of that extremity, chronic peripheral vascular disease, or a thromboembolic obstruction. The nurse immediately reports newly identified absence of any pulse.
39. The nurse responds to the call light of a patient who has had a cervical diskectomy earlier in the day. The patient states that she is having severe pain that had a sudden onset. What is the nurses most appropriate action? A) Palpate the surgical site. B) Remove the dressing to assess the surgical site. C) Call the surgeon to report the patients pain. D) Administer a dose of an NSAID.
Ans: C Feedback: If the patient experiences a sudden increase in pain, extrusion of the graft may have occurred, requiring reoperation. A sudden increase in pain should be promptly reported to the surgeon. Administration of an NSAID would be an insufficient response and the dressing should not be removed without an order. Palpation could cause further damage.
31. The nurse is discharging a patient home after surgery for trigeminal neuralgia. What advice should the nurse provide to this patient in order to reduce the risk of injury? A) Avoid watching television or using a computer for more than 1 hour at a time. B) Use OTC antibiotic eye drops for at least 14 days. C) Avoid rubbing the eye on the affected side of the face. D) Rinse the eye on the affected side with normal saline daily for 1 week.
Ans: C Feedback: If the surgery results in sensory deficits to the affected side of the face, the patient is instructed not to rub the eye because the pain of a resulting injury will not be detected. There is no need to limit TV viewing or to rinse the eye daily. Antibiotics may or may not be prescribed, and these would not reduce the risk of injury.
20. A patient with amyotrophic lateral sclerosis (ALS) is being visited by the home health nurse who is creating a care plan. What nursing diagnosis is most likely for a patient with this condition? A) Chronic confusion B) Impaired urinary elimination C) Impaired verbal communication D) Bowel incontinence
Ans: C Feedback: Impaired communication is an appropriate nursing diagnosis; the voice in patients with ALS assumes a nasal sound and articulation becomes so disrupted that speech is unintelligible. Intellectual function is marginally impaired in patients with late ALS. Usually, the anal and bladder sphincters are intact Test Bank - Brunner & Suddarth's Textbook of Medical-Surgical Nursing 14e (Hinkle 2017) 1322 because the spinal nerves that control muscles of the rectum and urinary bladder are not affected.
12. The nurse is providing an educational workshop about coronary artery disease (CAD) and its risk factors. The nurse explains to participants that CAD has many risk factors, some that can be controlled and some that cannot. What risk factors would the nurse list that can be controlled or modified? A) Gender, obesity, family history, and smoking B) Inactivity, stress, gender, and smoking C) Obesity, inactivity, diet, and smoking D) Stress, family history, and obesity
Ans: C Feedback: The risk factors for CAD that can be controlled or modified include obesity, inactivity, diet, stress, and smoking. Gender and family history are risk factors that cannot be controlled.
3. The nurse is caring for an adult patient who had symptoms of unstable angina upon admission to the hospital. What nursing diagnosis underlies the discomfort associated with angina? A) Ineffective breathing pattern related to decreased cardiac output B) Anxiety related to fear of death C) Ineffective cardiopulmonary tissue perfusion related to coronary artery disease (CAD) D) Impaired skin integrity related to CAD
Ans: C Feedback: Ineffective cardiopulmonary tissue perfusion directly results in the symptoms of discomfort associated with angina. Anxiety and ineffective breathing may result from angina chest pain, but they are not the causes. Skin integrity is not impaired by the effects of angina.
35. A patient with Parkinsons disease is experiencing episodes of constipation that are becoming increasingly frequent and severe. The patient states that he has been achieving relief for the past few weeks by using OTC laxatives. How should the nurse respond? A) Its important to drink plenty of fluids while youre taking laxatives. B) Make sure that you supplement your laxatives with a nutritious diet. C) Lets explore other options, because laxatives can have side effects and create dependency. D) You should ideally be using herbal remedies rather than medications to promote bowel function.
Ans: C Feedback: Laxatives should be avoided in patients with Parkinsons disease due to the risk of adverse effects and dependence. Herbal bowel remedies are not necessarily less risky.
39. A patient who is postoperative day 1 following a CABG has produced 20 mL of urine in the past 3 hours and the nurse has confirmed the patency of the urinary catheter. What is the nurses most appropriate action? A) Document the patients low urine output and monitor closely for the next several hours. B) Contact the dietitian and suggest the need for increased oral fluid intake. C) Contact the patients physician and suggest assessment of fluid balance and renal function. D) Increase the infusion rate of the patients IV fluid to prompt an increase in renal function.
Ans: C Feedback: Nursing management includes accurate measurement of urine output. An output of less than 1 mL/kg/h may indicate hypovolemia or renal insufficiency. Prompt referral is necessary. IV fluid replacement may be indicated, but is beyond the independent scope of the dietitian or nurse.
19. A 33-year-old patient presents at the clinic with complaints of weakness, incoordination, dizziness, and Test Bank - Brunner & Suddarth's Textbook of Medical-Surgical Nursing 14e (Hinkle 2017) 1303 loss of balance. The patient is hospitalized and diagnosed with MS. What sign or symptom, revealed during the initial assessment, is typical of MS? A) Diplopia, history of increased fatigue, and decreased or absent deep tendon reflexes B) Flexor spasm, clonus, and negative Babinskis reflex C) Blurred vision, intention tremor, and urinary hesitancy D) Hyperactive abdominal reflexes and history of unsteady gait and episodic paresthesia in both legs
Ans: C Feedback: Optic neuritis, leading to blurred vision, is a common early sign of MS, as is intention tremor (tremor when performing an activity). Nerve damage can cause urinary hesitancy. In MS, deep tendon reflexes are increased or hyperactive. A positive Babinskis reflex is found in MS. Abdominal reflexes are absent with MS.
3. The nurse is writing a care plan for a patient with brain metastases. The nurse decides that an appropriate nursing diagnosis is anxiety related to lack of control over the health circumstances. In establishing this plan of care for the patient, the nurse should include what intervention? A) The patient will receive antianxiety medications every 4 hours. B) The patients family will be instructed on planning the patients care. C) The patient will be encouraged to verbalize concerns related to the disease and its treatment. D) The patient will begin intensive therapy with the goal of distraction.
Ans: C Feedback: Patients need the opportunity to exercise some control over their situation. A sense of mastery can be gained as they learn to understand the disease and its treatment and how to deal with their feelings. Distraction and administering medications will not allow the patient to gain control over anxiety. Delegating planning to the family will not help the patient gain a sense of control and autonomy.
39. A 48-year-old patient has been diagnosed with trigeminal neuralgia following recent episodes of unilateral face pain. The nurse should recognize what implication of this diagnosis? Test Bank - Brunner & Suddarth's Textbook of Medical-Surgical Nursing 14e (Hinkle 2017) 1312 A) The patient will likely require lifelong treatment with anticholinergic medications. B) The patient has a disproportionate risk of developing myasthenia gravis later in life. C) The patient needs to be assessed for MS. D) The disease is self-limiting and the patient will achieve pain relief over time.
Ans: C Feedback: Patients that develop trigeminal neuralgia before age 50 should be evaluated for the coexistent of MS because trigeminal neuralgia occurs in approximately 5% of patients with MS. Treatment does not include anticholinergics and the disease is not self-limiting. Trigeminal neuralgia is not associated with an increased risk of myasthenia gravis.
22. The nurse is caring for a patient newly diagnosed with a primary brain tumor. The patient asks the nurse where his tumor came from. What would be the nurses best response? A) Your tumor originated from somewhere outside the CNS. B) Your tumor likely started out in one of your glands. C) Your tumor originated from cells within your brain itself. D) Your tumor is from nerve tissue somewhere in your body.
Ans: C Feedback: Primary brain tumors originate from cells and structures within the brain. Secondary brain tumors are metastatic tumors that originate somewhere else in the body. The scenario does not indicate that the patients tumor is a pituitary tumor or a neuroma.
17. A nurse is planning the care of a 28-year-old woman hospitalized with a diagnosis of myasthenia gravis. What approach would be most appropriate for the care and scheduling of diagnostic procedures for this patient? A) All at one time, to provide a longer rest period B) Before meals, to stimulate her appetite C) In the morning, with frequent rest periods D) Before bedtime, to promote rest
Ans: C Feedback: Procedures should be spaced to allow for rest in between. Procedures should be avoided before meals, or the patient may be too exhausted to eat. Procedures should be avoided near bedtime if possible.
28. The ED nurse is caring for a patient with a suspected MI. What drug should the nurse anticipate administering to this patient? A) Oxycodone B) Warfarin C) Morphine D) Acetaminophen
Ans: C Feedback: The patient with suspected MI is given aspirin, nitroglycerin, morphine, an IV beta- blocker, and other medications, as indicated, while the diagnosis is being confirmed. Tylenol, warfarin, and oxycodone are not typically used.
22. The critical care nurse is caring for 25-year-old man admitted to the ICU with a brain abscess. What is a priority nursing responsibility in the care of this patient? A) Maintaining the patients functional independence B) Providing health education C) Monitoring neurologic status closely D) Promoting mobility
Ans: C Feedback: Vigilant neurologic monitoring is a key aspect of caring for a patient who has a brain abscess. This supersedes education, ADLs, and mobility, even though these are all valid and important aspects of nursing care.
21. The nurse educator is discussing neoplasms with a group of recent graduates. The educator explains that the effects of neoplasms are caused by the compression and infiltration of normal tissue. The physiologic changes that result can cause what pathophysiologic events? Select all that apply. A) Intracranial hemorrhage B) Infection of cerebrospinal fluid C) Increased ICP D) Focal neurologic signs E) Altered pituitary function
Ans: C, D, E Feedback: The effects of neoplasms are caused by the compression and infiltration of tissue. A variety of physiologic changes result, causing any or all of the following pathophysiologic events: increased ICP and cerebral edema, seizure activity and focal neurologic signs, hydrocephalus, and altered pituitary function.
The emergency nurse is admitting a patient experiencing a GI bleed who is believed to be in the compensatory stage of shock. What assessment finding would be most consistent with the early stage of compensation? A) Increased urine output B) Decreased heart rate C) Hyperactive bowel sounds D) Cool, clammy skin
Ans: D Feedback: In the compensatory stage of shock, the body shunts blood from the organs, such as the skin and kidneys, to the brain and heart to ensure adequate blood supply. As a result, the patient's skin is cool and clammy. Also in this compensatory stage, blood vessels vasoconstrict, the heart rate increases, bowel sounds are hypoactive, and the urine output decreases.
Sepsis is an evolving process, with neither clearly definable clinical signs and symptoms nor predictable progression. As the ICU nurse caring for a patient with sepsis, the nurse knows that tissue perfusion declines during sepsis and the patient begins to show signs of organ dysfunction. What sign would indicate to the nurse that end-organ damage may be occurring? A) Urinary output increases B) Skin becomes warm and dry C) Adventitious lung sounds occur in the upper airway D) Heart and respiratory rates are elevated
Ans: D Feedback: As sepsis progresses, tissues become less perfused and acidotic, compensation begins to fail, and the patient begins to show signs of organ dysfunction. The cardiovascular system also begins to fail, the blood pressure does not respond to fluid resuscitation and vasoactive agents, and signs of end-organ damage are evident (e.g., renal failure, pulmonary failure, hepatic failure). As sepsis progresses to septic shock, the blood pressure drops, and the skin becomes cool, pale, and mottled. Temperature may be normal or below normal. Heart and respiratory rates remain rapid. Urine production ceases, and multiple organ dysfunction progressing to death occurs. Adventitious lung sounds occur throughout the lung fields, not just in the upper fields of the lungs.
An immunocompromised older adult has developed a urinary tract infection and the care team recognizes the need to prevent an exacerbation of the patient's infection that could result in urosepsis and septic shock. What action should the nurse perform to reduce the patient's risk of septic shock? A) Apply an antibiotic ointment to the patient's mucous membranes, as ordered. B) Perform passive range-of-motion exercises unless contraindicated C) Initiate total parenteral nutrition (TPN) D) Remove invasive devices as soon as they are no longer needed
Ans: D Feedback: Early removal of invasive devices can reduce the incidence of infections. Broad application of antibiotic ointments is not performed. TPN may be needed, but this does not directly reduce the risk of further infection. Range-of-motion exercises are not a relevant intervention.
The nurse is caring for a patient admitted with cardiogenic shock. The patient is experiencing chest pain and there is an order for the administration of morphine. In addition to pain control, what is the main rationale for administering morphine to this patient? A) It promotes coping and slows catecholamine release. B) It stimulates the patient so he or she is more alert. C) It decreases gastric secretions. D) It dilates the blood vessels.
Ans: D Feedback: For patients experiencing chest pain, morphine is the drug of choice because it dilates the blood vessels and controls the patient's anxiety. Morphine would not be ordered to promote coping or to stimulate the patient. The rationale behind using morphine would not be to decrease gastric secretions.
11. The nurse caring for a patient diagnosed with Guillain-Barr syndrome is planning care with regard to the clinical manifestations associated this syndrome. The nurses communication with the patient should reflect the possibility of what sign or symptom of the disease? A) Intermittent hearing loss B) Tinnitus C) Tongue enlargement D) Vocal paralysis
Ans: D Feedback: Guillain-Barr syndrome is a disorder of the vagus nerve. Clinical manifestations include vocal paralysis, dysphagia, and voice changes (temporary or permanent hoarseness). Hearing deficits, tinnitus, and tongue enlargement are not associated with the disease.
In all types of shock, nutritional demands increase rapidly as the body depletes its stores of glycogen. Enteral nutrition is the preferred method of meeting these increasing energy demands. What is the basis for enteral nutrition being the preferred method of meeting the body's needs? A) It slows the proliferation of bacteria and viruses during shock. B) It decreases the energy expended through the functioning of the GI system. C) It assists in expanding the intravascular volume of the body. D) It promotes GI function through direct exposure to nutrients.
Ans: D Feedback: Parenteral or enteral nutritional support should be initiated as soon as possible. Enteral nutrition is preferred, promoting GI function through direct exposure to nutrients and limiting infectious complications associated with parenteral feeding. Enteral feeding does not decrease the proliferation of microorganisms or the amount of energy expended through the functioning of the GI system and it does not assist in expanding the intravascular volume of the body.
When caring for a patient in shock, one of the major nursing goals is to reduce the risk that the patient will develop complications of shock. How can the nurse best achieve this goal? A) Provide a detailed diagnosis and plan of care in order to promote the patient's and family's coping. B) Keep the physician updated with the most accurate information because in cases of shock the nurse often cannot provide relevant interventions. C) Monitor for significant changes and evaluate patient outcomes on a scheduled basis focusing on blood pressure and skin temperature. D) Understand the underlying mechanisms of shock, recognize the subtle and more obvious signs, and then provide rapid assessment.
Ans: D Feedback: Shock is a life-threatening condition with a variety of underlying causes. It is critical that the nurse apply the nursing process as the guide for care. Shock is unpredictable and rapidly changing so the nurse must understand the underlying mechanisms of shock. The nurse must also be able to recognize the subtle as well as more obvious signs and then provide rapid assessment and response to provide the patient with the best chance for recovery. Coping skills are important, but not the ultimate priority. Keeping the physician updated with the most accurate information is important, but the nurse is in the best position to provide rapid assessment and response, which gives the patient the best chance for survival. Monitoring for significant changes is critical, and evaluating patient outcomes is always a part of the nursing process, but the subtle signs and symptoms of shock are as important as the more obvious signs, such as blood pressure and skin temperature. Assessment must lead to diagnosis and interventions.
The critical care nurse is preparing to initiate an infusion of a vasoactive medication to a patient in shock. The nurse knows that vasoactive medications are given in all forms of shock. What is the primary goal of this aspect of treatment? A) To prevent the formation of infarcts of emboli B) To limit stroke volume and cardiac output C) To prevent pulmonary and peripheral edema D) To maintain adequate mean arterial pressure
Ans: D Feedback: Vasoactive medications can be administered in all forms of shock to improve the patient's hemodynamic stability when fluid therapy alone cannot maintain adequate MAP. Specific medications are selected to correct the particular hemodynamic alteration that is impeding cardiac output. These medications help increase the strength of myocardial contractility, regulate the heart rate, reduce myocardial resistance, and initiate vasoconstriction. They are not specifically used to prevent emboli, edema, or infarcts.
14. The public health nurse is participating in a health fair and interviews a patient with a history of hypertension, who is currently smoking one pack of cigarettes per day. She denies any of the most common manifestations of CAD. Based on these data, the nurse would expect the focuses of CAD treatment most likely to be which of the following? A) Drug therapy and smoking cessation B) Diet and drug therapy C) Diet therapy only D) Diet therapy and smoking cessation
Ans: D Feedback: Due to the absence of symptoms, dietary therapy would likely be selected as the first-line treatment for possible CAD. Drug therapy would be determined based on a number of considerations and diagnostics findings, but would not be directly indicated. Smoking cessation is always indicated, regardless of the presence or absence of symptoms.
7. The nurse is creating a plan of care for a patient who has a recent diagnosis of MS. Which of the following should the nurse include in the patients care plan? A) Encourage patient to void every hour. B) Order a low-residue diet. C) Provide total assistance with all ADLs. Test Bank - Brunner & Suddarth's Textbook of Medical-Surgical Nursing 14e (Hinkle 2017) 1298 D) Instruct the patient on daily muscle stretching.
Ans: D Feedback: A patient diagnosed with MS should be encouraged to increase the fiber in his or her diet and void 30 minutes after drinking to help train the bladder. The patient should participate in daily muscle stretching to help alleviate and relax muscle spasms.
32. The nurse working on the coronary care unit is caring for a patient with ACS. How can the nurse best meet the patients psychosocial needs? A) Reinforce the fact that treatment will be successful. B) Facilitate a referral to a chaplain or spiritual leader. C) Increase the patients participation in rehabilitation activities. D) Directly address the patients anxieties and fears.
Ans: D Feedback: Alleviating anxiety and decreasing fear are important nursing functions that reduce the sympathetic stress response. Referrals to spiritual care may or may not be appropriate, and this does not relieve the nurse of responsibility for addressing the patients psychosocial needs. Treatment is not always successful, and false hope should never be fostered. Participation in rehabilitation may alleviate anxiety for some patients, but it may exacerbate it for others.
28. The nurse caring for a patient in ICU diagnosed with Guillain-Barr syndrome should prioritize monitoring for what potential complication? A) Impaired skin integrity B) Cognitive deficits C) Hemorrhage D) Autonomic dysfunction
Ans: D Feedback: Based on the assessment data, potential complications that may develop include respiratory failure and autonomic dysfunction. Skin breakdown, decreased cognition, and hemorrhage are not complications of Guillain-Barr syndrome.
2. A patient presents to the walk-in clinic complaining of intermittent chest pain on exertion, which is eventually attributed to angina. The nurse should inform the patient that angina is most often attributable to what cause? A) Decreased cardiac output B) Decreased cardiac contractility C) Infarction of the myocardium D) Coronary arteriosclerosis
Ans: D Feedback: In most cases, angina pectoris is due to arteriosclerosis. The disease is not a result of impaired cardiac output or contractility. Infarction may result from untreated angina, but it is not a cause of the disease.
11. A patient has just been diagnosed with Parkinsons disease and the nurse is planning the patients subsequent care for the home setting. What nursing diagnosis should the nurse address when educating the patients family? A) Risk for infection B) Impaired spontaneous ventilation C) Unilateral neglect D) Risk for injury
Ans: D Feedback: Individuals with Parkinsons disease face a significant risk for injury related to the effects of dyskinesia. Unilateral neglect is not characteristic of the disease, which affects both sides of the body. Parkinsons disease does not directly constitute a risk for infection or impaired respiration. Test Bank - Brunner & Suddarth's Textbook of Medical-Surgical Nursing 14e (Hinkle 2017) 1318
36. Preoperative education is an important part of the nursing care of patients having coronary artery revascularization. When explaining the pre- and postoperative regimens, the nurse would be sure to include education about which subject? A) Symptoms of hypovolemia B) Symptoms of low blood pressure C) Complications requiring graft removal D) Intubation and mechanical ventilation
Ans: D Feedback: Most patients remain intubated and on mechanical ventilation for several hours after surgery. It is important that patients realize that this will prevent them from talking, and the nurse should reassure them that the staff will be able to assist them with other means of communication. Teaching would generally not include symptoms of low blood pressure or hypovolemia, as these are not applicable to most patients. Teaching would also generally not include rare complications that would require graft removal.
12. The nurse is preparing to provide care for a patient diagnosed with myasthenia gravis. The nurse should know that the signs and symptoms of the disease are the result of what? Test Bank - Brunner & Suddarth's Textbook of Medical-Surgical Nursing 14e (Hinkle 2017) 1300 A) Genetic dysfunction B) Upper and lower motor neuron lesions C) Decreased conduction of impulses in an upper motor neuron lesion D) A lower motor neuron lesion
Ans: D Feedback: Myasthenia gravis is characterized by a weakness of muscles, especially in the face and throat, caused by a lower neuron lesion at the myoneural junction. It is not a genetic disorder. A combined upper and lower neuron lesion generally occurs as a result of spinal injuries. A lesion involving cranial nerves and their axons in the spinal cord would cause decreased conduction of impulses at an upper motor neuron.
10. A patient, diagnosed with cancer of the lung, has just been told he has metastases to the brain. What change in health status would the nurse attribute to the patients metastatic brain disease? A) Chronic pain B) Respiratory distress C) Fixed pupils D) Personality changes
Ans: D Feedback: Neurologic signs and symptoms include headache, gait disturbances, visual impairment, personality changes, altered mentation (memory loss and confusion), focal weakness, paralysis, aphasia, and seizures. Pain, respiratory distress, and fixed pupils are not among the more common neurologic signs and symptoms of metastatic brain disease.
12. The nurse is caring for a patient with Huntington disease who has been admitted to the hospital for treatment of malnutrition. What independent nursing action should be implemented in the patients plan of care? A) Firmly redirect the patients head when feeding. B) Administer phenothiazines after each meal as ordered. C) Encourage the patient to keep his or her feeding area clean. D) Apply deep, gentle pressure around the patients mouth to aid swallowing.
Ans: D Feedback: Nursing interventions for a patient who has inadequate nutritional intake should include the following: Apply deep gentle pressure around the patients mouth to assist with swallowing, and administer phenothiazines prior to the patients meal as ordered. The nurse should disregard the mess of the feeding area and treat the person with dignity. Stiffness and turning away by the patient during feeding are uncontrollable choreiform movements and should not be interrupted.
13. A patient with suspected Creutzfeldt-Jakob disease (CJD) is being admitted to the unit. The nurse would expect what diagnostic test to be ordered for this patient? A) Cerebral angiography B) ABG analysis C) CT D) EEG
Ans: D Feedback: The EEG reveals a characteristic pattern over the duration of CJD. A CT scan may be used to rule out disorders that may mimic the symptoms of CJD. ABGs would not be necessary until the later stages of CJD; they would not be utilized as a diagnostic test. Cerebral angiography is not used to diagnose CJD.
7. A 37-year-old man is brought to the clinic by his wife because he is experiencing loss of motor function and sensation. The physician suspects the patient has a spinal cord tumor and hospitalizes him for diagnostic testing. In light of the need to diagnose spinal cord compression from a tumor, the nurse will most likely prepare the patient for what test? A) Anterior-posterior x-ray B) Ultrasound Test Bank - Brunner & Suddarth's Textbook of Medical-Surgical Nursing 14e (Hinkle 2017) 1316 C) Lumbar puncture D) MRI
Ans: D Feedback: The MRI scan is the most commonly used diagnostic procedure. It is the most sensitive diagnostic tool that is particularly helpful in detecting epidural spinal cord compression and vertebral bone metastases.
32. A patient who was diagnosed with Parkinsons disease several months ago recently began treatment with levodopa-carbidopa. The patient and his family are excited that he has experienced significant symptom relief. The nurse should be aware of what implication of the patients medication regimen? Test Bank - Brunner & Suddarth's Textbook of Medical-Surgical Nursing 14e (Hinkle 2017) 1327 A) The patient is in a honeymoon period when adverse effects of levodopa-carbidopa are not yet evident. B) Benefits of levodopa-carbidopa do not peak until 6 to 9 months after the initiation of treatment. C) The patients temporary improvement in status is likely unrelated to levodopa-carbidopa. D) Benefits of levodopa-carbidopa often diminish after 1 or 2 years of treatment.
Ans: D Feedback: The beneficial effects of levodopa therapy are most pronounced in the first year or two of treatment. Benefits begin to wane and adverse effects become more severe over time. However, a honeymoon period of treatment is not known.
17. The nurse is providing care for a patient with high cholesterol and triglyceride values. In teaching the patient about therapeutic lifestyle changes such as diet and exercise, the nurse realizes that the desired goal for cholesterol levels is which of the following? A) High HDL values and high triglyceride values B) Absence of detectable total cholesterol levels C) Elevated blood lipids, fasting glucose less than 100 D) Low LDL values and high HDL values
Ans: D Feedback: The desired goal for cholesterol readings is for a patient to have low LDL and high HDL values. LDL exerts a harmful effect on the coronary vasculature because the small LDL particles can be easily transported into the vessel lining. In contrast, HDL promotes the use of total cholesterol by transporting LDL to the liver, where it is excreted. Elevated triglycerides are also a major risk factor for cardiovascular disease. A goal is also to keep triglyceride levels less than 150 mg/dL. All individuals possess detectable levels of total cholesterol.
36. A 73-year-old man comes to the clinic complaining of weakness and loss of sensation in his feet and legs. Assessment of the patient shows decreased reflexes bilaterally. Why would it be a challenge to diagnose a peripheral neuropathy in this patient? A) Older adults are often vague historians. B) The elderly have fewer peripheral nerves than younger adults. C) Many older adults are hesitant to admit that their body is changing. D) Many symptoms can be the result of normal aging process.
Ans: D Feedback: The diagnosis of peripheral neuropathy in the geriatric population is challenging because many symptoms, such as decreased reflexes, can be associated with the normal aging process. In this scenario, Test Bank - Brunner & Suddarth's Textbook of Medical-Surgical Nursing 14e (Hinkle 2017) 1311 the patient has come to the clinic seeking help for his problem; this does not indicate a desire on the part of the patient to withhold information from the health care giver. The normal aging process does not include a diminishing number of peripheral nerves.
37. A patient in the cardiac step-down unit has begun bleeding from the percutaneous coronary intervention (PCI) access site in her femoral region. What is the nurses most appropriate action? A) Call for assistance and initiate cardiopulmonary resuscitation. B) Reposition the patients leg in a nondependent position. C) Promptly remove the femoral sheath. D) Call for help and apply pressure to the access site.
Ans: D Feedback: The femoral sheath produces pressure on the access site. Pressure will temporarily reduce bleeding and allow for subsequent interventions. Removing the sheath would exacerbate bleeding and repositioning would not halt it. CPR is not indicated unless there is evidence of respiratory or cardiac arrest.
6. An OR nurse is preparing to assist with a coronary artery bypass graft (CABG). The OR nurse knows that the vessel most commonly used as source for a CABG is what? A) Brachial artery B) Brachial vein C) Femoral artery D) Greater saphenous vein
Ans: D Feedback: The greater saphenous vein is the most commonly used graft site for CABG. The right and left internal mammary arteries, radial arteries, and gastroepiploic artery are other graft sites used, though not as frequently. The femoral artery, brachial artery, and brachial vein are never harvested.
40. A patient presents at the clinic complaining of pain and weakness in her hands. On assessment, the nurse notes diminished reflexes in the upper extremities bilaterally and bilateral loss of sensation. The nurse knows that these findings are indicative of what? A) Guillain-Barr syndrome B) Myasthenia gravis C) Trigeminal neuralgia D) Peripheral nerve disorder
Ans: D Feedback: The major symptoms of peripheral nerve disorders are loss of sensation, muscle atrophy, weakness, diminished reflexes, pain, and paresthesia (numbness, tingling) of the extremities. Trigeminal neuralgia is a condition of the fifth cranial nerve that is characterized by paroxysms of pain in the area innervated by any of the three branches, but most commonly the second and third branches of the trigeminal nerve. Myasthenia gravis, an autoimmune disorder affecting the myoneural junction, is characterized by varying degrees of weakness of the voluntary muscles. Guillain-Barr syndrome is an autoimmune attack on the peripheral nerve myelin.
15. A male patient with a metastatic brain tumor is having a generalized seizure and begins vomiting. What should the nurse do first? A) Perform oral suctioning. B) Page the physician. C) Insert a tongue depressor into the patients mouth. D) Turn the patient on his side.
Ans: D Feedback: The nurses first response should be to place the patient on his side to prevent him from aspirating emesis. Inserting something into the seizing patients mouth is no longer part of a seizure protocol. Obtaining supplies to suction the patient would be a delegated task. Paging or calling the physician would only be necessary if this is the patients first seizure.
16. A middle-aged woman has sought care from her primary care provider and undergone diagnostic testing that has resulted in a diagnosis of MS. What sign or symptom is most likely to have prompted the woman to seek care? A) Cognitive declines B) Personality changes C) Contractures D) Difficulty in coordination
Ans: D Test Bank - Brunner & Suddarth's Textbook of Medical-Surgical Nursing 14e (Hinkle 2017) 1302 Feedback: The primary symptoms of MS most commonly reported are fatigue, depression, weakness, numbness, difficulty in coordination, loss of balance, spasticity, and pain. Cognitive changes and contractures usually occur later in the disease.
9. While assessing the patient at the beginning of the shift, the nurse inspects a surgical dressing covering the operative site after the patients cervical diskectomy. The nurse notes that the drainage is 75% saturated with serosanguineous discharge. What is the nurses most appropriate action? A) Page the physician and report this sign of infection. B) Reinforce the dressing and reassess in 1 to 2 hours. C) Reposition the patient to prevent further hemorrhage. D) Inform the surgeon of the possibility of a dural leak.
Ans: D Test Bank - Brunner & Suddarth's Textbook of Medical-Surgical Nursing 14e (Hinkle 2017) 1317 Feedback: After a cervical diskectomy, the nurse will monitor the operative site and dressing covering this site. Serosanguineous drainage may indicate a dural leak. This constitutes a risk for meningitis, but is not a direct sign of infection. This should be reported to the surgeon, not just reinforced and observed.
23. An ED nurse is assessing an adult woman for a suspected MI. When planning the assessment, the nurse should be cognizant of what signs and symptoms of MI that are particularly common in female patients? Select all that apply. A) Shortness of breath B) Chest pain C) Anxiety D) Numbness E) Weakness
Ans: D, E Feedback: Although these symptoms are not wholly absent in men, many women have been found to have atypical symptoms of MI, including indigestion, nausea, palpitations, and numbness. Shortness of breath, chest pain, and anxiety are common symptoms of MI among patients of all ages and genders.
The cardiac nurse is caring for a patient who has been diagnosed with dilated cardiomyopathy (DCM). Echocardiography is likely to reveal what pathophysiological finding? A) Decreased ejection fraction B) Decreased heart rate C) Ventricular hypertrophy D) Mitral valve regurgitation
Ans: Decreased ejection fraction Feedback: DCM is distinguished by significant dilation of the ventricles without simultaneous hypertrophy. The ventricles have elevated systolic and diastolic volumes, but a decreased ejection fraction. Bradycardia and mitral valve regurgitation do not typically occur in patients with DCM.
A patient who has undergone a valve replacement with a mechanical valve prosthesis is due to be discharged home. During discharge teaching, the nurse should discuss the importance of antibiotic prophylaxis prior to which of the following? A) Exposure to immunocompromised individuals B) Future hospital admissions C) Dental procedures D) Live vaccinations
Ans: Dental procedures Feedback: Following mechanical valve replacement, antibiotic prophylaxis is necessary before dental procedures involving manipulation of gingival tissue, the periapical area of the teeth or perforation of the oral mucosa (not including routine anesthetic injections, placement of orthodontic brackets, or loss of deciduous teeth). There are no current recommendations around antibiotic prophylaxis prior to vaccination, future hospital admissions, or exposure to people who are immunosuppressed.
The nurse is reviewing the echocardiography results of a patient who has just been diagnosed with dilated cardiomyopathy (DCM). What changes in heart structure characterize DCM? A) Dilated ventricles with atrophy of the ventricles B) Dilated ventricles without hypertrophy of the ventricles C) Dilation and hypertrophy of all four heart chambers D) Dilation of the atria and hypertrophy of the ventricles
Ans: Dilated ventricles without hypertrophy of the ventricles Feedback: DCM is characterized by significant dilation of the ventricles without significant concomitant hypertrophy and systolic dysfunction. The ventricles do not atrophy in patients with DCM.
The patient has just returned to the floor after balloon valvuloplasty of the aortic valve and the nurse is planning appropriate assessments. The nurse should know that complications following this procedure include what? Select all that apply. A) Emboli B) Mitral valve damage C) Ventricular dysrhythmia D) Atrial-septal defect E) Plaque formation
Ans: Emboli, Mitral valve damage, Ventricular dysrhythmia Feedback: Possible complications include aortic regurgitation, emboli, ventricular perforation, rupture of the aortic valve annulus, ventricular dysrhythmia, mitral valve damage, and bleeding from the catheter insertion sites. Atrial-septal defect and plaque formation are not complications of a balloon valvuloplasty.
A patient who has recently recovered from a systemic viral infection is undergoing diagnostic testing for myocarditis. Which of the nurse's assessment findings is most consistent with myocarditis? A) Sudden changes in level of consciousness (LOC) B) Peripheral edema and pulmonary edema C) Pleuritic chest pain D) Flulike symptoms
Ans: Flulike symptoms Feedback: The most common symptoms of myocarditis are flulike. Chest pain, edema, and changes in LOC are not characteristic of myocarditis.
A patient has been living with dilated cardiomyopathy for several years but has experienced worsening symptoms despite aggressive medical management. The nurse should anticipate what potential treatment? A) Heart transplantation B) Balloon valvuloplasty C) Cardiac catheterization D) Stent placement
Ans: Heart transplantation Feedback: When heart failure progresses and medical treatment is no longer effective, surgical intervention, including heart transplantation, is considered. Valvuloplasty, stent placement, and cardiac catheterization will not address the pathophysiology of cardiomyopathy.
The nurse on the hospital's infection control committee is looking into two cases of hospital-acquired infective endocarditis among a specific classification of patients. What classification of patients would be at greatest risk for hospital-acquired endocarditis? A) Hemodialysis patients B) Patients on immunoglobulins C) Patients who undergo intermittent urinary catheterization D) Children under the age of 12
Ans: Hemodialysis patients Feedback: Hospital-acquired infective endocarditis occurs most often in patients with debilitating disease or indwelling catheters and in patients who are receiving hemodialysis or prolonged IV fluid or antibiotic therapy. Patients taking immunosuppressive medications or corticosteroids are more susceptible to fungal endocarditis. Patients on immunoglobulins, those who need in and out catheterization, and children are not at increased risk for nosocomial infective endocarditis.
A 17-year-old boy is being treated in the ICU after going into cardiac arrest during a football practice. Diagnostic testing reveals cardiomyopathy as the cause of the arrest. What type of cardiomyopathy is particularly common among young people who appear otherwise healthy? A) Dilated cardiomyopathy (DCM). B) Arrhythmogenic right ventricular cardiomyopathy (ARVC) C) Hypertrophic cardiomyopathy (HCM) D) Restrictive or constrictive cardiomyopathy (RCM)
Ans: Hypertrophic cardiomyopathy (HCM) Feedback: With HCM, cardiac arrest (i.e., sudden cardiac death) may be the initial manifestation in young people, including athletes. DCM, ARVC, and RCM are not typically present in younger adults who appear otherwise healthy.
The nurse is creating a plan of care for a patient with a cardiomyopathy. What priority goal should underlie most of the assessments and interventions that are selected for this patient? A) Absence of complications B) Adherence to the self-care program C) Improved cardiac output D) Increased activity tolerance
Ans: Improved cardiac output Feedback: The priority nursing diagnosis of a patient with cardiomyopathy would include improved or maintained cardiac output. Regardless of the category and cause, cardiomyopathy may lead to severe heart failure, lethal dysrhythmias, and death. The pathophysiology of all cardiomyopathies is a series of progressive events that culminate in impaired cardiac output. Absence of complications, adherence to the self-care program, and increased activity tolerance should be included in the care plan, but they do not have the priority of improved cardiac output.
1. A perioperative nurse is caring for a postoperative patient. The patient has a shallow respiratory pattern and is reluctant to cough or to begin mobilizing. The nurse should address the patient's increased risk for what complication? A) Acute respiratory distress syndrome (ARDS) B) Atelectasis C) Aspiration D) Pulmonary embolism
B Feedback: A shallow, monotonous respiratory pattern coupled with immobility places the patient at an increased risk of developing atelectasis. These specific factors are less likely to result in pulmonary embolism or aspiration. ARDS involves an exaggerated inflammatory response and does not normally result from factors such as immobility and shallow breathing.
A patient with hypertrophic cardiomyopathy (HCM) has been admitted to the medical unit. During the nurse's admission interview, the patient states that she takes over-the-counter "water pills" on a regular basis. How should the nurse best respond to the fact that the patient has been taking diuretics? A) Encourage the patient to drink at least 2 liters of fluid daily. B) Increase the patient's oral sodium intake. C) Inform the care provider because diuretics are contraindicated. D) Ensure that the patient's fluid balance is monitored vigilantly.
Ans: Inform the care provider because diuretics are contraindicated. Feedback: Diuretics are contraindicated in patients with HCM, so the primary care provider should be made aware. Adjusting the patient's sodium or fluid intake or fluid monitoring does not address this important contraindication.
.The staff educator is presenting a workshop on valvular disorders. When discussing the pathophysiology of aortic regurgitation the educator points out the need to emphasize that aortic regurgitation causes what? A) Cardiac tamponade B) Left ventricular hypertrophy C) Right-sided heart failure D) Ventricular insufficiency
Ans: Left ventricular hypertrophy Feedback: Aortic regurgitation eventually causes left ventricular hypertrophy. In aortic regurgitation, blood from the aorta returns to the left ventricle during diastole in addition to the blood normally delivered by the left atrium. The left ventricle dilates, trying to accommodate the increased volume of blood. Aortic regurgitation does not cause cardiac tamponade, right-sided heart failure, or ventricular insufficiency.
The nurse is caring for a patient who is scheduled to undergo mechanical valve replacement. Patient education should include which of the following? A) Use of patient-controlled analgesia B) Long-term anticoagulant therapy C) Steroid therapy D) Use of IV diuretics
Ans: Long-term anticoagulant therapy Feedback: Mechanical valves necessitate long-term use of required anticoagulants. Diuretics and steroids are not indicated and patient-controlled analgesia may or may be not be used in the immediate postoperative period.
A patient newly admitted to the telemetry unit is experiencing progressive fatigue, hemoptysis, and dyspnea. Diagnostic testing has revealed that these signs and symptoms are attributable to pulmonary venous hypertension. What valvular disorder should the nurse anticipate being diagnosed in this patient? A) Aortic regurgitation B) Mitral stenosis C) Mitral valve prolapse D) Aortic stenosis
Ans: Mitral stenosis Feedback: The first symptom of mitral stenosis is often dyspnea on exertion as a result of pulmonary venous hypertension. Symptoms usually develop after the valve opening is reduced by one-third to one-half its usual size. Patients are likely to show progressive fatigue as a result of low cardiac output. The enlarged left atrium may create pressure on the left bronchial tree, resulting in a dry cough or wheezing. Patients may expectorate blood (i.e., hemoptysis) or experience palpitations, orthopnea, paroxysmal nocturnal dyspnea (PND), and repeated respiratory infections. Pulmonary venous hypertension is not typically caused by aortic regurgitation, mitral valve prolapse, or aortic stenosis.
The nurse is caring for a patient with right ventricular hypertrophy and consequently decreased right ventricular function. What valvular disorder may have contributed to this patient's diagnosis? A) Mitral valve regurgitation B) Aortic stenosis C) Aortic regurgitation D) Mitral valve stenosis
Ans: Mitral valve stenosis Feedback: Because no valve protects the pulmonary veins from the backward flow of blood from the atrium, the pulmonary circulation becomes congested. As a result, the right ventricle must contract against an abnormally high pulmonary arterial pressure and is subjected to excessive strain. Eventually, the right ventricle fails. None of the other listed valvular disorders has this pathophysiological effect.
A patient has been admitted with an aortic valve stenosis and has been scheduled for a balloon valvuloplasty in the cardiac catheterization lab later today. During the admission assessment, the patient tells the nurse he has thoracolumbar scoliosis and is concerned about lying down for any extended period of time. What is a priority action for the nurse? A) Arrange for an alternative bed. B) Measure the degree of the curvature. C) Notify the surgeon immediately. D) Note the scoliosis on the intake assessment.
Ans: Notify the surgeon immediately. Feedback: Most often used for mitral and aortic valve stenosis, balloon valvuloplasty is contraindicated for patients with left atrial or ventricular thrombus, severe aortic root dilation, significant mitral valve regurgitation, thoracolumbar scoliosis, rotation of the great vessels, and other cardiac conditions that require open heart surgery. Therefore notifying the physician would be the priority over further physical assessment. An alternative bed would be unnecessary and documentation is not a sufficient response.
The nurse is preparing a patient for cardiac surgery. During the procedure, the patient's heart will be removed and a donor heart implanted at the vena cava and pulmonary veins. What procedure will this patient undergo? A) Orthotopic transplant B) Xenograft C) Heterotropic transplant D) Homograft
Ans: Orthotopic transplant Feedback: Orthotopic transplantation is the most common surgical procedure for cardiac transplantation. The recipient's heart is removed, and the donor heart is implanted at the vena cava and pulmonary veins. Some surgeons still prefer to remove the recipient's heart, leaving a portion of the recipient's atria (with the vena cava and pulmonary veins) in place. Homografts, or allografts (i.e., human valves), are obtained from cadaver tissue donations and are used for aortic and pulmonic valve replacement. Xenografts and heterotropic transplantation are not terms used to describe heart transplantation.
A cardiac surgery patient's new onset of signs and symptoms is suggestive of cardiac tamponade. As a member of the interdisciplinary team, what is the nurse's most appropriate action? A) Prepare to assist with pericardiocentesis. B) Reposition the patient into a prone position. C) Administer a dose of metoprolol. D) Administer a bolus of normal saline.
Ans: Prepare to assist with pericardiocentesis. Feedback: Cardiac tamponade requires immediate pericardiocentesis. Beta-blockers and fluid boluses will not relieve the pressure on the heart and prone positioning would likely exacerbate symptoms.
A community health nurse is presenting an educational event and is addressing several health problems, including rheumatic heart disease. What should the nurse describe as the most effective way to prevent rheumatic heart disease? A) Recognizing and promptly treating streptococcal infections B) Prophylactic use of calcium channel blockers in high-risk populations C) Adhering closely to the recommended child immunization schedule D) Smoking cessation
Ans: Recognizing and promptly treating streptococcal infections Feedback: Group A streptococcus can cause rheumatic heart fever, resulting in rheumatic endocarditis. Being aware of signs and symptoms of streptococcal infections, identifying them quickly, and treating them promptly, are the best preventative techniques for rheumatic endocarditis. Smoking cessation, immunizations, and calcium channel blockers will not prevent rheumatic heart disease.
The nurse is caring for a patient with mitral stenosis who is scheduled for a balloon valvuloplasty. The patient tells the nurse that he is unsure why the surgeon did not opt to replace his damaged valve rather than repairing it. What is an advantage of valvuloplasty that the nurse should cite? A) The procedure can be performed on an outpatient basis in a physician's office. B) Repaired valves tend to function longer than replaced valves. C) The procedure is not associated with a risk for infection. D) Lower doses of antirejection drugs are required than with valve replacement.
Ans: Repaired valves tend to function longer than replaced valves. Feedback: In general, valves that undergo valvuloplasty function longer than prosthetic valve replacements and patients do not require continuous anticoagulation. Valvuloplasty carries a risk of infection, like all surgical procedures, and it is not performed in a physician's office. Antirejection drugs are unnecessary because foreign tissue is not introduced.
The nurse is caring for a recent immigrant who has been diagnosed with mitral valve regurgitation. The nurse should know that in developing countries the most common cause of mitral valve regurgitation is what? A) A decrease in gamma globulins B) An insect bite C) Rheumatic heart disease and its sequelae D) Sepsis and its sequelae
Ans: Rheumatic heart disease and its sequelae Feedback: The most common cause of mitral valve regurgitation in developing countries is rheumatic heart disease and its sequelae.
A patient has undergone a successful heart transplant and has been discharged home with a medication regimen that includes cyclosporine and tacrolimus. In light of this patient's medication regimen, what nursing diagnosis should be prioritized? A) Risk for injury B) Risk for infection C) Risk for peripheral neurovascular dysfunction D) Risk for unstable blood glucose
Ans: Risk for infection Feedback: Immunosuppressants decrease the body's ability to resist infections, and a satisfactory balance must be achieved between suppressing rejection and avoiding infection. These drugs do not create a heightened risk of injury, neurovascular dysfunction, or unstable blood glucose levels.
A patient is admitted to the critical care unit (CCU) with a diagnosis of cardiomyopathy. When reviewing the patient's most recent laboratory results, the nurse should prioritize assessment of which of the following? A) Sodium B) AST, ALT, and bilirubin C) White blood cell differential D) BUN
Ans: Sodium Feedback: Sodium is the major electrolyte involved with cardiomyopathy. Cardiomyopathy often leads to heart failure which develops, in part, from fluid overload. Fluid overload is often associated with elevated sodium levels. Consequently, sodium levels are followed more closely than other important laboratory values, including BUN, leukocytes, and liver function tests.
A nurse is planning discharge health education for a patient who will soon undergo placement of a mechanical valve prosthesis. What aspect of health education should the nurse prioritize in anticipation of discharge? A) The need for long-term antibiotics B) The need for 7 to 10 days of bed rest C) Strategies for preventing atherosclerosis D) Strategies for infection prevention
Ans: Strategies for infection prevention Feedback: Patients with a mechanical valve prosthesis (including annuloplasty rings and other prosthetic materials used in valvuloplasty) require education to prevent infective endocarditis. Despite these infections risks, antibiotics are not used long term. Activity management is important, but extended bed rest is unnecessary. Valve replacement does not create a heightened risk for atherosclerosis.
.A patient who has undergone valve replacement surgery is being prepared for discharge home. Because the patient will be discharged with a prescription for warfarin (Coumadin), the nurse should educate the patient about which of the following? A) The need for regularly scheduled testing of the patient's International Normalized Ratio (INR) B) The need to learn to sleep in a semi-Fowler's position for the first 6 to 8 weeks to prevent emboli C) The need to avoid foods that contain vitamin K D) The need to take enteric-coated ASA on a daily basis
Ans: The need for regularly scheduled testing of the patient's International Normalized Ratio (INR) Feedback: Patients who take warfarin (Coumadin) after valve replacement have individualized target INRs; usually between 2 and 3.5 for mitral valve replacement and 1.8 and 2.2 for aortic valve replacement. Natural sources of vitamin K do not normally need to be avoided and ASA is not indicated. Sleeping upright is unnecessary.
The nurse is caring for a patient with acute pericarditis. What nursing management should be instituted to minimize complications? A) The nurse keeps the patient isolated to prevent nosocomial infections. B) The nurse encourages coughing and deep breathing. C) The nurse helps the patient with activities until the pain and fever subside. D) The nurse encourages increased fluid intake until the infection resolves.
Ans: The nurse helps the patient with activities until the pain and fever subside. Feedback: To minimize complications, the nurse helps the patient with activity restrictions until the pain and fever subside. As the patient's condition improves, the nurse encourages gradual increases of activity. Actions to minimize complications of acute pericarditis do not include keeping the patient isolated. Due to pain, coughing and deep breathing are not normally encouraged. An increase in fluid intake is not always necessary.
A patient with mitral valve prolapse is admitted for a scheduled bronchoscopy to investigate recent hemoptysis. The physician has ordered gentamicin to be taken before the procedure. What is the rationale for this? A) To prevent bacterial endocarditis B) To prevent hospital-acquired pneumonia C) To minimize the need for antibiotic use during the procedure D) To decrease the need for surgical asepsis
Ans: To prevent bacterial endocarditis Feedback: Antibiotic prophylaxis is recommended for high-risk patients immediately before and sometimes after the following invasive procedures, such as bronchoscopy. Gentamicin would not be given to prevent pneumonia, to avoid antibiotic use during the procedure, or to decrease the need for surgical asepsis.
The nurse is teaching a patient diagnosed with aortic stenosis appropriate strategies for attempting to relieve the symptom of angina without drugs. What should the nurse teach the patient? A) To eat a small meal before taking nitroglycerin B) To drink a glass of milk before taking nitroglycerin C) To engage in 15 minutes of light exercise before taking nitroglycerin D) To rest and relax before taking nitroglycerin
Ans: To rest and relax before taking nitroglycerin Feedback: The venous dilation that results from nitroglycerin decreases blood return to the heart, thus decreasing cardiac output and increasing the risk of syncope and decreased coronary artery blood flow. The nurse teaches the patient about the importance of attempting to relieve the symptoms of angina with rest and relaxation before taking nitroglycerin and to anticipate the potential adverse effects. Exercising, eating, and drinking are not recommended prior to using nitroglycerin.
10. A patient who has sustained a nondepressed skull fracture is admitted to the acute medical unit. Nursing care should include which of the following? A) Preparation for emergency craniotomy B) Watchful waiting and close monitoring C) Administration of inotropic drugs D) Fluid resuscitation
B
16. An elderly woman found with a head injury on the floor of her home is subsequently admitted to the neurologic ICU. What is the best rationale for the following physician orders: elevate the HOB; keep the head in neutral alignment with no neck flexion or head rotation; avoid sharp hip flexion? A) To decrease cerebral arterial pressure B) To avoid impeding venous outflow C) To prevent flexion contractures D) To prevent aspiration of stomach contents
B
2. A patient is brought to the trauma center by ambulance after sustaining a high cervical spinal cord injury 1 hours ago. Endotracheal intubation has been deemed necessary and the nurse is preparing to assist. What nursing diagnosis should the nurse associate with this procedure? A) Risk for impaired skin integrity B) Risk for injury C) Risk for autonomic dysreflexia D) Risk for suffocation
B
20. Following a spinal cord injury a patient is placed in halo traction. While performing pin site care, the nurse notes that one of the traction pins has become detached. The nurse would be correct in implementing what priority nursing action? A) Complete the pin site care to decrease risk of infection. B) Notify the neurosurgeon of the occurrence. C) Stabilize the head in a lateral position. D) Reattach the pin to prevent further head trauma.
B
25. A neurologic flow chart is often used to document the care of a patient with a traumatic brain injury. At what point in the patients care should the nurse begin to use a neurologic flow chart? A) When the patients condition begins to deteriorate B) As soon as the initial assessment is made C) At the beginning of each shift D) When there is a clinically significant change in the patients condition
B
3. A nurse is caring for a critically ill patient with autonomic dysreflexia. What clinical manifestations would the nurse expect in this patient? A) Respiratory distress and projectile vomiting B) Bradycardia and hypertension C) Tachycardia and agitation D) Third-spacing and hyperthermia
B
6. The staff educator is precepting a nurse new to the critical care unit when a patient with a T2 spinal cord injury is admitted. The patient is soon exhibiting manifestations of neurogenic shock. In addition to monitoring the patient closely, what would be the nurses most appropriate action? A) Prepare to transfuse packed red blood cells. B) Prepare for interventions to increase the patients BP. C) Place the patient in the Trendelenberg position. D) Prepare an ice bath to lower core body temperature
B
8. A patient with spinal cord injury has a nursing diagnosis of altered mobility and the nurse recognizes the increased the risk of deep vein thrombosis (DVT). Which of the following would be included as an appropriate nursing intervention to prevent a DVT from occurring? A) Placing the patient on a fluid restriction as ordered B) Applying thigh-high elastic stockings C) Administering an antifibrinolyic agent D) Assisting the patient with passive range of motion (PROM) exercises
B
38. A firefighter was trapped in a fire and is admitted to the ICU for smoke inhalation. After 12 hours, the firefighter is exhibiting signs of ARDS and is intubated. What other supportive measures are initiated in a patient with ARDS? A) Psychological counseling B) Nutritional support C) High-protein oral diet D) Occupational therapy
B Feedback: Aggressive, supportive care must be provided to compensate for the severe respiratory dysfunction. This supportive therapy almost always includes intubation and mechanical ventilation. In addition, circulatory support, adequate fluid volume, and nutritional support are important. Oral intake is contraindicated by intubation. Counseling and occupational therapy would not be priorities during the acute stage of ARDS.
15. The nurse is caring for a patient suspected of having ARDS. What is the most likely diagnostic test ordered in the early stages of this disease to differentiate the patient's symptoms from those of a cardiac etiology? A) Carboxyhemoglobin level B) Brain natriuretic peptide (BNP) level C) C-reactive protein (CRP) level D) Complete blood count
B Feedback: Common diagnostic tests performed for patients with potential ARDS include plasma brain natriuretic peptide (BNP) levels, echocardiography, and pulmonary artery catheterization. The BNP level is helpful in distinguishing ARDS from cardiogenic pulmonary edema. The carboxyhemoglobin level will be increased in a client with an inhalation injury, which commonly progresses into ARDS. CRP and CBC levels do not help differentiate from a cardiac problem.
28. A patient with thoracic trauma is admitted to the ICU. The nurse notes the patient's chest and neck are swollen and there is a crackling sensation when palpated. The nurse consequently identifies the presence of subcutaneous emphysema. If this condition becomes severe and threatens airway patency, what intervention is indicated? A) A chest tube B) A tracheostomy C) An endotracheal tube D) A feeding tube
B Feedback: In severe cases in which there is widespread subcutaneous emphysema, a tracheostomy is indicated if airway patency is threatened by pressure of the trapped air on the trachea. The other listed tubes would neither resolve the subcutaneous emphysema nor the consequent airway constriction.
22. A hospital has been the site of an increased incidence of hospital-acquired pneumonia (HAP). What is an important measure for the prevention of HAP? A) Administration of prophylactic antibiotics B) Administration of pneumococcal vaccine to vulnerable individuals C) Obtaining culture and sensitivity swabs from all newly admitted patients D) Administration of antiretroviral medications to patients over age 65
B Feedback: Pneumococcal vaccination reduces the incidence of pneumonia, hospitalizations for cardiac conditions, and deaths in the general older adult population. A onetime vaccination of pneumococcal polysaccharide vaccine (PPSV) is recommended for all patients 65 years of age or older and those with chronic diseases. Antibiotics are not given on a preventative basis and antiretroviral medications do not affect the most common causative microorganisms. Culture and sensitivity testing by swabbing is not performed for pneumonia since the microorganisms are found in sputum.
26. A patient is receiving thrombolytic therapy for the treatment of pulmonary emboli. What is the best way for the nurse to assess the patient's oxygenation status at the bedside? A) Obtain serial ABG samples. B) Monitor pulse oximetry readings. C) Test pulmonary function. D) Monitor incentive spirometry volumes.
B Feedback: The nurse assesses the patient with pulmonary emboli frequently for signs of hypoxemia and monitors the pulse oximetry values to evaluate the effectiveness of the oxygen therapy. ABGs are accurate indicators of oxygenation status, but are not analyzed at the bedside. PFTs and incentive spirometry volumes do not accurately reveal oxygenation status.
33. A patient who involved in a workplace accident suffered a penetrating wound of the chest that led to acute respiratory failure. What goal of treatment should the care team prioritize when planning this patient's care? A) Facilitation of long-term intubation B) Restoration of adequate gas exchange C) Attainment of effective coping D) Self-management of oxygen therapy
B Feedback: The objectives of treatment are to correct the underlying cause of respiratory failure and to restore adequate gas exchange in the lung. This is priority over coping and self-care. Long-term ventilation may or may not be indicated.
9. A new employee asks the occupational health nurse about measures to prevent inhalation exposure of the substances. Which statement by the nurse will decrease the patient's exposure risk to toxic substances? A) "Position a fan blowing on the toxic substances to prevent the substance from becoming stagnant in the air." B) "Wear protective attire and devices when working with a toxic substance." C) "Make sure that you keep your immunizations up to date to prevent respiratory diseases resulting from toxins." D) "Always wear a disposable paper face mask when you are working with inhalable toxins."
B Feedback: When working with toxic substances, the employee must wear or use protective devices such as face masks, hoods, or industrial respirators. Immunizations do not confer protection from toxins and a paper mask is normally insufficient protection. Never position a fan directly blowing on the toxic substance as it will disperse the fumes throughout the area.
19. The nurse is caring for a patient who is rapidly progressing toward brain death. The nurse should be aware of what cardinal signs of brain death? Select all that apply. A) Absence of pain response B) Apnea C) Coma D) Absence of brain stem reflexes E) Absence of deep tendon reflexes
B C D
A nurse is performing the initial assessment of a patient who has a recent diagnosis of systemic lupus erythematosus (SLE). What skin manifestation would the nurse expect to observe on inspection? A) Petechiae B) Butterfly rash C) Jaundice D) Skin sloughing
B) Butterfly Rash An acute cutaneous lesion consisting of a butterfly-shaped rash across the bridge of the nose and cheeks occurs in SLE. Petechiae are pinpoint skin hemorrhages, which are not a clinical manifestation of SLE. Patients with SLE do not typically experience jaundice or skin sloughing.
34. A patient is brought to the ED by ambulance after a motor vehicle accident in which the patient received blunt trauma to the chest. The patient is in acute respiratory failure, is intubated, and is transferred to the ICU. What parameters of care should the nurse monitor most closely? Select all that apply. A) Coping B) Level of consciousness C) Oral intake D) Arterial blood gases E) Vital signs
B, D, E Feedback: Patients are usually treated in the ICU. The nurse assesses the patient's respiratory status by monitoring the level of responsiveness, ABGs, pulse oximetry, and vital signs. Oral intake and coping are not immediate priorities during the acute stage of treatment, but would become more important later during recovery.
A nurse is providing care for a patient who has just been diagnosed as being in the early stage of rheumatoid arthritis. The nurse should anticipate the administration of which of the following? A) Hydromorphone (Dilaudid) B) Methotrexate (Rheumatrex) C) Allopurinol (Zyloprim) D) Prednisone
B. Methotrexate (Rheumatrex) In the past, a step-wise approach starting with NSAIDs was standard of care. However, evidence clearly documenting the benefits of early DMARD (methotrexate [Rheumatrex], antimalarials, leflunomide [Arava], or sulfasalazine [Azulfidine]) treatment has changed national guidelines for management. Now it is recommended that treatment with the non-biologic DMARDs begin within 3 months of disease Test Bank - Brunner & Suddarth's Textbook of Medical-Surgical Nursing 14e (Hinkle 2017) 729 onset. Allopurinol is used to treat gout. Opioids are not indicated in early RA. Prednisone is used in unremitting RA.
A patient with rheumatoid arthritis comes to the clinic complaining of pain in the joint of his right great toe and is eventually diagnosed with gout. When planning teaching for this patient, what management technique should the nurse emphasize? A)Take OTC calcium supplements consistently. B)Restrict consumption of foods high in purines. C)Ensure fluid intake of at least 4 liters per day. D)Restrict weight-bearing on right foot.
B. Restrict consumption of foods high in purines. Although severe dietary restriction is not necessary, the nurse should encourage the patient to restrict consumption of foods high in purines, especially organ meats. Calcium supplementation is not necessary and activity should be maintained as tolerated. Increased fluid intake is beneficial, but it is not necessary for the patient to consume more than 4 liters daily.
A 21-year-old male has just been diagnosed with a spondyloarthropathy. What will be a priority nursing intervention for this patient? A)Referral for assistive devices B)Teaching about symptom management C)Referral to classes to stop smoking D)Setting up an exercise program
B. Teaching about symptom management Major nursing interventions in the spondyloarthropathies are related to symptom management and maintenance of optimal functioning. This is a priority over the use of assistive devices, smoking cessation, and exercise programs, though these topics may be of importance for some patients.
A nurse is assessing a patient for risk factors known to contribute to osteoarthritis. What assessment finding would the nurse interpret as a risk factor? A)The patient has a 30 pack-year smoking history. B)The patients body mass index is 34 (obese). C)The patient has primary hypertension. D)The patient is 58 years old.
B. The patients body mass index is 34 (obese) Risk factors for osteoarthritis include obesity and previous joint damage. Risk factors of OA do not include smoking or hypertension. Incidence increases with age, but a patient who is 58 would not yet face a significantly heightened risk.
A nurse is planning patient education for a patient being discharged home with a diagnosis of rheumatoid arthritis. The patient has been prescribed antimalarials for treatment, so the nurse knows to teach the patient to self-monitor for what adverse effect? A) Tinnitus B) Visual changes C) Stomatitis D) Hirsutism
B. Visual Changes Antimalarials may cause visual changes; regular ophthalmologic examinations are necessary.Tinnitus is associated with salicylate therapy, stomatitis is associated with gold therapy, and hirsutism is associated with corticosteroid therapy.
1. The ED nurse is caring for a patient who has been brought in by ambulance after sustaining a fall at home. What physical assessment finding is suggestive of a basilar skull fracture? A) Epistaxis B) Periorbital edema C) Bruising over the mastoid D) Unilateral facial numbness
C
15. A patient is admitted to the neurologic ICU with a spinal cord injury. When assessing the patient the nurse notes there is a sudden depression of reflex activity in the spinal cord below the level of injury. What should the nurse suspect? A) Epidural hemorrhage B) Hypertensive emergency C) Spinal shock D) Hypovolemia
C
26. The nurse planning the care of a patient with head injuries is addressing the patients nursing diagnosis of sleep deprivation. What action should the nurse implement? A) Administer a benzodiazepine at bedtime each night. B) Do not disturb the patient between 2200 and 0600. C) Cluster overnight nursing activities to minimize disturbances. D) Ensure that the patient does not sleep during the day
C
35. The nurse caring for a patient with a spinal cord injury notes that the patient is exhibiting early signs and symptoms of disuse syndrome. Which of the following is the most appropriate nursing action? A) Limit the amount of assistance provided with ADLs. B) Collaborate with the physical therapist and immobilize the patients extremities temporarily. C) Increase the frequency of ROM exercises. D) Educate the patient about the importance of frequent position changes.
C
36. Splints have been ordered for a patient who is at risk of developing footdrop following a spinal cord injury. The nurse caring for this patient knows that the splints are removed and reapplied when? A) At the patients request B) Each morning and evening C) Every 2 hours D) One hour prior to mobility exercises
C
39. The nurse recognizes that a patient with a SCI is at risk for muscle spasticity. How can the nurse best prevent this complication of an SCI? A) Position the patient in a high Fowlers position when in bed. B) Support the knees with a pillow when the patient is in bed. C) Perform passive ROM exercises as ordered. D) Administer NSAIDs as ordered.
C
9. Paramedics have brought an intubated patient to the RD following a head injury due to accelerationdeceleration motor vehicle accident. Increased ICP is suspected. Appropriate nursing interventions would include which of the following? A) Keep the head of the bed (HOB) flat at all times. B) Teach the patient to perform the Valsalva maneuver. C) Administer benzodiazepines on a PRN basis. D) Perform endotracheal suctioning every hour.
C
8. The nurse is providing discharge teaching for a patient who developed a pulmonary embolism after total knee surgery. The patient has been converted from heparin to sodium warfarin (Coumadin) anticoagulant therapy. What should the nurse teach the client? A) Coumadin will continue to break up the clot over a period of weeks B) Coumadin must be taken concurrent with ASA to achieve anticoagulation. C) Anticoagulant therapy usually lasts between 3 and 6 months. D) He should take a vitamin supplement containing vitamin K
C Feedback: Anticoagulant therapy prevents further clot formation, but cannot be used to dissolve a clot. The therapy continues for approximately 3 to 6 months and is not combined with ASA. Vitamin K reverses the effect of anticoagulant therapy and normally should not be taken.
23. When assessing for substances that are known to harm workers' lungs, the occupational health nurse should assess their potential exposure to which of the following? A) Organic acids B) Propane C) Asbestos D) Gypsum
C Feedback: Asbestos is among the more common causes of pneumoconiosis. Organic acids, propane, and gypsum do not have this effect.
24. A patient presents to the ED stating she was in a boating accident about 3 hours ago. Now the patient has complaints of headache, fatigue, and the feeling that he "just can't breathe enough." The nurse notes that the patient is restless and tachycardic with an elevated blood pressure. This patient may be in the early stages of what respiratory problem? A) Pneumoconiosis B) Pleural effusion C) Acute respiratory failure D) Pneumonia
C Feedback: Early signs of acute respiratory failure are those associated with impaired oxygenation and may include restlessness, fatigue, headache, dyspnea, air hunger, tachycardia, and increased blood pressure. As the hypoxemia progresses, more obvious signs may be present, including confusion, lethargy, tachycardia, tachypnea, central cyanosis, diaphoresis, and, finally, respiratory arrest. Pneumonia is infectious and would not result from trauma. Pneumoconiosis results from exposure to occupational toxins. A pleural effusion does not cause this constellation of symptoms.
21. A client presents to the walk-in clinic complaining of a dry, irritating cough and production of a minute amount of mucus-like sputum. The patient complains of soreness in her chest in the sternal area. The nurse should suspect that the primary care provider will assess the patient for what health problem? A) Pleural effusion B) Pulmonary embolism C) Tracheobronchitis D) Tuberculosis
C Feedback: Initially, the patient with tracheobronchitis has a dry, irritating cough and expectorates a scant amount of mucoid sputum. The patient may report sternal soreness from coughing and have fever or chills, night sweats, headache, and general malaise. Pleural effusion and pulmonary embolism do not normally cause sputum production and would likely cause acute shortness of breath. Hemoptysis is characteristic of TB.
39. The nurse is reviewing the electronic health record of a patient with an empyema. What health problem in the patient's history is most likely to have caused the empyema? A) Smoking B) Asbestosis C) Pneumonia D) Lung cancer
C Feedback: Most empyemas occur as complications of bacterial pneumonia or lung abscess. Cancer, smoking, and asbestosis are not noted to be common causes.
25. The nurse is caring for a 46-year-old patient recently diagnosed with the early stages of lung cancer. The nurse is aware that the preferred method of treating patients with non-small cell tumors is what? A) Chemotherapy B) Radiation C) Surgical resection D) Bronchoscopic opening of the airway
C Feedback: Surgical resection is the preferred method of treating patients with localized non-small cell tumors with no evidence of metastatic spread and adequate cardiopulmonary function. The other listed treatment options may be considered, but surgery is preferred.
29. The nurse is caring for a patient in the ICU admitted with ARDS after exposure to toxic fumes from a hazardous spill at work. The patient has become hypotensive. What is the cause of this complication to the ARDS treatment? A) Pulmonary hypotension due to decreased cardiac output B) Severe and progressive pulmonary hypertension C) Hypovolemia secondary to leakage of fluid into the interstitial spaces D) Increased cardiac output from high levels of PEEP therapy
C Feedback: Systemic hypotension may occur in ARDS as a result of hypovolemia secondary to leakage of fluid into the interstitial spaces and depressed cardiac output from high levels of PEEP therapy. Pulmonary hypertension, not pulmonary hypotension, sometimes is a complication of ARDS, but it is not the cause of the patient becoming hypotensive.
35. A gerontologic nurse is teaching a group of medical nurses about the high incidence and mortality of pneumonia in older adults. What is a contributing factor to this that the nurse should describe? A) Older adults have less compliant lung tissue than younger adults. B) Older adults are not normally candidates for pneumococcal vaccination. C) Older adults often lack the classic signs and symptoms of pneumonia. D) Older adults often cannot tolerate the most common antibiotics used to treat pneumonia.
C Feedback: The diagnosis of pneumonia may be missed because the classic symptoms of cough, chest pain, sputum production, and fever may be absent or masked in older adult patients. Mortality from pneumonia in the elderly is not a result of limited antibiotic options or lower lung compliance. The pneumococcal vaccine is appropriate for older adults.
11. The nurse is caring for a patient who is receiving oxygen therapy for pneumonia. How should the nurse best assess whether the patient is hypoxemic? A) Assess the patient's level of consciousness (LOC). B) Assess the patient's extremities for signs of cyanosis. C) Assess the patient's oxygen saturation level. D) Review the patient's hemoglobin, hematocrit, and red blood cell levels.
C Feedback: The effectiveness of the patient's oxygen therapy is assessed by the ABG analysis or pulse oximetry. ABG results may not be readily available. Presence or absence of cyanosis is not an accurate indicator of oxygen effectiveness. The patient's LOC may be affected by hypoxia, but not every change in LOC is related to oxygenation. Hemoglobin, hematocrit, and red blood cell levels do not directly reflect current oxygenation status.
27. The nurse is caring for an 82-year-old patient with a diagnosis of tracheobronchitis. The patient begins complaining of right-sided chest pain that gets worse when he coughs or breathes deeply. Vital signs are within normal limits. What would you suspect this patient is experiencing? A) Traumatic pneumothorax B) Empyema C) Pleuritic pain D) Myocardial infarction
C Feedback: The key characteristic of pleuritic pain is its relationship to respiratory movement. Taking a deep breath, coughing, or sneezing worsens the pain. Pleuritic pain is limited in distribution rather than diffuse; it usually occurs only on one side. The pain may become minimal or absent when the breath is held. It may be localized or radiate to the shoulder or abdomen. Later, as pleural fluid develops, the pain decreases. The scenario does not indicate any trauma to the patient, so a traumatic pneumothorax is implausible. Empyema is unlikely as there is no fever indicative of infection. Myocardial infarction would affect the patient's vital signs profoundly.
36. A patient has just been diagnosed with lung cancer. After the physician discusses treatment options and leaves the room, the patient asks the nurse how the treatment is decided upon. What would be the nurse's best response? A) "The type of treatment depends on the patient's age and health status." B) "The type of treatment depends on what the patient wants when given the options." C) "The type of treatment depends on the cell type of the cancer, the stage of the cancer, and the patient's health status." D) "The type of treatment depends on the discussion between the patient and the physician of which treatment is best."
C Feedback: Treatment of lung cancer depends on the cell type, the stage of the disease, and the patient's physiologic status (particularly cardiac and pulmonary status). Treatment does not depend solely on the patient's age or the patient's preference between the different treatment modes. The decision about treatment does not primarily depend on a discussion between the patient and the physician of which treatment is best, though this discussion will take place.
A clinic nurse is caring for a patient diagnosed with rheumatoid arthritis (RA). The patient tells the nurse that she has not been taking her medication because she usually cannot remove the childproof medication lids. How can the nurse best facilitate the patients adherence to her medication regimen? A)Encourage the patient to store the bottles with their tops removed. B)Have a trusted family member take over the management of the patients medication regimen. C)Encourage her to have her pharmacy replace the tops with alternatives that are easier to open. D)Have the patient approach her primary care provider to explore medication alternatives.
C. Encourage her to have her pharmacy replace the tops with alternatives that are easier to open. The patients pharmacy will likely be able to facilitate a practical solution that preserves the patients independence while still fostering adherence to treatment. There should be no need to change medications, and storing open medication containers is unsafe. Delegating medications to a family member is likely unnecessary at this point and promotes dependence.
A patient with systemic lupus erythematosus (SLE) is preparing for discharge. The nurse knows that the patient has understood health education when the patient makes what statement? A)I'll make sure I get enough exposure to sunlight to keep up my vitamin D levels. B)I'll try to be as physically active as possible between flare-ups. C)I'll make sure to monitor my body temperature on a regular basis. D)I'll stop taking my steroids when I get relief from my symptoms.
C. I'll make sure to monitor my body temperature on a regular basis. Fever can signal an exacerbation and should be reported to the physician. Sunlight and other sources of ultraviolet light may precipitate severe skin reactions and exacerbate the disease. Fatigue can cause a flare-up of SLE. Patients should be encouraged to pace activities and plan rest periods. Corticosteroids must be gradually tapered because they can suppress the function of the adrenal gland. As well, these drugs should not be independently adjusted by the patient.
A patient with SLE has come to the clinic for a routine check-up. When auscultating the patients apical heart rate, the nurse notes the presence of a distinct scratching sound. What is the nurses most appropriate action? A)Reposition the patient and auscultate posteriorly. B)Document the presence of S3 and monitor the patient closely. C)Inform the primary care provider that a friction rub may be present. D)Inform the primary care provider that the patient may have pneumonia.
C. Inform the primary care provider that a friction rub may be present. Patients with SLE are susceptible to developing a pericardial friction rub, possibly associated with myocarditis and accompanying pleural effusions; this warrants prompt medical follow-up. This finding is not characteristic of pneumonia and does not constitute S3. Posterior auscultation is unlikely to yield additional meaningful data.
A nurse is performing the health history and physical assessment of a patient who has a diagnosis of rheumatoid arthritis (RA). What assessment finding is most consistent with the clinical presentation of RA? A)Cool joints with decreased range of motion B)Signs of systemic infection C)Joint stiffness, especially in the morning D)Visible atrophy of the knee and shoulder joints
C. Joint stiffness, especially in the morning In addition to joint pain and swelling, another classic sign of RA is joint stiffness, especially in the morning. Joints are typically swollen, not atrophied, and systemic infection does not accompany the disease. Joints are often warm rather than cool.
A nurse is educating a patient with gout about lifestyle modifications that can help control the signs and symptoms of the disease. What recommendation should the nurse make? A)Ensuring adequate rest B)Limiting exposure to sunlight C)Limiting intake of alcohol D)Smoking cessation
C. Limiting intake of alcohol Alcohol and red meat can precipitate an acute exacerbation of gout. Each of the other listed actions is consistent with good health, but none directly addresses the factors that exacerbate gout.
A patient with SLE asks the nurse why she has to come to the office so often for check-ups. What would be the nurses best response? A)Taking care of you in the best way involves seeing you face to face. B)Taking care of you in the best way involves making sure you are taking your medication the way it is ordered. C)Taking care of you in the best way involves monitoring your disease activity and how well the prescribed treatment is working. D)Taking care of you in the best way involves drawing blood work every month.
C. Taking care of you in the best way involves monitoring your disease activity and how well the prescribed treatment is working. The goals of treatment include preventing progressive loss of organ function, reducing the likelihood of acute disease, minimizing disease-related disabilities, and preventing complications from therapy. Management of SLE involves regular monitoring to assess disease activity and therapeutic effectiveness. Stating the benefit of face-to-face interaction does not answer the patients question. Blood work is not necessarily drawn monthly and assessing medication adherence is not the sole purpose of visits.
37. A patient in the ICU is status post embolectomy after a pulmonary embolus. What assessment parameter does the nurse monitor most closely on a patient who is postoperative following an embolectomy? A) Pupillary response B) Pressure in the vena cava C) White blood cell differential D) Pulmonary arterial pressure
D Feedback: If the patient has undergone surgical embolectomy, the nurse measures the patient's pulmonary arterial pressure and urinary output. Pressure is not monitored in a patient's vena cava. White cell levels and pupillary responses would be monitored, but not to the extent of the patient's pulmonary arterial pressure.
5. The nurse caring for a patient recently diagnosed with lung disease encourages the patient not to smoke. What is the primary rationale behind this nursing action? A) Smoking decreases the amount of mucus production. B) Smoke particles compete for binding sites on hemoglobin. C) Smoking causes atrophy of the alveoli. D) Smoking damages the ciliary cleansing mechanism.
D Feedback: In addition to irritating the mucous cells of the bronchi and inhibiting the function of alveolar macrophage (scavenger) cells, smoking damages the ciliary cleansing mechanism of the respiratory tract. Smoking also increases the amount of mucus production and distends the alveoli in the lungs. It reduces the oxygen-carrying capacity of hemoglobin, but not by directly competing for binding sites.
40. An 87-year-old patient has been hospitalized with pneumonia. Which nursing action would be a priority in this patient's plan of care? A) Nasogastric intubation B) Administration of probiotic supplements C) Bedrest D) Cautious hydration
D Feedback: Supportive treatment of pneumonia in the elderly includes hydration (with caution and with frequent assessment because of the risk of fluid overload in the elderly); supplemental oxygen therapy; and assistance with deep breathing, coughing, frequent position changes, and early ambulation. Mobility is not normally discouraged and an NG tube is not necessary in most cases. Probiotics may or may not be prescribed for the patient.
32. A 54-year-old man has just been diagnosed with small cell lung cancer. The patient asks the nurse why the doctor is not offering surgery as a treatment for his cancer. What fact about lung cancer treatment should inform the nurse's response? A) The cells in small cell cancer of the lung are not large enough to visualize in surgery. B) Small cell lung cancer is self-limiting in many patients and surgery should be delayed. C) Patients with small cell lung cancer are not normally stable enough to survive surgery. D) Small cell cancer of the lung grows rapidly and metastasizes early and extensively.
D Feedback: Surgery is primarily used for NSCLCs, because small cell cancer of the lung grows rapidly and metastasizes early and extensively. Difficult visualization and a patient's medical instability are not the limiting factors. Lung cancer is not a self-limiting disease.
30. The home care nurse is monitoring a patient discharged home after resolution of a pulmonary embolus. For what potential complication would the home care nurse be most closely monitoring this patient? A) Signs and symptoms of pulmonary infection B) Swallowing ability and signs of aspiration C) Activity level and role performance D) Residual effects of compromised oxygenation
D Feedback: The home care nurse should monitor the patient for residual effects of the PE, which involved a severe disruption in respiration and oxygenation. PE has a noninfectious etiology; pneumonia is not impossible, but it is a less likely sequela. Swallowing ability is unlikely to be affected; activity level is important, but secondary to the effects of deoxygenation.
A patient is suspected of having rheumatoid arthritis and her diagnostic regimen includes aspiration of synovial fluid from the knee for a definitive diagnosis. The nurse knows that which of the following procedures will be involved? A) Angiography B) Myelography C) Paracentesis D) Arthocentesis
D. Arthrocentesis Arthrocentesis involves needle aspiration of synovial fluid. Angiography is an x-ray study of circulation with a contrast agent injected into a selected artery. Myelography is an x-ray of the spinal subarachnoid space taken after the injection of a contrast agent into the spinal subarachnoid space through a lumbar puncture. Paracentesis is removal of fluid (ascites) from the peritoneal cavity through a small surgical incision or puncture made through the abdominal wall under sterile conditions.
A nurse is providing care for a patient who has a recent diagnosis of giant cell arteritis (GCA). What aspect of physical assessment should the nurse prioritize? A)Assessment for subtle signs of bleeding disorders B)Assessment of the metatarsal joints and phalangeal joints C)Assessment for thoracic pain that is exacerbated by activity D)Assessment for headaches and jaw pain
D. Assessment for headaches and jaw pain. Assessment of the patient with GCA focuses on musculoskeletal tenderness, weakness, and decreased function. Careful attention should be directed toward assessing the head (for changes in vision, headaches, and jaw claudication). There is not a particular clinical focus on the potential for bleeding, hand and foot pain, or thoracic pain.
Allopurinol (Zyloprim) has been ordered for a patient receiving treatment for gout. The nurse caring for this patient knows to assess the patient for bone marrow suppression, which may be manifested by which of the following diagnostic findings? A)Hyperuricemia B)Increased erythrocyte sedimentation rate C)Elevated serum creatinine D)Decreased platelets
D. Decreased platelets Thrombocytopenia occurs in bone marrow suppression. Hyperuricemia occurs in gout, but is not caused by bone marrow suppression. Increased erythrocyte sedimentation rate may occur from inflammation associated with gout, but is not related to bone marrow suppression. An elevated serum creatinine level may indicate renal damage, but this is not associated with the use of allopurinol.
A nurse is assessing a patient with rheumatoid arthritis. The patient expresses his intent to pursue complementary and alternative therapies. What fact should underlie the nurses response to the patient? A)New evidence shows CAM to be as effective as medical treatment. B)CAM therapies negate many of the benefits of medications. C)CAM therapies typically do more harm than good. D)Evidence shows minimal benefits from most CAM therapies.
D. Evidence shows minimal benefits from most CAM therapies. A recent systematic review of complementary and alternative medicine (CAM) examined the efficacy of herbal medicine, acupuncture, Tai chi and biofeedback for the treatment of rheumatoid arthritis and osteoarthritis. Although acupuncture treatment for pain management showed some promise, in all modalities the evidence was ambiguous. There is not enough evidence of the effectiveness of CAM and more rigorous research is needed.
A 40-year-old woman was diagnosed with Raynauds phenomenon several years earlier and has sought care because of a progressive worsening of her symptoms. The patient also states that many of her skin surfaces are stiff, like the skin is being stretched from all directions. The nurse should recognize the need for medical referral for the assessment of what health problem? A)Giant cell arteritis (GCA) B)Fibromyalgia (FM) C)Rheumatoid arthritis (RA) D)Scleroderma
D. Scleroderma Scleroderma starts insidiously with Raynauds phenomenon and swelling in the hands. Later, the skin and the subcutaneous tissues become increasingly hard and rigid and cannot be pinched up from the underlying structures. This progression of symptoms is inconsistent with GCA, FM, or RA.
A patient is diagnosed with giant cell arteritis (GCA) and is placed on corticosteroids. A concern for this patient is that he will stop taking the medication as soon as he starts to feel better. Why must the nurse emphasize the need for continued adherence to the prescribed medication? A)To avoid complications such as venous thromboembolism B)To avoid the progression to osteoporosis C)To avoid the progression of GCA to degenerative joint disease D)To avoid complications such as blindness
D. To avoid complications such as blindness The nurse must emphasize to the patient the need for continued adherence to the prescribed medication regimen to avoid complications of giant cell arteritis, such as blindness. VTE, OP, and degenerative joint disease are not among the most common complications for GCA.
35. A patient with Cushing syndrome as a result of a pituitary tumor has been admitted for a transsphenoidal hypophysectomy. What would be most important for the nurse to monitor before, during, and after surgery? A) Blood glucose B) Assessment of urine for blood C) Weight D) Oral temperature
ans: A Feedback: Before, during, and after this surgery, blood glucose monitoring and assessment of stools for blood are carried out. The patients blood sugar is more likely to be volatile than body weight or temperature. Hematuria is not a common complication.
28. Following an addisonian crisis, a patients adrenal function has been gradually regained. The nurse should ensure that the patient knows about the need for supplementary glucocorticoid therapy in which of the following circumstances? A) Episodes of high psychosocial stress B) Periods of dehydration C) Episodes of physical exertion D) Administration of a vaccine
ans: A Feedback: During stressful procedures or significant illnesses, additional supplementary therapy with glucocorticoids is required to prevent addisonian crisis. Physical activity, dehydration and vaccine administration would not normally be sufficiently demanding such to require glucocorticoids.
23. A nurse works in a walk-in clinic. The nurse recognizes that certain patients are at higher risk for different disorders than other patients. What patient is at a greater risk for the development of hypothyroidism? A) A 75-year-old female patient with osteoporosis B) A 50-year-old male patient who is obese C) A 45-year-old female patient who used oral contraceptives D) A 25-year-old male patient who uses recreational drugs
ans: A Feedback: Even though osteoporosis is not a risk factor for hypothyroidism, the condition occurs most frequently in older women.
1. The nurse is caring for a patient diagnosed with hypothyroidism secondary to Hashimotos thyroiditis. When assessing this patient, what sign or symptom would the nurse expect? A) Fatigue B) Bulging eyes C) Palpitations D) Flushed skin
ans: A Feedback: Symptoms of hypothyroidism include extreme fatigue, hair loss, brittle nails, dry skin, voice huskiness or hoarseness, menstrual disturbance, and numbness and tingling of the fingers. Bulging eyes, palpitations, and flushed skin would be signs and symptoms of hyperthyroidism.
32. The nurse providing care for a patient with Cushing syndrome has identified the nursing diagnosis of risk for injury related to weakness. How should the nurse best reduce this risk? A) Establish falls prevention measures. B) Encourage bed rest whenever possible. C) Encourage the use of assistive devices. D) Provide constant supervision.
ans: A Feedback: The nurse should take action to prevent the patients risk for falls. Bed rest carries too many harmful effects, however, and assistive devices may or may not be necessary. Constant supervision is not normally required or practicable.
10. You are developing a care plan for a patient with Cushing syndrome. What nursing diagnosis would have the highest priority in this care plan? A) Risk for injury related to weakness B) Ineffective breathing pattern related to muscle weakness C) Risk for loneliness related to disturbed body image D) Autonomic dysreflexia related to neurologic changes
ans: A Feedback: The nursing priority is to decrease the risk of injury by establishing a protective environment. The patient who is weak may require assistance from the nurse in ambulating to prevent falls or bumping corners or furniture. The patients breathing will not be affected and autonomic dysreflexia is not a plausible risk. Loneliness may or may not be an issue for the patient, but safety is a priority.
13. A patient presents at the walk-in clinic complaining of diarrhea and vomiting. The patient has a documented history of adrenal insufficiency. Considering the patients history and current symptoms, the nurse should anticipate that the patient will be instructed to do which of the following? A) Increase his intake of sodium until the GI symptoms improve. B) Increase his intake of potassium until the GI symptoms improve. C) Increase his intake of glucose until the GI symptoms improve. D) Increase his intake of calcium until the GI symptoms improve.
ans: A Feedback: The patient will need to supplement dietary intake with added salt during episodes of GI losses of fluid through vomiting and diarrhea to prevent the onset of addisonian crisis. While the patient may experience the loss of other electrolytes, the major concern is the replacement of lost sodium.
31. A patient is undergoing testing for suspected adrenocortical insufficiency. The care team should ensure that the patient has been assessed for the most common cause of adrenocortical insufficiency. What is the most common cause of this health problem? A) Therapeutic use of corticosteroids B) Pheochromocytoma C) Inadequate secretion of ACTH D) Adrenal tumor
ans: A Feedback: Therapeutic use of corticosteroids is the most common cause of adrenocortical insufficiency. The other options also cause adrenocortical insufficiency, but they are not the most common causes.
3. A patient with thyroid cancer has undergone surgery and a significant amount of parathyroid tissue has been removed. The nurse caring for the patient should prioritize what question when addressing potential complications? A) Do you feel any muscle twitches or spasms? B) Do you feel flushed or sweaty? C) Are you experiencing any dizziness or lightheadedness? D) Are you having any pain that seems to be radiating from your bones?
ans: A Feedback: As the blood calcium level falls, hyperirritability of the nerves occurs, with spasms of the hands and feet and muscle twitching. This is characteristic of hypoparathyroidism. Flushing, diaphoresis, dizziness, and pain are atypical signs of the resulting hypocalcemia.
11. The nurse is performing a shift assessment of a patient with aldosteronism. What assessments should the nurse include? Select all that apply. A) Urine output B) Signs or symptoms of venous thromboembolism C) Peripheral pulses D) Blood pressure E) Skin integrity
ans: A, D Feedback: The principal action of aldosterone is to conserve body sodium. Alterations in aldosterone levels consequently affect urine output and BP. The patients peripheral pulses, risk of VTE, and skin integrity are not typically affected by aldosteronism.
36. What should the nurse teach a patient on corticosteroid therapy in order to reduce the patients risk of adrenal insufficiency? A) Take the medication late in the day to mimic the body's natural rhythms. B) Always have enough medication on hand to avoid running out. C) Skip up to 2 doses in cases of illness involving nausea. D) Take up to 1 extra dose per day during times of stress.
ans: B Feedback: The patient and family should be informed that acute adrenal insufficiency and underlying symptoms will recur if corticosteroid therapy is stopped abruptly without medical supervision. The patient should be instructed to have an adequate supply of the corticosteroid medication always available to avoid - 994 running out. Doses should not be skipped or added without explicit instructions to do so. Corticosteroids should normally be taken in the morning to mimic natural rhythms.
17. The nurse is planning the care of a patient with hyperthyroidism. What should the nurse specify in the patients meal plan? A) A clear liquid diet, high in nutrients B) Small, frequent meals, high in protein and calories C) Three large, bland meals a day D) A diet high in fiber and plant-sourced fat
ans: B Feedback: A patient with hyperthyroidism has an increased appetite. The patient should be counseled to consume several small, well-balanced meals. High-calorie, high-protein foods are encouraged. A clear liquid diet would not satisfy the patients caloric or hunger needs. A diet rich in fiber and fat should be avoided because these foods may lead to GI upset or increase peristalsis.
5. The nurse is caring for a patient with Addisons disease who is scheduled for discharge. When teaching the patient about hormone replacement therapy, the nurse should address what topic? A) The possibility of precipitous weight gain B) The need for lifelong steroid replacement C) The need to match the daily steroid dose to immediate symptoms D) The importance of monitoring liver function
ans: B Feedback: Because of the need for lifelong replacement of adrenal cortex hormones to prevent addisonian crises, the patient and family members receive explicit education about the rationale for replacement therapy and proper dosage. Doses are not adjusted on a short-term basis. Weight gain and hepatotoxicity are not common adverse effects.
15. While assisting with the surgical removal of an adrenal tumor, the OR nurse is aware that the patients vital signs may change upon manipulation of the tumor. What vital sign changes would the nurse expect to see? A) Hyperthermia and tachypnea B) Hypertension and heart rate changes C) Hypotension and hypothermia D) Hyperthermia and bradycardia
ans: B Feedback: Manipulation of the tumor during surgical excision may cause release of stored epinephrine and norepinephrine, with marked increases in BP and changes in heart rate. The use of sodium nitroprusside and alpha-adrenergic blocking agents may be required during and after surgery. While other vital sign changes may occur related to surgical complications, the most common changes are related to hypertension and changes in the heart rate.
39. A patient who has been taking corticosteroids for several months has been experiencing muscle wasting. The patient has asked the nurse for suggestions to address this adverse effect. What should the nurse recommend? A) Activity limitation to conserve energy B) Consumption of a high-protein diet C) Use of OTC vitamin D and calcium supplements D) Passive range-of-motion exercises
ans: B Feedback: Muscle wasting can be partly addressed through increased protein intake. Passive ROM exercises maintain flexibility, but do not build muscle mass. Vitamin D and calcium supplements do not decrease muscle wasting. Activity limitation would exacerbate the problem.
22. The physician has ordered a fluid deprivation test for a patient suspected of having diabetes insipidus. During the test, the nurse should prioritize what assessments? A) Temperature and oxygen saturation B) Heart rate and BP C) Breath sounds and bowel sounds D) Color, warmth, movement, and sensation of extremities
ans: B Feedback: The fluid deprivation test is carried out by withholding fluids for 8 to 12 hours or until 3% to 5% of the body weight is lost. The patients condition needs to be monitored frequently during the test, and the test is terminated if tachycardia, excessive weight loss, or hypotension develops. Consequently, BP and heart rate monitoring are priorities over the other listed assessments.
18. A patient with a diagnosis of syndrome of inappropriate antidiuretic hormone secretion (SIADH) is being cared for on the critical care unit. The priority nursing diagnosis for a patient with this condition is what? A) Risk for peripheral neurovascular dysfunction B) Excess fluid volume C) Hypothermia D) Ineffective airway clearance
ans: B Feedback: The priority nursing diagnosis for a patient with SIADH is excess fluid volume, as the patient retains fluids and develops a sodium deficiency. Restricting fluid intake is a typical intervention for managing this syndrome. Temperature imbalances are not associated with SIADH. The patient is not at risk for neurovascular dysfunction or a compromised airway.
26. The nurses assessment of a patient with thyroidectomy suggests tetany and a review of the most recent blood work corroborate this finding. The nurse should prepare to administer what intervention? A) Oral calcium chloride and vitamin D B) IV calcium gluconate C) STAT levothyroxine D) Administration of parathyroid hormone (PTH)
ans: B Feedback: When hypocalcemia and tetany occur after a thyroidectomy, the immediate treatment is administration of IV calcium gluconate. This has a much faster therapeutic effect than PO calcium or vitamin D supplements. PTH and levothyroxine are not used to treat this complication.
37. The nurse is caring for a patient at risk for an addisonian crisis. For what associated signs and symptoms should the nurse monitor the patient? Select all that apply. A) Epistaxis B) Pallor C) Rapid respiratory rate D) Bounding pulse E) Hypotension
ans: B, C, E Feedback: The patient at risk is monitored for signs and symptoms indicative of addisonian crisis, which can include shock; hypotension; rapid, weak pulse; rapid respiratory rate; pallor; and extreme weakness. Epistaxis and a bounding pulse are not symptoms or signs of an addisonian crisis.
25. A patient has been admitted to the critical care unit with a diagnosis of thyroid storm. What interventions should the nurse include in this patients immediate care? Select all that apply. A) Administering diuretics to prevent fluid overload B) Administering beta blockers to reduce heart rate C) Administering insulin to reduce blood glucose levels D) Applying interventions to reduce the patients temperature E) Administering corticosteroids
ans: B, D Feedback: Thyroid storm necessitates interventions to reduce heart rate and temperature. Diuretics, insulin, and steroids are not indicated to address the manifestations of this health problem.
20. The nurse is assessing a patient diagnosed with Graves disease. What physical characteristics of Graves disease would the nurse expect to find? A) Hair loss B) Moon face C) Bulging eyes D) Fatigue
ans: C Feedback: Clinical manifestations of the endocrine disorder Graves disease include exophthalmos (bulging eyes) and fine tremor in the hands. Graves disease is not associated with hair loss, a moon face, or fatigue.
29. A 30 year-old female patient has been diagnosed with Cushing syndrome. What psychosocial nursing diagnosis should the nurse most likely prioritize when planning the patients care? A) Decisional conflict related to treatment options B) Spiritual distress related to changes in cognitive function C) Disturbed body image related to changes in physical appearance D) Powerlessness related to disease progression
ans: C Feedback: Cushing syndrome causes characteristic physical changes that are likely to result in disturbed body image. Decisional conflict and powerless may exist, but disturbed body image is more likely to be present. Cognitive changes take place in patients with Cushing syndrome, but these may or may not cause spiritual distress.
9. The nurse caring for a patient with Cushing syndrome is describing the dexamethasone suppression test scheduled for tomorrow. What does the nurse explain that this test will involve? A) Administration of dexamethasone orally, followed by a plasma cortisol level every hour for 3 hours B) Administration of dexamethasone IV, followed by an x-ray of the adrenal glands C) Administration of dexamethasone orally at 11 PM, and a plasma cortisol level at 8 AM the next morning D) Administration of dexamethasone intravenously, followed by a plasma cortisol level 3 hours after the drug is administered
ans: C Feedback: Dexamethasone (1 mg) is administered orally at 11 PM, and a plasma cortisol level is obtained at 8 AM the next morning. This test can be performed on an outpatient basis and is the most widely used and sensitive screening test for diagnosis of pituitary and adrenal causes of Cushing syndrome.
16. A patient has returned to the floor after having a thyroidectomy for thyroid cancer. The nurse knows that sometimes during thyroid surgery the parathyroid glands can be injured or removed. What laboratory finding may be an early indication of parathyroid gland injury or removal? A) Hyponatremia B) Hypophosphatemia C) Hypocalcemia D) Hypokalemia
ans: C Feedback: Injury or removal of the parathyroid glands may produce a disturbance in calcium metabolism and result in a decline of calcium levels (hypocalcemia). As the blood calcium levels fall, hyperirritability of the nerves occurs, with spasms of the hands and feet and muscle twitching. This group of symptoms is known as tetany and must be reported to the physician immediately, because laryngospasm may occur and obstruct the airway. Hypophosphatemia, hyponatremia, and hypokalemia are not expected responses to parathyroid injury or removal. In fact, parathyroid removal or injury that results in hypocalcemia may lead to hyperphosphatemia.
4. The nurse is caring for a patient with a diagnosis of Addisons disease. What sign or symptom is most closely associated with this health problem? A) Truncal obesity B) Hypertension C) Muscle weakness D) Moon face
ans: C Feedback: Patients with Addisons disease demonstrate muscular weakness, anorexia, gastrointestinal symptoms, fatigue, emaciation, dark pigmentation of the skin, and hypotension. Patients with Cushing syndrome demonstrate truncal obesity, moon face, acne, abdominal striae, and hypertension.
2. A patient has been admitted to the post-surgical unit following a thyroidectomy. To promote comfort and safety, how should the nurse best position the patient? A) Side-lying (lateral) with one pillow under the head B) Head of the bed elevated 30 degrees and no pillows placed under the head C) Semi-Fowlers with the head supported on two pillows D) Supine, with a small roll supporting the neck
ans: C Feedback: When moving and turning the patient, the nurse carefully supports the patients head and avoids tension on the sutures. The most comfortable position is the semi-Fowlers position, with the head elevated and supported by pillows.
38. A patient has been assessed for aldosteronism and has recently begun treatment. What are priority areas for assessment that the nurse should frequently address? Select all that apply. A) Pupillary response B) Creatinine and BUN levels C) Potassium level D) Peripheral pulses E) BP
ans: C, E Feedback: Patients with aldosteronism exhibit a profound decline in the serum levels of potassium, and hypertension is the most prominent and almost universal sign of aldosteronism. Pupillary response, peripheral pulses, and renal function are not directly affected.
27. A patient has been taking prednisone for several weeks after experiencing a hypersensitivity reaction. To prevent adrenal insufficiency, the nurse should ensure that the patient knows to do which of the following? A) Take the drug concurrent with levothyroxine (Synthroid). B) Take each dose of prednisone with a dose of calcium chloride. C) Gradually replace the prednisone with an OTC alternative. D) Slowly taper down the dose of prednisone, as ordered.
ans: D Feedback: Corticosteroid dosages are reduced gradually (tapered) to allow normal adrenal function to return and to prevent steroid-induced adrenal insufficiency. There are no OTC substitutes for prednisone and neither calcium chloride nor levothyroxine addresses the risk of adrenal insufficiency.
19. A patient with hypofunction of the adrenal cortex has been admitted to the medical unit. What would the nurse most likely find when assessing this patient? A) Increased body temperature B) Jaundice C) Copious urine output D) Decreased BP
ans: D Feedback: Decreased BP may occur with hypofunction of the adrenal cortex. Decreased function of the adrenal cortex does not affect the patients body temperature, urine output, or skin tone.
30. A patient with pheochromocytoma has been admitted for an adrenalectomy to be performed the following day. To prevent complications, the nurse should anticipate preoperative administration of which of the following? A) IV antibiotics B) Oral antihypertensives C) Parenteral nutrition D) IV corticosteroids
ans: D Feedback: IV administration of corticosteroids (methylprednisolone sodium succinate [Solu-Medrol]) may begin on the evening before surgery and continue during the early postoperative period to prevent adrenal insufficiency. Antibiotics, antihypertensives, and parenteral nutrition do not prevent adrenal insufficiency or other common complications of adrenalectomy.
24. A patient with a recent diagnosis of hypothyroidism is being treated for an unrelated injury. When administering medications to the patient, the nurse should know that the patients diminished thyroid function may have what effect? A) Anaphylaxis B) Nausea and vomiting C) Increased risk of drug interactions D) Prolonged duration of effect
ans: D Feedback: In all patients with hypothyroidism, the effects of analgesic agents, sedatives, and anesthetic agents are prolonged. There is no direct increase in the risk of anaphylaxis, nausea, or drug interactions, although these may potentially result from the prolonged half-life of drugs.
12. The home care nurse is conducting patient teaching with a patient on corticosteroid therapy. To achieve consistency with the bodys natural secretion of cortisol, when would the home care nurse instruct the patient to take his or her corticosteroids? A) In the evening between 4 PM and 6 PM B) Prior to going to sleep at night C) At noon every day D) In the morning between 7 AM and 8 AM
ans: D Feedback: In keeping with the natural secretion of cortisol, the best time of day for the total corticosteroid dose is in the morning from 7 to 8 AM. Large-dose therapy at 8 AM, when the adrenal gland is most active, produces maximal suppression of the gland. Also, a large 8 AM dose is more physiologic because it allows the body to escape effects of the steroids from 4 PM to 6 AM, when serum levels are normally low, thus minimizing cushingoid effects.
14. The nurse is caring for a patient with hyperparathyroidism. What level of activity would the nurse expect to promote? A) Complete bed rest B) Bed rest with bathroom privileges C) Out of bed (OOB) to the chair twice a day D) Ambulation and activity as tolerated
ans: D Feedback: Mobility, with walking or use of a rocking chair for those with limited mobility, is encouraged as much as possible because bones subjected to normal stress give up less calcium. Best rest should be discouraged because it increases calcium excretion and the risk of renal calculi. Limiting the patient to getting out of bed only a few times a day also increases calcium excretion and the associated risks.
40. The nurse is providing care for an older adult patient whose current medication regimen includes levothyroxine (Synthroid). As a result, the nurse should be aware of the heightened risk of adverse effects when administering an IV dose of what medication? A) A fluoroquinalone antibiotic B) A loop diuretic C) A proton pump inhibitor (PPI) D) A benzodiazepine
ans: D Feedback: Oral thyroid hormones interact with many other medications. Even in small IV doses, hypnotic and sedative agents may induce profound somnolence, lasting far longer than anticipated and leading to narcosis (stupor like condition). Furthermore, they are likely to cause respiratory depression, which can easily be fatal because of decreased respiratory reserve and alveolar hypoventilation. Antibiotics, PPIs and diuretics do not cause the same risk.
34. A patient on corticosteroid therapy needs to be taught that a course of corticosteroids of 2 weeks duration can suppress the adrenal cortex for how long? A) Up to 4 weeks B) Up to 3 months C) Up to 9 months D) Up to 1 year
ans: D Feedback: Suppression of the adrenal cortex may persist up to 1 year after a course of corticosteroids of only 2 weeks duration.
12. A patient is admitted to the neurologic ICU with a spinal cord injury. In writing the patients care plan, the nurse specifies that contractures can best be prevented by what action? A) Repositioning the patient every 2 hours B) Initiating range-of-motion exercises (ROM) as soon as the patient initiates C) Initiating (ROM) exercises as soon as possible after the injury D) Performing ROM exercises once a day
c
40. A patient is admitted to the neurologic ICU with a C4 spinal cord injury. When writing the plan of care for this patient, which of the following nursing diagnoses would the nurse prioritize in the immediate care of this patient? A) Risk for impaired skin integrity related to immobility and sensory loss B) Impaired physical mobility related to loss of motor function C) Ineffective breathing patterns related to weakness of the intercostal muscles D) Urinary retention related to inability to void spontaneously
c